Jump to content

Wikipedia:Reference desk/Humanities: Difference between revisions

From Wikipedia, the free encyclopedia
Content deleted Content added
Scsbot (talk | contribs)
edited by robot: adding date header(s)
Line 875: Line 875:


:: There is a list of such organizations on [http://julieslist.homestead.com/LegalResources.html this page]. [http://www.gaylawnet.com/attorneys/us_mi.htm Here is a similar list]. [http://www.equalitymi.org/report Equality Michigan] is a referral service for anyone with questions such as yours. [[User:Taknaran|Taknaran]] ([[User talk:Taknaran|talk]]) 10:59, 16 October 2012 (UTC)
:: There is a list of such organizations on [http://julieslist.homestead.com/LegalResources.html this page]. [http://www.gaylawnet.com/attorneys/us_mi.htm Here is a similar list]. [http://www.equalitymi.org/report Equality Michigan] is a referral service for anyone with questions such as yours. [[User:Taknaran|Taknaran]] ([[User talk:Taknaran|talk]]) 10:59, 16 October 2012 (UTC)

Now that I've thought about it for a bit, I'm thinking of contacting the parent company to see their anti-discrimination policy. Odds are it mentions sexual orientation because it is in the state law. If it does, I'll see if I can get them to work on making sure that all of their companies adopt a similar policy. [[Special:Contributions/108.194.140.240|108.194.140.240]] ([[User talk:108.194.140.240|talk]]) 00:13, 17 October 2012 (UTC)


== Marbles and Jews ==
== Marbles and Jews ==

Revision as of 00:13, 17 October 2012

Welcome to the humanities section
of the Wikipedia reference desk.
Select a section:
Want a faster answer?

Main page: Help searching Wikipedia

   

How can I get my question answered?

  • Select the section of the desk that best fits the general topic of your question (see the navigation column to the right).
  • Post your question to only one section, providing a short header that gives the topic of your question.
  • Type '~~~~' (that is, four tilde characters) at the end – this signs and dates your contribution so we know who wrote what and when.
  • Don't post personal contact information – it will be removed. Any answers will be provided here.
  • Please be as specific as possible, and include all relevant context – the usefulness of answers may depend on the context.
  • Note:
    • We don't answer (and may remove) questions that require medical diagnosis or legal advice.
    • We don't answer requests for opinions, predictions or debate.
    • We don't do your homework for you, though we'll help you past the stuck point.
    • We don't conduct original research or provide a free source of ideas, but we'll help you find information you need.



How do I answer a question?

Main page: Wikipedia:Reference desk/Guidelines

  • The best answers address the question directly, and back up facts with wikilinks and links to sources. Do not edit others' comments and do not give any medical or legal advice.
See also:



October 7

Paul Guilfoyle

Would anyone mind digging up a reference to verify Paul Guilfoyle's date of birth? I've tried, but I don't have any access to some of the sites that would provide me with that information. Kurtis (talk) 05:49, 7 October 2012 (UTC)[reply]

The Miami Herald says he was 63 this past April 28th, so it looks like he was born in 1949, not 1955 as a few places claim. Clarityfiend (talk) 09:32, 7 October 2012 (UTC)[reply]
Crap... I guess we need to narrow it down and figure out which sources have it right, don't we? Kurtis (talk) 09:40, 7 October 2012 (UTC)[reply]
The Guardian thinks he was born in 1951, but since the writer also believes he is the son of Paul Guilfoyle (actor born in 1902), I think we can dismiss that idea. Clarityfiend (talk) 09:43, 7 October 2012 (UTC)[reply]
I don't follow your reasoning there. It's perfectly believable that a 49-year old man could father a son. Rojomoke (talk) 10:14, 7 October 2012 (UTC)[reply]
The article says that they're not related. Now the thing that makes me believe that 1949 is correct is that the article also says he enrolled at Lehigh University in 1968. That fits well with 1949, but if he were born in 1955, he'd have to have been around 13 years old. Clarityfiend (talk) 12:23, 7 October 2012 (UTC)[reply]
And Lehigh says his grad year was 1972. [1]--184.147.123.169 (talk) 13:56, 7 October 2012 (UTC)[reply]

Mental institutions

What steps/criteria must be met (in the United States) to have someone involuntarily committed to a mental institution? (Note: This is NOT a request for advice on any current legal case.) 24.23.196.85 (talk) 06:18, 7 October 2012 (UTC)[reply]

Note that, in the US, being mentally unbalanced is not, in itself, sufficient reason for committal. They must also prove the person presents a danger to themself or others (and even those who are a danger to themself aren't always committed). It's going to vary by state, but here are some general steps:
1) You have to take them into custody. If they are posing an obvious threat to themself or others, say by brandishing a weapon and threatening to kill people, the police can do so immediately. If the danger is less obvious, a court hearing would be required where evidence of their behavior would be considered.
2) Once in temporary custody, psychiatrists would evaluate them and recommend to the court if they should be committed long-term, or released.
3) Another hearing is then held to rule on this evidence.
4) Once committed, periodic reviews are conducted to determine if they need to remain committed.
An alternative method is when somebody is found to have committed a crime, but to be insane (here the terminology differs, with some states calling this "innocent by reason of insanity" and others calling it "guilty, but insane"). In this case the sentencing phase covers the committal. StuRat (talk) 06:36, 7 October 2012 (UTC)[reply]
Is a ruling by a court of law always required or can a health authority order such incarceration? Roger (talk) 09:05, 7 October 2012 (UTC)[reply]
Depends on the jurisdiction. See Involuntary commitment for some details. Marnanel (talk) 14:54, 7 October 2012 (UTC)[reply]
OK, that more or less covers it. Thanks! 24.23.196.85 (talk) 23:21, 7 October 2012 (UTC)[reply]
You're welcome. I'll mark this resolved. StuRat (talk) 00:49, 8 October 2012 (UTC)[reply]
Resolved
Also see getting Baker Act'd, or as it is also known Flori-DUH ;-). Marketdiamond (talk) 08:21, 9 October 2012 (UTC)[reply]

Green Card Lottery

Hey, I signed up for the annual Green Card Lottery of the United States. My question is, if I win the lottery while I'm studying at university in another country, Can I finish my studies in that country or should I move to the U.S. immediately? Or I could finish school and then move? Thanks a lot! — Preceding unsigned comment added by 213.57.244.213 (talk) 13:25, 7 October 2012 (UTC)[reply]

Look at question #25 in the instructions PDF linked on this State Department page. -- Finlay McWalterTalk 13:33, 7 October 2012 (UTC)[reply]
Yeah, but can I go to the U.S. and get the green card, and then go back to my country until I graduated? Levanagr56 (talk) 14:51, 7 October 2012 (UTC)[reply]
That's, for me, already a legal question. You'll have to ask a lawyer about the details. BTW, the chances of winning are not very high, so, probably you won't need to worry about it. OsmanRF34 (talk) 19:42, 7 October 2012 (UTC)[reply]

Read the instructions. The instruction for DV-2014 says that if you enter the lottery now and win, the immigrant visa should be issued during the 2014 fiscal year, i.e. at some point between October 1, 2013, and September 30, 2014. I understand that immigrant visas are usually valid for 6 months, i.e. you'd have to enter the USA within 6 months since the visa is issued. Once you enter the country with the immigrant visa, you're a US permanent resident and are supposed to reside in the United States, and not stay abroad too much; otherwise, you may be deemed to have forfeited your permanent resident status. However, a permanent resident can apply for a re-entry permit, which (normally) would allow him to stay abroad for up to 2 years. (A re-entry permit cannot be extended, but one can come back to the US and apply for a new permit). That hopefully should be enough to finish one's education! -- Vmenkov (talk) 02:02, 9 October 2012 (UTC)[reply]

Bob Hoover anecdote - urban legend?

An anecdote is cited in a newspaper article about Dale Carnegie's book How to win friends and influence people: (in Norwegian)

Rough translation:

The famous test pilot and show pilot Bob Hoover was once mid-air air between Los Angeles and San Diego, when both engines suddenly stopped. He managed to make an emergency landing. Both Hoover and his crew survived, but the aircraft was destroyed. The first thing he did after landing was to examine the fuel tank, and sure enough: his plane had been filled with the wrong fuel.
When Hoover returned to the airport, he promptly requested to speak to the mechanic who had had filled his tank. The young mechanic was filled with grief and anxiety over the blunder that had destroyed an aircraft and almost cost three people their lives. Tears were running down his face when Hoover approached him. Then Hoover held his big arm around the young mechanic's shoulder, and said: "To prove that I am absolutely confident that you have learned from this, I want you to prepare my F-51 plane for tomorrow."

The accident is mentioned here in the article, but it appears Hoover was seriously injured, and thus would not be in a position to speak to the technician, and also states that the discovery of the wrong fuel having been used was a result of an investigation, not of a casual inspection by Hoover. From the context of the article I linked to, the anecdote probably appears in the book. This smells of bogus/urban legend to me. Does anyone here have more information? --NorwegianBlue talk 13:39, 7 October 2012 (UTC)[reply]

I think you're right. I can't find any information on the incident from Snopes or The Straight Dope. --BDD (talk) 16:30, 8 October 2012 (UTC)[reply]
I found the book (edit: Dale Carnegie's book) in full-text online. Lots of sites. Not sure about the copyright status and won't link directly, but googling "Bob Hoover, a famous test pilot and frequent per-former" [sic] will locate it. The anecdote is in the book, and claims that everyone aboard was unhurt in the crash, and that Hoover personally inspected the tank. It mentions a source for the accident (the magazine "Flight Operations"), none for what happened afterwards. The moral of the story, of course, is "never criticize anybody". The story struck me as too good to be true. Maybe Carnegie was practicing his own philosophy, "When dealing with people, let us remember we are not dealing with creatures of logic. We are dealing with creatures of emotion...", and adapted the facts? --NorwegianBlue talk 21:15, 8 October 2012 (UTC)[reply]
If Hoover's aircraft was refuelled with inappropriate fuel, how would he have made it to somewhere between LA and SD? A much more credible version of the accident is to be found at Bob Hoover#Hoover Nozzle and Hoover Ring where it explains that the aircraft crashed on take-off.Dolphin (t) 11:58, 9 October 2012 (UTC)[reply]

Assuming it's true, how would Hoover have been able to tell (without using a lab) whether the "wrong" fuel was used? --Dweller (talk) 12:56, 10 October 2012 (UTC)[reply]

It's not that improbable that they smell differently, and that Hoover would have been able to tell them apart. Our article cites his book "Forever flying" as source about the accident, so if anyone happens to have a copy: how badly was he hurt, and does he describe the talk with the mechanic? --NorwegianBlue talk 19:58, 10 October 2012 (UTC)[reply]
According to page 276 of Forever Flying, the accident occurred shortly after take off from Brown Field in San Diego. The instrument panel of the Shrike Commander was torn out of its mounts and fell on Hoover's shins. Apparently, he was not injured as he was able to walk around the aircraft, open the drain valve, and identify the jet fuel by its smell. The two passengers were not hurt. Hoover's talk with the line boy who serviced the Shrike is described on page 277 and references a newspaper article that quoted him as saying: There isn't a man alive who hasn't made a mistake. But I'm positive you'll never make this mistake again. That's why I want to make sure that you're the only one to refuel my plane tomorrow. I won't let anyone else on the field touch it. Skeet Shooter (talk) 01:34, 11 October 2012 (UTC)[reply]
Thanks a million! --NorwegianBlue talk 07:27, 11 October 2012 (UTC)[reply]
Resolved

Inhibitions and intelligence

Is there a correlation between the two? Ankh.Morpork 14:48, 7 October 2012 (UTC)[reply]

I'd say so. Being inhibited implies that you have thought of the negative consequences that can result. Less intelligent species can't do this. Of course, they do have fears which are more instinctual, but I wouldn't call those "inhibitions". StuRat (talk) 18:34, 7 October 2012 (UTC)[reply]
That would go a long way towards explaining the people on the Jerry Springer Show! Roger (talk) 19:08, 7 October 2012 (UTC)[reply]
The problem is, how do you quantify the properties of "intelligence" and "inhibition" such that one could draw a correlation. There are, of course, intelligence "tests" which purport to quantify it, but Intelligence_quotient#Criticism_and_views shows how many of these such tests are generally met with some bit of skepticism. --Jayron32 19:10, 7 October 2012 (UTC)[reply]
I can think of simple ways to test for level of inhibition (other than asking people to self-report). For example, ask randomly selected students to report to a room, one at a time, at various times, for a (fake) test, and ask them not to bring anything with them other than a number 2 pencil (and their clothes, of course). Have the room be devoid of anything to do or read, except for a newspaper on the instructor's desk. Tell the student the copier is broken and there will be a 10 minute delay until the test is printed out. Then observe and record whether they grab the newspaper to read (and how long they wait) while they wait. Yes, some people enjoy reading a newspaper far more than others, but almost everyone would prefer a paper to doing nothing at all. StuRat (talk) 19:22, 7 October 2012 (UTC)[reply]
The word 'intelligence' is the problem here. Some people have a high degrees of social intelligence and academic intelligence who are very, very uninhibited (like film stars with degrees and things --Hedy Lamarr springs instantly to mind). No. there is no connection other than a poplar stereotype. Declaration of conflicting interests: I do not fall into either category so I feel well placed to comments without fear or favour.--Aspro (talk) 19:43, 7 October 2012 (UTC)[reply]
Claiming that there is no connection requires as much evidence as claiming that there is a connection, and you haven't offered any evidence for either view. --140.180.242.9 (talk) 21:52, 7 October 2012 (UTC)[reply]
Indeed, to support what 140.180 is saying; this may or may not be an answerable question, but either way Aspro's answer isn't it. Every meaningful categorization or explanation of human behavior is going to have a significant number of outliers, cherry picking those outliers does not amount to a refutation of the principle. I happen to have no faith that there has been an established connection, but not because of Hedy Lamarr. --Jayron32 04:57, 8 October 2012 (UTC)[reply]
Poplar stereotype: "Those trees all have ugly bark !". :-) StuRat (talk) 22:18, 7 October 2012 (UTC) [reply]
Poplar stereotype: "East London is a dump!" Marnanel (talk) 22:34, 7 October 2012 (UTC)[reply]
  • To make this question concrete, we could frame it as asking whether there is a correlation between IQ and the dimension of personality called introversion. The literature on that is pretty sparse, but as far as I can see there is no evidence for a strong relationship. There does however seem to be a relationship between introversion and persistence, which plays a role in academic success. In other words, introverted people may do better in academia, but because of their habits, not because they are more intelligent. Looie496 (talk) 00:40, 8 October 2012 (UTC)[reply]
Inhibition has several meanings. Are we talking
  • inhibition: the conscious or unconscious constraint or curtailment of a process or behaviour, especially of impulses or desires, .. enabling the delay of gratification from pleasurable activities.
  • Social inhibition: a conscious or subconscious constraint by a person of behaviour of a social nature. The constraint may be in relation to behavior, appearance, or a subject matter for discussion, besides other matters,
  • inhibition as a feeling that makes one self-conscious and unable to act in a relaxed and natural way.?
  • Cognitive inhibition, memory inhibition ...
  • latent inhibition
Searching for intelligence and inhibitions returns results like:
Enough "research" it seems, but an answer... woops, I notice I'm in RD/Humanities, how did I get here? Ssscienccce (talk) 22:34, 8 October 2012 (UTC)[reply]

I was referring to social inhibitions. Based on my experience during a lecture, I was curious if there was any relationship between the intelligence of a question and the likelihood of it being asked. I was wondering if the more competent members of the audience were affected by some form of Dunning–Kruger effect or perhaps, were more likely to be introverted and inhibited in their general social intercourse. Ankh.Morpork 23:09, 9 October 2012 (UTC)[reply]

John Merbecke memorial?

I have been trying to flesh-out our article John Merbecke, the English musician and religious reformer. His music almost universally accompanied the Anglican Communion Service around the world from the 1850s to the 1970s, and was used by many other denominations too. However, I can't find anywhere where there is any sort of monument or memorial to him; not so much as a plaque or stained glass window. The only commemoration of him seems to be the Merbecke Choir. Can anybody find anything? Alansplodge (talk) 21:13, 7 October 2012 (UTC)[reply]

He's depicted in a window in Washington National Cathedral: http://www.flickr.com/photos/hilton_photos/1220135413/ Marnanel (talk) 22:30, 7 October 2012 (UTC)[reply]
Also at Wistow and at Kilham. Marnanel (talk) 22:32, 7 October 2012 (UTC)[reply]
Thank you, kind sir. Perhaps one day, someone will give him a window of his very own. Alansplodge (talk) 23:44, 7 October 2012 (UTC)[reply]

Record number of dowagers in Europe

When was there a record number of royal dowagers in European history? Dowagers as in widows of Sovereign Princes or Crown Princes, Sovereign/Peerage Dukes, Kings, Emperors etc, so they could be mothers, stepmothers, aunts, cousin-in-laws of the then present title holder. How would each dowager be referred if let say there are three dowager queens from three childless kings are still living in court? I will mention the commonly known case of Mary of Teck and Elizabeth Bowes-Lyon, so someone doesn't bring it up. Don't mention of Asian polygamous monarchs like China, where the dead emperor can leave hundred or thousands of dowager consorts. --The Emperor's New Spy (talk) 22:25, 7 October 2012 (UTC)[reply]

Well, from 1485 to 1492 there were three living Queen Mothers in England
Not sure if Cecily Nevil was styled as "Queen" however. (our article on her hedges... saying that when Edward VI became King, she "became an effective Queen Mother"). Blueboar (talk) 01:28, 8 October 2012 (UTC)[reply]
Cecily and Margaret weren't Queens though and each held a different title. They would be respectively, the Dowager Duchess of York, the Queen Dowager, and "My Lady the King's Mother" or just the Dowager Countess of Richmond. --The Emperor's New Spy (talk) 16:58, 8 October 2012 (UTC)[reply]
Do you mean for a single European monarchy, or do you mean in total across all of Europe? Because the first question should be answerable, but the second may be quite difficult to calculate if it hasn't already been done. For example, from 1574 - 1587 France had 3 living dowagers: Mary, Queen of Scots, Catherine de' Medici and Elisabeth of Austria, likewise again for about 8 months in 1589, with Catherine de' Medici, Elisabeth of Austria, and Louise of Lorraine. What you'd need to do to compile a list for any country is to search through the "lists of consorts" articles, like List of French consorts. See Category:Lists of queens to get yourself started. --Jayron32 17:33, 8 October 2012 (UTC)[reply]
Blueboar - I presume you mean Edward IV, not Edward VI. AlexTiefling (talk) 09:34, 9 October 2012 (UTC)[reply]

Note that in British peerage system, cousins-in-law, sisters-in-law, etc, are not styled as dowagers. A widow of a peer may be called dowager only if (a) her husband bore the title and (b) the current peer is a direct descendant of her deceased husband. Surtsicna (talk) 17:35, 12 October 2012 (UTC)[reply]


October 8

Follow-ups on Suaad Mohamud

In 2009, I read about the interesting case of Suaad Mohamud. Canada prevented her from re-entering the country after her visit to Kenya, because she didn't look like her passport photo, was 6-7 cm shorter than the height on her driver's license, couldn't name Lake Ontario, didn't know Canada's current or former prime ministers, couldn't describe what she did at work, couldn't name any of the subway stops she encountered every day, didn't know her only son's birthdate or birthplace, and claimed to attend 2 different colleges while being unable to name professors from either. Then DNA tests proved she was really Suaad, despite all the evidence to the contrary. Then she filed a lawsuit, the obligatory ending of all modern tragedies. You can go here to read more about it: http://www.cbc.ca/news/canada/toronto/story/2009/09/28/lawsuit-mohamud.html

I tried to follow up on this, but couldn't find anything. What happened to the lawsuit? What happened to the Canadian government's internal investigation? Did anyone care to interview her or her family members and ask why she didn't know anything? I know it's quite likely that after she came home safely, nobody bothered to follow up on the story. But just in case somebody did, I'm very curious to know what they discovered. --140.180.242.9 (talk) 03:26, 8 October 2012 (UTC)[reply]

Per you own source, it appears to be somewhat disputed what she did and didn't say. What you have described appears to be the government's explaination. Nil Einne (talk) 07:30, 8 October 2012 (UTC)[reply]
The source says the lawyer claims the government didn't give the full story. He didn't say that the story the government did give is inaccurate. --140.180.242.9 (talk) 22:05, 8 October 2012 (UTC)[reply]
In December 2010, federal court judge Sean Harrington ordered the federal government to pay her $13,500 in legal costs. “They (embassy officials) were arrogant, dismissive, refused to come to her aid and circled the wagons when cracks in their case began to appear,” he said. Four interviews and being put under pressure will result in inconsistent statements, not being able to remember some basic facts etc. It's easy to confuse people, and a list of all these inconsistencies may look more convincing than a complete transcript or a recording would. As for people ignorant of basic facts about a country, a few years ago when a reporter asked our (Belgian) prime minister to sing the national anthem, he sang the French one instead. Ssscienccce (talk) 23:14, 8 October 2012 (UTC)[reply]
But he didn't say it was 100% accurate either. As Ssscienccce has emphasised, we have no way of knowing what she did get right. Nil Einne (talk) 03:49, 9 October 2012 (UTC)[reply]
The Federal Court's decision is [here]. Zoonoses (talk) 04:29, 10 October 2012 (UTC)[reply]

Boni & Liveright : cowled monk ? soviet posters ?

Hello L.O. (Learned Ones) ! I’m translating Boni & Liveright for WP french, and

1° I can’t find its logo : "a cowled monk" (quite a witty logo, given the kind of B&L’s publications !) . Is someone able to show it ?

2° I don’t see in the WP english article any mention of the soviet posters I stumbled upon on Commons (http://commons.wikimedia.org/w/index.php?search=Boni+%26+Liveright&title=Special%3Asearch) . Were they really issued by B&L ? Any commentary on that surprising production ? Thanks a lot beforehand for your answers . T.y. Arapaima (talk) 10:38, 8 October 2012 (UTC)[reply]

See the Liveright logo here, and an earlier 1922 version here. Alansplodge (talk) 11:40, 8 October 2012 (UTC)[reply]
Re 2°: See Through the Russian Revolution, as explained in File:Soviet Poster 5.jpg. -- Vmenkov (talk) 02:23, 9 October 2012 (UTC)[reply]
Thanks a lot to both ! "cowled monk" : I thought he was hooded, but actually he is not, and seems quite open to the world...I discover Albert Rhys Williams , & am fascinated...Thanks again , t.y. Arapaima (talk) 09:07, 9 October 2012 (UTC)[reply]
Well, he is wearing a cowl, but not on his head! Alansplodge (talk) 12:22, 9 October 2012 (UTC)[reply]

Does this have a name?

Towns that have a name like Oklahoma, Pennsylvania or Paris, Kentucky or Poland, Ohio? Do you call towns like that something, an encyclopedic term? Thanks!Marketdiamond (talk) 12:12, 8 October 2012 (UTC)[reply]

Other than toponym? Probably not. I don't know that there has been devised a specific term for "places that share a name with another place". --Jayron32 12:50, 8 October 2012 (UTC)[reply]
Thanks Jayron32, to be very clear though it wouldn't be the proverbial "Springfield" so places that share name with another place isn't what I'm exactly asking its more like small one or two stoplight hamlets that share a name with a megopolis, nation, state or cultural center (Hollywood, Greenwich Village etc.), more the culture shock of a small small rustic town in Kentucky verses the world center that is Paris or the entire state of Oklahoma confused with a one stoplight burg in Pennsylvania, that kind of thing, not looking for the 1,000s of Glendales or Springfields etc. Marketdiamond (talk) 13:00, 8 October 2012 (UTC)[reply]
Well, the term in reverse (ignoring size, which for some reason you seem to feel is important here) is eponym, an eponym is the source of the name, so Paris, France is the eponym for Paris, Kentucky. Also, it does happen often enough the reverse as you describe; many American cities are considerably larger than their eponymous sources: Boston, Massachusetts is much larger than Boston, Lincolnshire after which it was named. Likewise, I think Memphis, Tennessee is larger than Memphis, Egypt ever was. And compare Philadelphia, Pennsylvania with Alaşehir, or the ancient Philadelphia. --Jayron32 16:45, 8 October 2012 (UTC)[reply]
Or for that matter Portland, Oregon, which was named after Portland, Maine. Pais (talk) 20:29, 8 October 2012 (UTC)[reply]
Which was itself named after Portland. Alansplodge (talk) 00:19, 9 October 2012 (UTC)[reply]
It doesn't always move west: Pasadena, Texas is named after Pasadena, California. --Trovatore (talk) 08:24, 9 October 2012 (UTC)[reply]
I don't know if it has a specific name, but check out List of places named after places in the United States. --Mr.98 (talk) 15:01, 8 October 2012 (UTC)[reply]
In that light, we should have List of U.S. places named for European cities.    → Michael J    16:38, 8 October 2012 (UTC)[reply]
I for one would like to see that article, but if someone creates it, please provide an inline cite to a reliable source to show the town was "named for a European city" and not just "Named for a US city which was named for a European city," such as some little US Boston, Memphis or Philadelphia which was named after the well known US city rather than the less well known (to frontiersmen) non-US city. (Also note that we would need similar lists for every name-source continent). Many articles about small towns have no discussion of the name origin, or declare, without a ref, what the town is named for. Edison (talk) 21:05, 8 October 2012 (UTC)[reply]
Indeed. For example, New Boston, New Hampshire was named for Boston, Massachusetts, which was in turn named for Boston, Lincolnshire. There are many places like that, where there's a chain of inspiration. Such lists probably aren't a wise idea, because there are many folk etymologies for place names, and really, of the millions of named places in the world, you could likely come up with some unwieldy number of articles of arbitrarily organized lists of cities. --Jayron32 02:01, 9 October 2012 (UTC)[reply]

Thank you all for all these great insights, sincerely I am learning a lot, alas no solution to the problem at hand though (is the mark of genius asking lots of questions on RefDesk that just truly have escaped human intelligence for the ages lol? Watch for it everyone I might invent a word ala Stephen Colbert lol. Thanks again for all the insightful responses! Marketdiamond (talk) 06:57, 9 October 2012 (UTC)[reply]

And for another response that doesn't answer the question, but warns folks to not always assume that names come from Europe, Melbourne, Florida got its name because the first postmaster came from Melbourne, Australia. — Preceding unsigned comment added by Trovatore (talkcontribs) 08:24, 9 October 2012‎
But Melbourne, Australia, was named after William Lamb, 2nd Viscount Melbourne, whose title came from Melbourne, Derbyshire. Which is in Europe.  :-) Ghmyrtle (talk) 09:40, 9 October 2012 (UTC)[reply]
True, but I'm really surprised to see a Brit writing that final sentence.  ;-) HiLo48 (talk) 11:21, 9 October 2012 (UTC)[reply]
In an extreme case, Cartagena, Colombia was named after Cartagena, Spain, which was named after Carthage, which means "New City" - implicitly, "New Tyre". So Cartagena, Colombia, is effectively New New New Tyre. AlexTiefling (talk) 09:54, 9 October 2012 (UTC)[reply]
Wow, four continents in once chain! An extreme case indeed. Can anyone find a better one? — Kpalion(talk) 10:22, 9 October 2012 (UTC)[reply]
Four I counted 3 but yes that is impressive. Marketdiamond (talk) 00:40, 10 October 2012 (UTC)[reply]
Tyre -- Asia; Carthage -- Africa; Cartagena, Spain -- Europe; Cartagena, Colombia -- South America. AnonMoos (talk) 07:20, 10 October 2012 (UTC)[reply]
Thanks for the clarification, but I call translation foul on that Tyre one! Impressive none the less! Marketdiamond (talk) 08:24, 10 October 2012 (UTC)[reply]

"your mind is always on your music"!

there is a comic/caricature with a woman being played like a cello (you know, she looks like this - http://3.bp.blogspot.com/-lcIyObjNdyc/TxB9qmQIysI/AAAAAAAABds/FZXMtLTIcYM/s1600/man-ray-cello-woman.jpg - but since it's a caricature/comic she looks even more like a cello), as she says something like "Your mind is always on your music!"

can someone find a link to this comic for me? thanks. --91.120.48.242 (talk) 15:34, 8 October 2012 (UTC)[reply]

I found Cello Woman by Hara Reita. Close but no cigar, I suppose. Alansplodge (talk) 16:10, 8 October 2012 (UTC)[reply]
According to this site, it's a photo by Man Ray of Alice Prin, aka Kiki, Queen of Montparnasse. This site concurs and states the title is Le Violon d’Ingres. Clarityfiend (talk) 21:58, 8 October 2012 (UTC)[reply]
The only cartoon I can find of it isn't very close to your description. Clarityfiend (talk) 00:12, 9 October 2012 (UTC)[reply]
I think the OP was looking for a caricature "like this" but "even more like a cello". I've drawn a blank so far. Alansplodge (talk) 00:13, 9 October 2012 (UTC)[reply]

Voter registration requirements in New York in 1864

Last night's episode of Copper (TV series) took place on the 1864 Presidential election day. The show mainly concerns Irish-American immigrants living in the Five Points area of Manhattan. It showed Irish voters being given free whiskey in order to vote for the Boss Tweed/Tammany Hall machine. It got me to wondering, what were the voter requirements during that period? Were there citizenship requirements for these Irish immigrants to vote, or could any male over the age of 21 vote, regardless of how long they had lived in the US? 69.62.243.48 (talk) 18:03, 8 October 2012 (UTC)[reply]

It's a solid question, and I don't begrudge you for asking it. It would be worthwhile to find an answer, but I will note that I don't think Tammany Hall was particularly concerned about the legality of what it was doing in any other aspect of its hold on power, so I'm not sure it would have mattered much what local voting laws were at the time. Tammany Hall operated on the "Golden Rule": he who has the gold, rules. It was a fantastically corrupt organization. --Jayron32 18:26, 8 October 2012 (UTC)[reply]
The book "Boss Tweed"" by Ackerman, 2011, describes New York City vote fraud techniques in 1868 (things were likely much the same in 1864 so far as the letter of the law).. A voter's name and address needed to be on the voting rolls for him to be able to vote. Tammany Hall would make up lists of fake names and addresses, and groups of paid fraudsters would go and register to vote under those names, often accompanied by naturalization papers with real signatures of the proper judges, but filled out in false names or by persons who had not been resident in the US for a sufficient time. The Naturalization Law of 1802 was the applicable law in the 1860's. The applicant had to be a "free white person," and present a certificate from a court, recorded when he arrived, showing when he arrived and from where, and swear allegiance, and show that he had resided in the US for 5 years and in the state or territory for 1 year. Here is a "Petition for Naturalization" from that era. The applicant had to say that he renounced loyalty to his former ruler, and state when and where he arrived and on what ship. So it was not a simple matter of an Irishman getting off the boat and voting; he needed naturalization papers to register, but they could be bought cheap or provided free by political functionaries. Then the party bosses followed up by sending people to vote multiple times under various names not their own. Soldiers from New York who were away from home could also file a form to allow someone else to vote for them, an obvious method of possible vote buying. In addition, they got telegraph reports from areas of New York state where the other party was strong, then added just enough fake votes to win statewide contests. This same process of finding out how many votes were needed was used in Kentucky at least through the Roosevelt administration and was known as "counting in" the election. Slow returns were a signature of vote fraud. In Memphis Tennessee, "Boss Crump" was also known for finding as many votes as were needed and providing them. This was more precise than just throwing in a bunch of fake votes and failing to win on the one hand, or bringing down a Congressional investigation through overkill. Going beyond proof of residence or of naturalization, some states required proof that a "poll tax" had been paid, making it harder for poor people, including African-Americans and recent immigrants, to vote. It was sometimes waived if a "Grandfather Clause" was satisfied, with a voter bringing in sworn proof that his grandfather had been eligible to vote before the American Civil War. Edison (talk) 20:24, 8 October 2012 (UTC)[reply]

"Sleepy suburb"

Does anyone know what the origin of this term is? For example Sleepy suburb lives through nightmare. Having lived in suburban and urban areas, I can't say I was generally more tired in one than the other. Mark Arsten (talk) 21:14, 8 October 2012 (UTC)[reply]

Our article Commuter town says: A commuter town may also be known as a bedroom community or bedroom suburb (Canada and U.S. usage), a dormitory town or dormitory suburb (UK Commonwealth and Ireland usage), or less commonly a dormitory village (UK Commonwealth and Ireland). These terms suggest that residents sleep in these neighborhoods, but normally work elsewhere; they also suggest that these communities have little commercial or industrial activity beyond a small amount of retail, oriented toward serving the residents. Maybe this is part of the reason that the two words have stuck together? --NorwegianBlue talk 21:32, 8 October 2012 (UTC)[reply]
That, and particular people are always fantastically fond of all alliteration. StuRat (talk) 21:55, 8 October 2012 (UTC)[reply]
I always thought it had to do with the fact "they roll up the sidewalk at dusk" in the suburbs and anyone desiring a nightlife had to go into the big city. Rmhermen (talk) 22:24, 8 October 2012 (UTC)[reply]
I think the statement has origins in all of the above. Suburbs are bedrooms communities (places where people sleep and do little else), but they are also noted (by many, not just me) as places of such mind-numbing boredom that it lulls people to perpetual drowsiness. Compare to The City That Never Sleeps, an epithet applied so several decidedly unsurbuban places. --Jayron32 22:26, 8 October 2012 (UTC)[reply]
From Collins American English Dictionary: sleepy: not very active; dull; quiet ⇒ "a sleepy little town". I come across the expression "sleepy little town" quite often (seems almost a cliché for some writers, when the main character reminisces about his youth), so maybe sleepy suburb is simply a variation on that theme? Ssscienccce (talk) 23:54, 8 October 2012 (UTC)[reply]

Google's "n-Grams" site is your friend for the study of the history of cliches: http://books.google.com/ngrams/graph?content=sleepy+suburb&year_start=1800&year_end=2000&corpus=0&smoothing=2 , and, specifically for the early history. The earliest quote known to Google Books is from 1871: "... A river- boat soon brought us to that interesting but sleepy suburb of London".-- Vmenkov (talk) 02:28, 9 October 2012 (UTC)[reply]

So the original "sleepy suburb" was Chelsea. Interesting. Ghmyrtle (talk) 10:32, 9 October 2012 (UTC)[reply]
Chelsea was apparently also the earliest [model of a] shady suburb. -- Vmenkov (talk) 15:30, 9 October 2012 (UTC)[reply]
How about Sleepy Hollow? Best-Story-Ever, Washington Irving was a genius, imagine how much $ all those films made! Marketdiamond (talk) 08:13, 9 October 2012 (UTC)[reply]
Who could sleep in a place like that ? :-) StuRat (talk) 07:39, 10 October 2012 (UTC) [reply]
Lodging and Visitor board persons (Historic Hudson Valley) sleep real well especially during November after all those tourism $$s are counted. BTW, wasn't it originally "Greensburg"? Marketdiamond (talk) 08:22, 10 October 2012 (UTC)[reply]

British Marines' right to search US cities and arrest British deserters after the Revolutionary War

I certainly don't take old TV dramas as reliable sources for history, but I recently watched an old episode of the Daniel Boone TV show, episode 106 "Then Who Will They Hang from the Yardarm If Willy Gets Away?" Original Air Date: 02-08-68, and was puzzled. In it, British Marines, sometime after the American Revolution is over, are in Charleston South Carolina, searching for Willy, a young British sailor who deserted his ship. They announce to all that under "the treaty" they have the right to search buildings and wagons for the deserter.The deserter moves to Kentucky, but another British sailor goes there and blackmails him with threats of turning him over to the British, who would likely hang him. Did the writers make this up completely, or was there some provision for foreign powers to search for and capture deserters in a US port city? Edison (talk) 23:21, 8 October 2012 (UTC)[reply]

The British certainly claimed a privilege to stop U.S. ships in international waters and search them for deserters; see War of 1812#Impressment... -- AnonMoos (talk) 23:40, 8 October 2012 (UTC)[reply]
P.S. One episode of the Daniel Boone TV show had an Aztec prince in full pre-Columbian regalia turning up in Kentucky! AnonMoos (talk) 23:48, 8 October 2012 (UTC)[reply]
I think the writer of that episode must have thought he was putting on a coon-skin cap, but actually put a live raccoon on his head, which then attacked him, causing brain damage. StuRat (talk) 23:53, 8 October 2012 (UTC) [reply]
Don't laugh about the Aztec thing in Kentucky . . . check out Meadowcroft Rockshelter . . . 16,000 years ago and there are tons more "sites" up and down the Ohio Valley, in fact the largest settlement in North America was in present day Illinois near the Mississippi and Ohio junction. Marketdiamond (talk) 06:53, 9 October 2012 (UTC)[reply]
That was 16,000 years ago. By the time of Daniel Boone, the Aztecs were gone. StuRat (talk) 07:21, 9 October 2012 (UTC)[reply]
They were "mound builders" etc., not Aztecs... AnonMoos (talk) 09:24, 9 October 2012 (UTC)[reply]
Re: Meadowcroft and Moundbuilders, we do have a Champ article, so if a pea sized brained reptile could survive 16,000 years a tribe that resembled "Aztecs" could conceivably . . . btw never saw that episode but could the average U.S. college graduate differentiate an Aztec vs. a Ohio Valley 18th century Indian chief in full regalia? And if not how was Boone to call him anything different given no schooling . . . Columbus after all called them Indians because they looked the part of Asian sub-continentals, also see American Pepper vs. Black Pepper the Indian vine--the other Indian not Aztec Indian. Confused? So may have been Daniel Boone ;-). Marketdiamond (talk) 08:16, 10 October 2012 (UTC)[reply]
Whatever -- if you had seen the actual TV episode in question, you would have probably found it rather laughable, as I did, since it showed a self-identifying Aztec prince (dressed suitably according to Hollywood ideas) traveling a long way to seek out Daniel Boone. In any case, the Aztecs did not actually become politically prominent until ca. 1428, and Kentucky is far out of the range of the Uto-Aztecan languages... AnonMoos (talk) 21:11, 10 October 2012 (UTC)[reply]

I don't see anything about deserters in "the treaty". Jay intended for Jay's Treaty to include a negotiated end to this practice, but was unsuccessful, according to our article. The legal argument the British used was that the empire did not recognize the right of its own subjects to become citizens of another country. Therefore, anyone who was ever a British subject was always a subject to British law, including those born in the Colonies prior to the Treaty of Paris (but yes, they did have an awful false positive rate, and impressed many sailors who had never been British subjects). The issue was ignored even in the Treaty of Ghent (wikisource:Treaty of Ghent). The British never officially reversed their legal opinion on the matter, but the practice stopped after the War of 1812, or so says this unreferenced section. Anyway, on that basis, the TV show made it up. The United States never endorsed the practice of impressment (be it as a matter of drafting subjects or recovering deserters). The British believed they had a legal argument, but it was not based on any treaty. And their legal argument didn't extend to invading sovereign territory to recover deserters. Someguy1221 (talk) 00:14, 9 October 2012 (UTC)[reply]

Thanks for the helpful links. Maybe I missed in my haphazard viewing some allegation of murder, if for instance the fugitive was accused of participation in a mutiny, which might require turning the offender over, per article 27 of Jay's Treaty, approved 1796, as opposed to simply deserting peacefully from a ship. Edison (talk) 02:42, 9 October 2012 (UTC)[reply]
At the time of the U.S. Civil War, it was pointed out that the British position with respect to the Trent Affair was 180° diametrically opposed to the British position in 1812... AnonMoos (talk) 04:13, 9 October 2012 (UTC)[reply]
As was the American position. It should not surprise us that this change in position occurred... opinions about stopping and searching neutral shipping during wars usually depend on who is the searcher and who is the searchee (the searcher feels it is justified, the searchee does not). Blueboar (talk) 15:42, 9 October 2012 (UTC)[reply]


October 9

Yongle Encyclopedia online

Is there an online version of the Yongle Encyclopedia. I tried to find one to add, as an external link, to the article, but I couldn't find one. Is there one and if so where is it. Please respond at Talk:Yongle_Encyclopedia#Yongle_Encyclopedia_online. Emmette Hernandez Coleman (talk) 03:43, 9 October 2012 (UTC)[reply]

Only about 4% of the Yongle Encyclopaedia is extant. The National Library of China has published a 164-volume facsimile re-print of the majority of the surviving volumes, priced at 250,000 yuan (around US$40,000 or £25,000). Some publishers have made electronic versions, but these come in multi-CD/DVD sets and are not avilable as an online version. Some websites claim to offer e-book versions, but if you inspect them closely, not only are these huge, they are also only about a quarter or so of the extant volumes. --PalaceGuard008 (Talk) 09:43, 9 October 2012 (UTC)[reply]

Arveprins

When did the Danish heir stop being called Hereditary Prince and became Crown Prince?--The Emperor's New Spy (talk) 09:32, 9 October 2012 (UTC)[reply]

They are two distinct titles. Arveprins ("hereditary prince") is the Danish equivalent of heir presumptive. Kronprins ("crown prince") is the Danish equivalent of heir apparent. Gabbe (talk) 10:02, 9 October 2012 (UTC)[reply]
Oh I totally forgot about Knud. Then how is Prince-Elect, the title of the heir before absolute hereditary monarchy, translated to in Danish.--The Emperor's New Spy (talk) 10:59, 9 October 2012 (UTC)[reply]
The Danish for Prince-Elect is udvalgt prins. Gabbe (talk) 11:06, 9 October 2012 (UTC)[reply]

Why is Angela Merkel visiting Greece?

Isn't that too obvious that it will trigger anti-German protests? OsmanRF34 (talk) 10:25, 9 October 2012 (UTC)[reply]

Well, yes... but according to this article the visit is intended, among other things, to show that, within limits, she is willing to help the Greek people out of the present position. No doubt the security situation has been taken into account. Ghmyrtle (talk) 10:29, 9 October 2012 (UTC)[reply]
It's a roman triumph, or germanic triumph, to celebrate their victory over Greece. Amazingly, she is even wearing the same jacket as when Germany won in soccer against Greece. From a diplomatic point of view, it is a faux-pas. Gorgeop (talk) 12:08, 9 October 2012 (UTC)[reply]
(Bad joke time - and apologies in advance. Angela Merkel arrives at Greek immigration control - "Name?" "Angela Merkel." "Nationality?" "German." "Occupation?" "No, I'm just here for a visit...") AndyTheGrump (talk) 12:13, 9 October 2012 (UTC)[reply]
Omggg I actually laughed! Nice one (seriously). --Jethro B 00:45, 10 October 2012 (UTC)[reply]

The visit could be intended to show Merkel towards German domestic public as tough in negotiations with Greece. --Soman (talk) 12:28, 9 October 2012 (UTC)[reply]

"It'll trigger protests" is hardly a reason to avoid diplomacy. In fact, if a simple visit would trigger protests, that's a good sign that diplomacy is needed. --140.180.242.9 (talk) 16:36, 9 October 2012 (UTC)[reply]
Really folks, feel free to reopen if you have something to offer better than a joke. μηδείς (talk) 22:35, 9 October 2012 (UTC)[reply]
Ghmyrtle already did. StuRat (talk) 23:02, 9 October 2012 (UTC)[reply]
This article in the Guardian suggests that the main purpose is to show that Germany really does want Greece to stay in the Union. The protests are not going to shake her resolve. Bielle (talk) 02:07, 10 October 2012 (UTC)[reply]

The 4 top dogs are ladies

As of an hour ago, Australia has females in all the following roles:

Is this a first for a Commonwealth realm?

I know New Zealand and Canada have both had female governors-general, prime ministers and lower house speakers, but I don't know if all the terms overlapped. Other realms may have pipped Australia if these 2 countries haven't. -- Jack of Oz [Talk] 11:42, 9 October 2012 (UTC)[reply]

New Zealand beat you to it, I'm afraid - for around a year in 2005-6 Margaret Wilson was Speaker, Silvia Cartwright Governor-General, and Helen Clark PM. Andrew Gray (talk) 13:00, 9 October 2012 (UTC)[reply]
...and, come to think of it, Sian Elias was the Chief Justice of New Zealand throughout that period, so you had the judiciary led by a woman as well. Andrew Gray (talk) 13:02, 9 October 2012 (UTC)[reply]
Not to mention Theresa Gattung being the CEO of Telecom New Zealand, the largest (by market capitalisation) company on the NZX (by quite a big margin at the time). Nil Einne (talk) 13:10, 9 October 2012 (UTC)[reply]
Oh and at one stage Jenny Shipley was opposition leader [2]. Albeit with the speaker seat occupied by Jonathan Hunt at the time, so Australia simply has to kick out Tony Abbott and replace him with a woman (Julie Bishop perhaps?) and get a new female Chief Justice and Attorney-General and you can take a new arguably significant record. For added bonus (the ASX and NZX are quite different so perhaps it's not a fair comparison) get a female CEO of BHP Billiton too. Nil Einne (talk) 13:13, 9 October 2012 (UTC)[reply]
I should have checked (or thought carefully) about this earlier as I suspected it but wasn't sure. At the time of Silvia Cartwright's swearing in, Christine Fletcher was mayor of Auckland (NZ's most populous city [3] but since this was pre supercity days, not really accurate to say 'by far') [4] so you can add another to the record I mentioned above (which just to re-emphasise does not include the speaker). The source also mentions Marie Shroff being Secretary of Cabinet and Clerk of the Executive Council. (If you're wondering Jenny Shipley losing the leader role to Bill English brought this record to the end, followed soon after by John Banks taking over as mayor. At least if my suspicions of the laters date are right.) Nil Einne (talk) 13:54, 9 October 2012 (UTC)[reply]
Very good answers, folks. And NZ was the first nation to allow women to vote, too. Must be something about the cool southern air. -- Jack of Oz [Talk] 20:50, 9 October 2012 (UTC)[reply]
A little off-topic, but NZ was the first nation to allow all women to vote in national elections. Other countries allowed some women to vote, or to vote in local elections only, earlier. See Timeline of women's suffrage for details. --Tango (talk) 11:32, 10 October 2012 (UTC)[reply]
I'm concerned about Jack's wisdom(?) in describing those four ladies in his original question as dogs. HiLo48 (talk) 20:06, 9 October 2012 (UTC)[reply]
I didn't describe the ladies as dogs. I described the top dogs as ladies. That's what I call " a huge difference".
If the occupants of these four positions were all males, nobody would have objected to them being called "top dogs". It would be sexist to raise an objection to the use of that expression only when they're all females.
Some may see connections (female dog = bitch) that were never there and never intended. That's for them to resolve. -- Jack of Oz [Talk] 20:50, 9 October 2012 (UTC) [reply]

Oliver Cromwell and malaria

How did the first catch malaria? Gorgeop (talk) 12:19, 9 October 2012 (UTC)[reply]

If you're asking how he caught malaria — the banal answer is "from a mosquito." Malaria was still extent in 17th century England, though if I recall correctly, it was significantly less than in 16th century England on account of marsh drainage projects. --Mr.98 (talk) 12:43, 9 October 2012 (UTC)[reply]
Cromwell suffered from, but may not have died from, what was called "ague" - a rather general term for febrile diseases and similar ailments. It's a modern interpretation that this was, in fact, malaria; "malaria" is an Italian term which didn't come into use in English for centuries after that. A (generally non-fatal) strain of malaria was endemic to coastal England, marshy and boggy places, and fenland areas like East Anglia and Cambridgeshire, from the 1400s or earlier. Cromwell had every opportunity to be bitten by an infected mosquito during his many visits to these places. In some patients malaria can be recrudescent, in that it comes back periodically, sometimes decades after infection. Refs for this: The Death of Oliver Cromwell By H.F. McMain, pp 85-90 ISBN 0813121337, and the Wellcome Trust's The history of malaria in England. The former ref says malaria remained endemic until the 19th century when modern drainage removed much of the mosquito breeding habitat. -- Finlay McWalterTalk 12:47, 9 October 2012 (UTC)[reply]
There are credible claims that not only did Oliver Cromwell die of malaria - but that he refused the "Jesuit bark" medicine, quinine, which might have saved him. AndyTheGrump (talk) 12:52, 9 October 2012 (UTC)[reply]
It may be worth noting that it was not until shortly before 1900 that the association between malaria and mosquitoes was demonstrated. Before that, people knew that it was associated with swamps and marshes, but they didn't know why. The most common belief was that it was caused by some sort of gas or vapor that effused from swamps. Looie496 (talk) 16:18, 9 October 2012 (UTC)[reply]
Hence "mal-aria" or "bad air" --Jayron32 16:45, 9 October 2012 (UTC)[reply]
See Cromwell's health & death which suggests that Cromwell (like many of his soldiers) contracted malaria in Ireland in 1649. Alansplodge (talk) 00:37, 10 October 2012 (UTC)[reply]

Recognize a track

moved to Wikipedia:Reference desk/Entertainment

How did Charles Darwin earn a living?

What did he do? How did he earn a living? Nothing much is said about his professional life, other than the fact he discovered the foundation of modern biology. What did he do BEFORE he published his famous theory and AFTER he sailed on the HMS Beagle? Become a public speaker of science? Write science books? Work as an Anglican country clergyman in a small parish? 140.254.226.212 (talk) 19:06, 9 October 2012 (UTC)[reply]

Read our article Charles Darwin and come back to us if you have any further questions. --TammyMoet (talk) 19:09, 9 October 2012 (UTC)[reply]
I did. It mainly describes what he contributed to science, not what he did for a living. 140.254.226.212 (talk) 19:32, 9 October 2012 (UTC)[reply]
According to this site, Darwin was independently wealthy - his father was a successful doctor and inventor, and his wife was a member of the Wedgewood crockery family - so he didn't need to earn a living, which gave him plenty of time to do his research. --Nicknack009 (talk) 19:41, 9 October 2012 (UTC)[reply]
I see. I suppose being a wealthy person or an aristocrat or nobleman/woman may mean higher education, and that means more time for intellectual and academic pursuits back in those days. Another example would be Lady Ada Lovelace. 140.254.226.212 (talk) 19:47, 9 October 2012 (UTC)[reply]
Members of the upper classes definitely had the inner track on becoming scientists in those days. Regarding Darwin, he must have made some money on The Voyage of the Beagle, which was a bestseller, and of course a few of his later books were huge bestsellers; but that income probably wasn't all that significant to him. Looie496 (talk) 19:58, 9 October 2012 (UTC)[reply]
(edit conflict) Indeed. The term "Idle rich" is a common English idiom to refer to people who have enough money that they never have to earn another cent for the rest of their lives. The modern, vulgar idiom for this is "fuck you money", i.e. so much money that you can tell any person in the world "fuck you" without consequences. Many people who are that wealthy still continue to work, because you have to do something with your time. Darwin had "fuck you money", he used it to fund his intellectual pursuits. Lots of intellectuals prior to the twentieth century were independently wealthy, many probably because they were independently wealthy. There was also the "Royal Society" and other similar agencies available at the time: it pooled resources and distributed it to fund various scientific pursuits, so even people who weren't financially independent could do science with a grant. The same system exists today, via government agencies such as the National Institutes of Health or independent agencies like the National Geographic Society. --Jayron32 20:02, 9 October 2012 (UTC)[reply]
Lots of intellectuals prior to the twentieth century were independently wealthy, many probably because they were independently wealthy. - What does this mean, Jayron? -- Jack of Oz [Talk] 20:36, 9 October 2012 (UTC)[reply]
That is, lots of people were employed as intellectuals because they otherwise didn't have to work 18 hours a day, seven days a week digging coal out of the ground or dragging a plow across the earth. That is, it is likely that many (not 100%, so go ahead and present counterexamples, because it won't disprove this point) people involved in intellectual pursuits were only able to do so because they had the means to provide themselves with enough leisure time to devote to studying a topic in detail. I agree, my phrasing was awkward, but what I meant to say was something along the lines of "Most intellectuals were employed as intellectuals because they had the wealth to do so". --Jayron32 21:18, 9 October 2012 (UTC)[reply]
Oh, I see. Thanks. -- Jack of Oz [Talk] 21:46, 9 October 2012 (UTC)[reply]
(edit conflict with above)Compare Darwin to Alfred Russel Wallace, who independently theorized natural selection as a mechanism by which evolution could occur. Wallace supported himself by selling specimens from his explorations (and his writings), but was for a period during his middle-age pretty financially insecure, and never wealthy like Darwin was. It's hard to generalize too much, but the mid 19th century can perhaps be seen as the transitional period from the gentleman scientist to the professional scientist. Buddy431 (talk) 21:51, 9 October 2012 (UTC)[reply]
There's still a distinction to be made, however, here because (with apologies to Fifelfoo who usually brings these things up), Wallace wasn't strictly "working class". He's middle class; the same class broadly that Darwin was: Darwin's family was more successful at being middle class, but as Wallace's biography notes, his father had "received a law degree" and Wallace himself " inherited some income-generating property". Neither was a member of the inherited Aristocratic class (i.e. neither was of the Nobility). Being the son of a Lawyer (even a non-practicing one) and being involved in land ownership and investment are hallmarks of a middle-class lifestyle and upbringing and outlook on life. The actual amount of wealth is important, but cannot be overstated here: Wallace and Darwin probably shared a lot more in value because they were both part of the Middle Class than either would have with a member of either the aristocracy or of the working or peasant classes. That is, Darwin has more in common with Wallace than he would a titled Duke of similar means has himself, and Wallace has more in common with Darwin than he would with someone from a long line of unskilled, uneducated laborers. Class standing (not just wealth) comes with inherent training and skills and worldview that open up certain types of lifestyle, regardless of wealth. It cannot be looked at solely as a function of cash in hand. Take a look from a different socioeconomic system: The early U.S.: Thomas Jefferson spent his whole adult life essentially broke, all the time. He was never not in debt, and severely, and yet (what would seem paradoxical) he lived the life of a landed aristocrat, spent money like it was in infinite supply, and generally lived like an aristocrat despite never having any actual money. How? Because he knew how to live like an aristocrat, and had access to the sort of networks and skills that made the money issue irrelevent. Looking at his balance sheet, he should have been living in the gutter begging for hand-outs. Why wasn't he? Because he came from a certain class that lived a certain lifestyle, and he knew how to work the system (inherently) to maintain that lifestyle without cash. Likewise, with Wallace and Darwin, it isn't the difference in actual cash-on-hand at any one time that defines and explains their shared pursuit of intellectualism: it's their status as belonging to the middle class, and not the working class. I think I may have overstated the wealth issue above a bit too strongly, wealth is certainly a factor, but it is also important to look at a person's entire social situation, which includes the values and worldview imparted by one's standing in a specific social class. --Jayron32 22:13, 9 October 2012 (UTC)[reply]
Thanks Jayron32 for the coverage, I agree with your class analysis on this point. History of Science does go into the difference between an era of scientists whose profession was leisure, and scientists for whom it was "just a job" even if it was a feted and well paid job that consumed years and required the liquidation of the meagre advances on rent or mortgages that labour aristocrats accumulate. David Philip Miller does a fair bit of work here ('The Paradoxes of Patenting at General Electric: Isador Ladoff's Journey from Siberian Exile to the Heart of Corporate Capitalism', Isis 102 (2011): 634–658. ; 'Was Matthew Boulton a Scientist? Operating between the Abstract and the Entrepreneurial', in Malcolm Dick, Kenneth Quickenden and Sally Baggott (eds), Matthew Boulton: Enterprising Industrialist of the Enlightenment (Ashgate, forthcoming, 2012)). By the time you get to thoroughly 20th century science, the majority of employees are either workers or some new and undiscovered class; both reliant upon the capitalist firm and payment for exertion for their subsistence. In fact, you may enjoy DP Miller & P.H. Reill (eds), Visions of Empire: Voyages, Botany and Representations of Nature (Cambridge: Cambridge University Press, 1996). The current term for the mechanisation of scientific inquiry under wage labour is technoscience last time I checked, but I'm not an expert here. Fifelfoo (talk) 22:26, 9 October 2012 (UTC)[reply]
Technoscience is an imprecisely used term usually to just mean "I don't think science and technology are separate things." I've never seen it used in a specifically labor context by practitioners of the history of science or science studies. --Mr.98 (talk) 02:11, 10 October 2012 (UTC)[reply]
Bugger, I thought I was reading the vague research programme thingies I've seen right; bugger all academic fads. Fifelfoo (talk) 03:11, 10 October 2012 (UTC)[reply]
I took it to mean "they became intellectuals because they could afford to be". A look at 19th-century censuses shows "Living on own means" in the Occupation column against a surprising number of names. Estate taxes did exist but did not really begin to bite until the late 19th and early 20th century; before this, invested capital or property that yielded a reliable income was passed down from to generation to generation without seriously aggressive depletion on each transfer. If it did get depleted, or if the eldest son got the lion's share, you could always enter the Church, use your social connections to secure a parish or parishes that yielded a healthy annual sum, pay a pittance to a poor curate for doing the actual work, and shazam! You're a respectable gentleman with enough leisure for intellectual pursuits and enough money to pay for them. - Karenjc 22:04, 9 October 2012 (UTC)[reply]
And on that point, a substantial number of British scientists, naturalists, archaeologists, historians etc etc in the 18th and 19th centuries, were indeed Anglican clergymen. Alansplodge (talk) 00:27, 10 October 2012 (UTC)[reply]
Ordination as a minister was the normal condition of residency for a lecturer in English universities and I'd guess elsewhere. Dmcq (talk) 12:03, 10 October 2012 (UTC)[reply]
It should be noted that in the late 19th century, when Darwin was really came into his scientific greatness, was when science was actually becoming a profession more broadly in England. Prior to the period, science was still primarily an endeavor to be pursued by classes that could afford not to have "real" jobs. By the end of the 19th century, you could have middle class and even working class scientists (and they called themselves "scientists," not "natural philosophers"). Darwin was a member of the older guard; his class situation was considerably different than Wallace's or Huxley's. (And it was mentioned once, earlier, but Darwin's multi/inbred-membership in the Darwin–Wedgwood family was responsible for most of his income.)
One point that Janet Browne makes in her biographies of Darwin is that the vast majority of the "data" that Darwin used in formulating and defending his theories was obtained by sending letters to correspondents all over the world. His postage bills were, by mid-19th standards, a veritable fortune. The money matters not just for the idle time it gave him. --Mr.98 (talk) 02:11, 10 October 2012 (UTC)[reply]
I'm not sure where to put this snippet, so I'll put it here. As our article says, "Darwin's father organised investments, enabling his son to be a self-funded gentleman scientist". The answer was, as I implied, there in our article. --TammyMoet (talk) 08:50, 10 October 2012 (UTC)[reply]

latch-hooking

A search in Wikipedia.org for "latch-hooking" information will only bring the user to information about "rug hooking". I was hoping to find information about how the yarn strips are precut for the latch-hook kits for making bath rugs, (though I am not personally interested in bath rugs), which can be purchased in craft supply stores. I am hoping to be able to purchase a home version electric machine for cutting yarn into strips the length I want, so I can stop cutting them by hand. Other types of INTERNET SEARCHES elsewhere bring me to various types of machines with little information, leaving me without confidence that it is what I need, if it even exists. If it does not exist, I will request a local university have an engineering student invent it.173.216.80.111 (talk) 20:45, 9 October 2012 (UTC)[reply]

While I'm certain that such a machine must exist, the question is whether there's a version for home use. In theory it sounds fairly simple: a device to pull yarn a set distance, cut it, push the cut yarn into a collection basket, then grab the new end, pull it the set distance again, and repeat until the set number is reached or the yarn is exhausted. An industrial version might be designed to do 100 pieces of yarn at a time, though. StuRat (talk) 00:29, 10 October 2012 (UTC)[reply]
Searching Google under pre cut lengths of yarn, I found the following: 1) a homemade tool at www.ehow.com/how_5576880_cut-latch-hook-yarn.html (that's a blacklisted site, but I don't know why) for cutting multiple strands at a time; and 2) a U.S. Patent application (with drawings) for a hand-held device that seems to load and cut from a continuous length of yarn. Bielle (talk) 00:49, 10 October 2012 (UTC)[reply]
That blacklisted site just says to wrap it around a piece of cardboard to get the length right, then cut it manually. That probably is the most sensible method for home use, unless you intend to sell the electric device to others. It's similar to the "Cricut", which cuts pieces of paper following patterns, and apparently there's a market for that. StuRat (talk) 01:00, 10 October 2012 (UTC)[reply]
The point of the home-made cardboard is that you can cut multiple pieces at once and they are all of equal length. It's the "multiple" that the OP seemed most interested to find. Bielle (talk) 02:01, 10 October 2012 (UTC)[reply]
I'm not sure how good that cardboard would be at guaranteeing a common length. The cardboard can bend, and the yarn can stretch. A piece of wood should solve the bending issue. As for stretching, you just have to be careful to apply the same amount of force to each piece of yarn. StuRat (talk) 03:12, 10 October 2012 (UTC)[reply]
I've used ehow and a number of similar sites like associated content before, I don't think I ever actually looked in to the reasons for the blocklist, but it isn't surprising. These are sites which accept content from random people and pay them a small percentage of the advertising revenue, therefore there's strong incentive for people who don't care much about being considered spammers, to spam their content which is almost definitely not WP:RS so unsuitable for references and generally not suitable for external links either anyway. Nil Einne (talk) 04:15, 10 October 2012 (UTC)[reply]


October 10

French Guiana

What is French Guiana's status relative to the rest of France? Is it like a colony, or is it more analogous to Hawaii? --168.7.237.39 (talk) 03:35, 10 October 2012 (UTC)[reply]

See Politics of French Guiana. 69.62.243.48 (talk) 03:54, 10 October 2012 (UTC)[reply]
To be more explicit, French Guina is both an Overseas_department and a Regions of France. They are not analagous to a U.S. state like Hawaii, in that they cannot pass statutory laws (they can levy taxes and have discretionary power over a lot of spending in the region, just like all regions of France). They are like Hawaii in that they are governed pretty much identically to regions and departments in Metropolitan France. They have proportional representation in the National Assembly (France) and French Senate. The phrase "An Integral Part of France" is often used with the overseas departments and regions. Contrast this to French Polynesia, an "Overseas country of France", or Saint Pierre and Miquelon, an "Overseas collectivity". Buddy431 (talk) 05:47, 10 October 2012 (UTC)[reply]
To expand and clarify on Buddy's excellent answer, France is not organized like the United States. It has nothing like U.S. states. France does have "Regions" and "Departments" and "Municipalities", but these are more like "counties" and "civil townships" in the United States: they are administrative divisions which have highly limited powers on their own; the actual power to govern France and to pass laws lies solely with the French government in Paris, France is a unitary state, where as the U.S. is a federation, so comparisons to U.S. States aren't entirely useful in trying to understand France. That being said, French Guiana is a Department. It is often called an "overseas department", but that qualification is one of description and not of quality: French Guiana is no less part of France than any other Department. In that way it is like Hawaii. --Jayron32 06:19, 10 October 2012 (UTC)[reply]
Most people think of France as being a European country. It is that, but it's also a North American country, a South American country, and an African country. -- Jack of Oz [Talk] 06:41, 10 October 2012 (UTC)[reply]
Very true; but as a percentage of population, by culture, by hegemony, and by the general social and economic spheres, it is basically a European country. It's not inaccurate to call it a European country except in the most pedantic, technical manner. --Jayron32 06:48, 10 October 2012 (UTC)[reply]
Of course. I would never describe France as other than a European country, unless the context demanded it, as it did above. It is overwhelmingly European in all the ways that matter to most people. But geographers are people too. -- Jack of Oz [Talk] 21:45, 10 October 2012 (UTC)[reply]
[citation needed] --Jayron32 13:21, 11 October 2012 (UTC)[reply]

I tried in vain to convince other editors that French Guiana technically speaking is an integral part of France at Wikipedia:Categories_for_discussion/Log/2010_June_20#Category:Brazil_.E2.80.93_French_Guiana_border, but somehow the supposition/misconception that any place in the tropics has to be a colony is quite strong. If anyone is up for reopening a CfD again, I'm up for it. --Soman (talk) 07:55, 10 October 2012 (UTC)[reply]

Just remind them that Alaska, Hawaii and Texas are all part of the same country. -- Jack of Oz [Talk] 10:59, 10 October 2012 (UTC)[reply]
Reopening a CFD from 2.5 years ago about an issue that was later resolved by other means Yeah, you have fun with that. --Jayron32 12:19, 10 October 2012 (UTC)[reply]
How was it resolved? --Soman (talk) 21:50, 10 October 2012 (UTC)[reply]

Jefferson's views on government

When Jefferson was writing the Declaration of Independence, what did he believe an ideal government should provide for its citizens? --Jethro B 04:18, 10 October 2012 (UTC)[reply]

See Thomas_Jefferson#Political_philosophy_and_views --Jayron32 04:32, 10 October 2012 (UTC)[reply]
I've seen that. I was wondering when he was writing the Dec of Indep himself, and how it's reflected in the Dec. --Jethro B 04:34, 10 October 2012 (UTC)[reply]
It's actually a bit tough to understand Jefferson's political philosophy because, paradoxically, he didn't write much aside from that. Much of what we know about his political philosophy comes from copies of his speeches, what others have written about him, etc. Unlike many of his contemporaries, like John Adams or James Madison or Alexander Hamilton, all of whom wrote extensive and detailed works on the nature and organization of government, Jefferson's only proper "book" written in his lifetime for publication was Notes on the State of Virginia, which does contain some of his political philsophy, but meanders into lots of other issues and isn't really a focused work of political thought like the Federalist Papers (Madison/Hamilton/Jay) or Adams's Thoughts on Government. Jefferson wrote a handful of other pamphlets, tracts, and short articles but wasn't a big writer. If you want to get to know him best, then Notes on the State of Virginia is going to be your best shot; but you're going to have to slog through some rather boring and unrelated stuff on navigability of rivers and descriptions of native animals. Also rather inconsistently, Jefferson's practice of politics ran rather counter to his professed philosophy: For example, he claimed that he favored a republic founded on the agrarian life of the simple gentlemen farmer as the ideal citizen, yet he spent most of his political life criticizing Adams as a "monarchist" and elitist, Adams the gentleman farmer and Jefferson the Aristocrat. He's a complex fellow. --Jayron32 05:06, 10 October 2012 (UTC)[reply]
Just to clarify something I said which may be taken the wrong way: Jefferson was an extensive writer. He wrote constantly, but most of his writings are personal letters and were not works intended for publication. He didn't write works of political philosophy specifically; he wrote lots of stuff from which one can extract his political philosophy. But I don't know that he had as many concrete ideas on the minutia of government the way that Madison or Adams did; he was more of a "big idea" guy, from my impressions of reading his works and from reading analyses of his works. --Jayron32 05:28, 10 October 2012 (UTC)[reply]
In his Notes on the State of Virginia, he states "The legitimate powers of government extend to such acts only as are injurious to others." See here page 285. So that may, perhaps be a statement of his opinion as to the legitimate ends of government. Its a pithy quote, but it doesn't amount to a whole lot. --Jayron32 05:34, 10 October 2012 (UTC)[reply]

Jayron32 is mostly correct, unfortunately the U.S. Supreme Court never got that memo . . . ala Separation of church and state, as Jayron32 puts it "personal letters and were not works intended for publication", but that is a whole other discussion on how some will force themselves to see a unified political theory in what may have simply been the daydream musing spitballing of a founding father. Marketdiamond (talk) 08:03, 10 October 2012 (UTC)[reply]

Thankfully, the U.S. Supreme Court has Other official documents written by people who aren't Thomas Jefferson to help them resolve the issue of Church and State. --Jayron32 12:16, 10 October 2012 (UTC)[reply]
Also, Jefferson wrote most of his more political letters with the expectation and understanding that they were not really "private", but that they would be read and repeated by many people. --Stephan Schulz (talk) 13:43, 10 October 2012 (UTC)[reply]

The answer to your question depends on who you ask. People have usually characterized Jefferson's view of government as expressed in the Declaration as what we would today call "libertarianism" or "classical liberalism", that is, a limited government with emphasis on individual rights. In the 1970s, Garry Wills controversially challenged this view. Start with this essay to learn more. —Kevin Myers 16:13, 10 October 2012 (UTC)[reply]

Occurrence of Demonic Possession

After watching a television show about demonic possession, I was curious to know, have their been any cases of possession where the victim was a non-believer? It seems to me that possession seems to occur amongst people that believe it's possible in the first place.70.171.18.234 (talk) 05:17, 10 October 2012 (UTC)[reply]

Even more significantly: diagnosis is often performed by people predisposed to belief in its possibility. So is treatment, which can be lethally dangerous to the subject. Some of the victims are too young to have informed beliefs on the subject. (I went to a school where we were taught that illness was a physical manifestation of evil - not quite the same thing, but closely related. I am a Christian, but I didn't believe the claim then and I don't believe it now.) I am unaware of any cases of demonic possession occurring where the people responsible for diagnosis (whether including the victim or not) did not already believe that such a thing can happen. AlexTiefling (talk) 10:19, 10 October 2012 (UTC)[reply]
I have no sympathy for claims of demonic possession or any other religious nonsense, but your first sentence is a tautology. How can diagnosis be performed by people who don't believe in its possibility? If you don't believe demonic possession is possible, why would you ever diagnose it? The same goes for treatment: if you don't believe in demonic possession, why would you treat it? What could "treatment" possibly mean you don't think the thing you're treating exists? --140.180.242.9 (talk) 17:55, 10 October 2012 (UTC)[reply]
It's a truism rather than a tautology, and that was rather my point, as well as (I suspect) the point of the original question. The (fallacious) diagnosis is only made by people who believe in it. AlexTiefling (talk) 21:02, 10 October 2012 (UTC)[reply]
It's unlikely, largely because of selection bias. When someone displays unusual behaviour and/or an apparent personality change, the people around them are the first to notice the problem. If the sufferer believes in demonic possession, they are likely to be part of a network (family, friends, church) that believes in demonic possession. Possession is therefore one possible diagnosis. If the sufferer does not believe in possession then they are less likely to surround themselves in their personal life by people who accept it as an article of faith. Possession is therefore unlikely to be considered as a possibility by those who are concerned for them, who are more likely to seek a medical or psychological diagnosis, which will not (in most countries) include demonic possession. It is perfectly possible that believers could diagnose possession in a nonbeliever, with potentially dangerous consequences if the sufferer is prevented from, or incapable of, getting a second opinion or access to mainstream medical care. Googling "learning difficulties demonic possession", for example, while writing this answer, was quite a disturbing experience. - Karenjc 12:49, 10 October 2012 (UTC)[reply]
The Demonic possession article has something to say about this in general. In the UK there has been some concern about alleged cases of possession leading to serious crime including child abuse and murder (see this report from the BBC) and it seems to involve people "from Africa, South Asia and Europe". For a particular example see the rather distressing case of Victoria Climbié. In that particular case, I am unsure whether an 8 year old child is capable of understanding or actually believing in demonic possession, although she might believe whatever an adult tells her. Astronaut (talk) 18:54, 10 October 2012 (UTC)[reply]

Domestic Twinning

I notice in List of twin towns and sister cities in Italy that some places are linked to others also in Italy. For example, Busto Arsizio is twinned with Domodossola, and Cagliari has three Italian twins (quadruplets?).
As promotion of international relationships was, I thought, pretty central to the whole twinning idea, what's the point of this, and do any other countries do likewise? I've checked a few of the other entries in Category:Lists of twin towns and sister cities and can't see any. Rojomoke (talk) 06:48, 10 October 2012 (UTC)[reply]

It's not necessarily a promotion of international relationships. Germany still has a number of cities which are twinned with other German cities. This, of course, is because Germany was virtually cut in half for a while. Italy has historically been a nation with a number of states for most of its history. This is possibly why. In Italian, it is called 'comune gemellato' or 'gemellaggio'. As for whether other countries do this, I cannot answer that. KägeTorä - (影虎) (TALK) 07:48, 10 October 2012 (UTC)[reply]
In China, there are some more affluent cities or towns on the eastern seaboard who are twinned with poorer cities or towns in the west, via a central government program designed to help boost development in the interior. --PalaceGuard008 (Talk) 09:44, 11 October 2012 (UTC)[reply]

Exactly how does Rebecca Black get to maintain composure better than Jason Russell?

As you know, Jason suffered an epic breakdown on the streets of San Diego after a mix of success and acerbic criticism of his Kony 2012 campaign, and was hospitalized for about 6 1/2 months.

However, Rebecca Black received more acerbic criticism in her Friday music video and was more of a one-hit wonder (though her hit was due to the derideable qualities of her song(s)), and yet, over 18 months later, she remained sane, calm and composed. Not to mention that Rebecca Black is 19 years Jason's junior.

So how is it that when Rebecca is so much younger, hence presumably quite a bit more delicate, she never broke down amidst and in reaction to the negative responses to her songs? What's her secret? What did she have that Jason didn't? What did she not have that Jason had, that (may have) directly caused Jason's breakdown? --70.179.167.78 (talk) 09:09, 10 October 2012 (UTC)[reply]

Well, differences in handling stress have been noted in people, and must have both genetic and environmental causes, but I don't believe we know the details of exactly what makes one person handle stress better than another. This would be valuable info, though, so we could know who can handle high stress jobs, and who will cave under the pressure. StuRat (talk) 09:32, 10 October 2012 (UTC)[reply]
I think the highly serious intent of Kony 2012 may be relevant. Other factors certainly come into play, but I wouldn't wish to speculate on them in a way which might breach our policy on biographies of living people. AlexTiefling (talk) 10:15, 10 October 2012 (UTC)[reply]
You're talking about n=2. Two people are not trends. Whether Black or Russell are more "indicative" of trends of stress is unestablished. They are individuals with different pasts, and their situations are not comparable, either. I'm not sure any reasonable comparison can be made on this front. --Mr.98 (talk) 13:04, 10 October 2012 (UTC)[reply]
Also, much (all, IMO) of the flak Rebecca Black got was bullying by young men butthurt that she got attention that should have been theirs by right of birth. The Kony 2011 flak, on the other hand, was based on substantive criticism of the issues at hand. What's easier to shrug off: the wholly irrational, wildly overblown hatred of random spoiled, entitled strangers, or thoughtful criticism and plausible allegations of fraud? --NellieBlyMobile (talk) 17:04, 10 October 2012 (UTC)[reply]
There are plenty of people around the world who have been through much worse than either Rebecca or Jason, but didn't break down and run around naked in public. Think about people in criminal courts, facing death penalties or life in prison. Remember Casey Anthony, whom half the country thought was guilty of murder? What about Jerry Sandusky, O.J. Simpson, or Ted Bundy? How about people who get shipwrecked, tortured, put in North Korean labor camps, or forced to fight in a trench in WWI? How about people who live in besieged cities, go through famines, get cystic fibrosis (and slowly drown from the inside), get persecuted for religious reasons, get marched to death in a genocide, get lost in a desert, are captured as child soldiers and forced to beat their relatives to death? Billions of humans and other animals--including both our heroes and our villains--have endured tremendous adversity and overcome it or died trying. You only heard about Jason's breakdown on the news because it was highly abnormal--if people usually broke down in that kind of situation, you wouldn't have heard about it, because everyone would have already known that he would break down. Similar, you never heard the news proclaim "NEWSFLASH: REBECCA BLACK DID NOT RUN AROUND ON THE STREET NAKED!!!" because we expect teenage girls to be capable of enduring that kind of humiliation without having a complete breakdown, at least not in public, so it's not newsworthy that she did exactly that. --140.180.242.9 (talk) 17:43, 10 October 2012 (UTC)[reply]

Israeli PM's red line for Iran

"The reference desk does not answer requests for opinions or predictions about future events" - and see WP:NOTFORUM. There are no encyclopaedic answers to the OP's original question
The following discussion has been closed. Please do not modify it.

What did he mean with that? That if it's crossed there will be war? or what? thank you. Iowafromiowa (talk) 11:04, 10 October 2012 (UTC)[reply]

In the context of a dictator who the Israelis are convinced is "close" to a nuclear weapon and giving speeches multiple times a year on how Israel must cease to exist or be pushed into the sea etc. then yes the red line would be war, the alternative to hear the Israelis tell it would be their eventual destruction. Forgetting the politics of it for a moment if someone with a weapon kept threating you, many juries would aquit if you hit back or even killed the person making those threats with the weapon, many police departments see their officers hitting back or even shooting dead a perp making threats while branishing a weapon as an acceptable use of force as well, humans are humans despite the scale. Marketdiamond (talk) 11:30, 10 October 2012 (UTC)[reply]
This is both misleading about the "red line" (Netanyahu has not said it will be "war"), as well as about Iran. Mahmoud Ahmadinejad is not a dictator — his will is not absolute, he does not have the ability to dictate policies. He is elected, and he's just about at the end of his term, at that. Iran isn't completely free, but calling it a dictatorship is misleading . They have a complicated civil society, balances of powers, and Ahmadinejad, for all of his rhetoric and attention, is actually comparatively weak within the Iranian political ecosystem. (It also isn't the case that he has ever advocated Israel being pushed into the sea or anything that strong. Most of his famous anti-Israeli state statements have been mistranslated, but that doesn't stop people from repeating them, even after correction multiple times. He makes inflammatory comments, to be sure, but his rhetoric is not as bellicose as that.) The comparison with individuals and juries is not germane and likely not legally correct, either. --Mr.98 (talk) 13:20, 10 October 2012 (UTC)[reply]
Yeah, Ahmadinejad's a peace-loving democratically-elected (2009 rigged elections and Green Revolution, say what?) bunny rabbit whose vision of peace consists of transforming Israel/West Bank/Gaza into one big amusement park for everyone to have fun! The proof is one controversy regarding a speech he gave? We could just as easily cite tens of other explicit speeches he gave calling for Israel's destruction, such as these recent ones ([5], [6], [7], etc). No surprise that Secretary General of the United Nations Ban ki-Moon has called on Ahmadinejad explicitly to stop threatening Israel. --Jethro B 23:39, 10 October 2012 (UTC)[reply]
I'm no lover of Ahmadinejad — don't make a straw man out of what I said. He's not a dictator, though. There were 2009 election irregularities, but typically we view things on that level as a spectrum of corruption rather than "dictatorship vs. democracy." The "wipe off the map" speech is the most famous speech of his, and is constantly brought up as proof of his allegedly genocidal intentions. His speeches, when not accidentally or purposefully mistranslated, generally call for an end of the specific Zionist regime in Israel, not Jewish people. His audience for that is obvious — his own domestic base, and like many in the Middle East he gets a lot of domestic capital by subscribing in a vague way to the Palestinian cause. He's never said anything though that would imply that he intends to nuke Israel, and there is exactly zero reason to suspect that's his goal. That doesn't mean Israel should be happy about the idea of a nuclear Iran, but there are facts and then there is just hyperbole. --Mr.98 (talk) 01:37, 11 October 2012 (UTC)[reply]
I never said he's a dictator - just that it's likely his rule is illegitimate. At either rate, I (and most analysts will agree) view him as more of a "puppet" of the Supreme Leader. I don't know if that speech is used the most often, I do not care. If I wanted to make a case of explicit statements calling explicitly for Israel's destruction, I'd use the numerous times he has said this explicitly without dispute - even Ban ki-Moon has condemned him for this explicitly.
As for using the nukes on a country (be it Israel, Saudi Arabia, American bases, etc)... I don't read crystal balls, so I don't know what'd happen. The possibility certainly exists, and is not one anyone would like to live under. You have a fanatical enough regime and fanatical enough leaders, believing in a radical interpretation of their religion, you don't know what could happen, if they're willing to make that sacrifice. Even if not, Iran still supports terror organizations (yeah, this isn't an article so I'm going to say that) like Hamas and Hezbollah (let alone their own IRGC), so imagine if they do get nukes what could happen. It's not about using them. If they would get it, that would be a significant deterrent to responding to any terrorist attacks by these organizations. Or the prospects of setting off a nuclear arms race (Saudi Arabia has confirmed they'd seek nukes) in a volatile Middle East isn't too good either... All of this without actually using the nukes. Even just attaining the capability to produce nuclear weapons is scary enough - in just a few weeks, in some underground new facility that no one knows about, with just a few top scientists, a nuke can be producd at any time. North Korea all over.
But it's like what I always say - is this something we want to risk? Should we turn a blind eye to it, give the benefit of the doubt, and be naive and hope for peace? I wish we could. I don't think it's realistic though. --Jethro B 02:26, 11 October 2012 (UTC)[reply]
I think that by dictator he means Ayatollah Ali Khamenei, rather than Ahmadinejad. Futurist110 (talk) 22:13, 10 October 2012 (UTC)[reply]
Khamenei is a theological leader, but he's not a dictator, either. (I think of the Iranian mullahs as being roughly equivalent to the US Supreme Court — they are not democratically elected, they are not subject to very much representative power, they have certain abilities to make very big decisions, but they are not completely unlimited in their abilities, nor can they totally disregard the reality of things.) Iran's political system is more complicated than that. Abstracting it down to "one guy must be in charge, they're backwards and cruel" is just a reflection of ignorance. --Mr.98 (talk) 01:37, 11 October 2012 (UTC)[reply]
Ironically that metaphor applies to both sides. Israel is a nuclear power who is constantly threatening to attack Iran.A8875 (talk) 16:05, 10 October 2012 (UTC)[reply]
However, Israel doesn't refuse to acknowledge Iran's existence ("the Persian entity ?"), set it's destruction as a goal, and specifically fund groups dedicated to that goal. StuRat (talk) 20:47, 10 October 2012 (UTC)[reply]
Yeah, and also the main reason that Israel is threatening to attack Iran is because Iran was threatening Israel first (such as by sponsoring terrorist attacks on Israeli civilians and as StuRat said, to set the destruction of Israel as its goal). To be honest I think it's very stupid for Iran to alienate Israel and the West this much. Iran is close to some hostile Sunni Arab states, and if the Iranian leadership was more rational an alliance with Israel might have been much smarter than threatening to eliminate Israel. Futurist110 (talk) 22:16, 10 October 2012 (UTC)[reply]
Israel does directly fund groups that commit acts of terrorism and assassination in Iran — that is pretty well known. They do certainly desire to destabilize the Iranian regime; "regime change" and "regime annihilation" are fairly synonymous in this context. Iran does not set the killing of the Israeli population as its goal; it's goal is to end the Israeli occupation of Palestinian territory and its establishment as a specifically Jewish (as opposed to pluralistic) state. You can disagree with that pretty solidly; my point is that viewing this as a simple "Iran wants to kill Israel, who has never done anything to anybody" is a really naive approach to it. I don't favor Iran's intentions, but I'm pretty wary of Netayahu's intentions as well. --Mr.98 (talk) 01:37, 11 October 2012 (UTC)[reply]
What wonderful goals! Guess there must be some Freudian slips there, with the few diffs I provided above as just recent examples of explicitly saying "destroy Israel." And burning the American, British, and Israeli flags, while chanting "Death to Israel! Death to America! Death to England!"
Btw, elections are coming up in Israel in January, like they do every 3/4 years. It's called a democracy, where the people elect their own regime. No one needs to tell them who to elect or to wish for 30 years for some "regime change," regardless of whether the "regime" is left or right on the political spectrum, always that same "wish." --Jethro B 02:26, 11 October 2012 (UTC)[reply]
Regarding "end the Israeli occupation of Palestinian territory", since all of Israel is defined by Iran as "occupied Palestinian territory", this means that Israel must cease to exist to satisfy Iran. Israel has no such goal for Iran. StuRat (talk) 02:35, 11 October 2012 (UTC)[reply]
One point that should be clarified is the difference between attacking a country and fighting its people and attacking nuclear facilities of a country (or at least, having the intention of only this, who knows what would develop). Tragically, I don't think this has really been stressed by world leaders advocating this (even as a last resort, as all do), although judging by Operation Opera, Israel's miraculous raid on the Iraqi nuclear reactor that helped Operation Desert Storm significantly and to liberate Kuwait, which targeted only the Osirak reactor. --Jethro B 23:39, 10 October 2012 (UTC)[reply]
The main question being debated by the people who actually make decisions about this is whether attacking the nuclear facilities would lead to a wider conflict, and whether it would slow, rather than hasten, Iranian nuclear ambitions. It's a thorny problem and there are different types of examples. It is not quite the same thing as the Iraqi situation; the Iranian nuclear complex is far more robust and spread out. (Another point of comparison is the Israeli bombing of the Syrian reactor a few years back.) There have been some excellent articles on all of this in the New Yorker not too far back; it's clear that Netayahu thinks that they can bomb in a contained way, but it's also clear that most other Israeli defense analysts are dubious of this. --Mr.98 (talk) 01:37, 11 October 2012 (UTC)[reply]
There are definitely risks and any military option should obviously be a last final doomsday resort, something that everyone agrees on. You gotta assess the risks, assess the consequences, and compare it to the consequences of not doing such an action, and then make a choice. Luckily, I'm not the one making that choice. --Jethro B 02:26, 11 October 2012 (UTC)[reply]
Netayahu's invocation of the "red line" was as a threat in and of itself. ("At this late hour, there is only one way to peacefully prevent Iran from getting atomic bombs. That's by placing a clear red line on Iran's nuclear weapons program. Red lines don't lead to war; red lines prevent war.") In the context of his speech, he said that in the past, making firm stands against certain outcomes has led to positive diplomatic success in the past. (He specifically invoked the Cuban Missile Crisis as an example of this, though it is misleading — JFK did take a hard stand, but he also bargained and compromised to get the outcome he wanted.) He didn't actually say he was drawing a red line, just that a red line should be drawn (specifically at the place in which Iran had enough HEU separated for a single bomb). It's commonly interpreted that Netayahu is supportive of conventional bombing of Iranian nuclear facilities, and that he would like US commitment to supporting Israel in this endeavor. But he's been deliberately vague about this. There is also significant technical uncertainty about where the "red line" would be or how you would know if it had been crossed, as well. --Mr.98 (talk) 13:20, 10 October 2012 (UTC)[reply]
I viewed the remark as saying an ultimatum should be set, which threatens an attack (not necessarily war) if Iran does not stop. StuRat (talk) 20:49, 10 October 2012 (UTC)[reply]
Yes, that's exactly right. "Don't pass this line, lest you shall NOT pass GO, you shall NOT collect $200." If Iran is willing to compromise, that's great, but this is exactly what the IAEA has been trying to do, and each time they try and attempt to gain accessd to an underground site like Parchin, they're smiled at but turned away at the door. --Jethro B 23:39, 10 October 2012 (UTC)[reply]
The IAEA has actually been able to get plenty of access to Iranian sites — Iran is probably the most inspected country in the world right now. The difficulties hinge on how much access has to be given, and how to establish exactly when the IAEA has access to any given site. The IAEA is not allowed to just inspect anything it wants, willy nilly, because it would compromise other military research. Iran is allowed to keep secrets — it just isn't allowed to violate the NPT. There are practical difficulties in enforcing such a thing, and the details are somewhat boring and esoteric aspects of international law (e.g. did Iran accede to the NPT's Additional Protocol because the Shah approved of it?). --Mr.98 (talk) 01:37, 11 October 2012 (UTC)[reply]
Interesting discussion per the response to my post you can parse it all you want but Iran is a dictatorship, technically the Soviet Union Premier was "elected" (and even Hitler was) but no one had any doubts it was an iron ruled dictatorship. Also you may parse the red line thing all you want as well, the effect of the speech was clear, submit or risk war, who's right and who's wrong, StuRat made some excellent points in that analysis but the OP's question was what was meant by the speech, not how to resolve the conflict or which nation is more guilty. I have other thoughts on the domino effect of this if it does occur but I'll leave at that. Marketdiamond (talk) 23:04, 10 October 2012 (UTC)[reply]
The situation is Iran is nothing like the situation in the USSR. Democracy is a spectrum, it is not an all-or-nothing thing. There are actual Presidential elections in Iran, and the results of them actually matter when it comes to things like nuclear policy. That is a real difference from the USSR, Nazi Germany, or any other totalitarian regimes you want to mention. It does not mean that Iran is a free and open society. Ahmadinejad will be gone in 2013, which is more than you can say for, say, Mr. Putin. --Mr.98 (talk) 01:37, 11 October 2012 (UTC)[reply]
He'll be gone, definitely, especially after his fallout with the Supreme Leader... --Jethro B 02:26, 11 October 2012 (UTC)[reply]
You clearly know nothing about Iran's constitution if you think it's similar to the Soviet Union's. If Iran is a dictatorship, who's the dictator? Surely you're not under the impression that Ahmadinejad has any room to maneuver to the left on the nuclear issue, considering that both Khamenei and his electoral opponents (particularly Mousavi) are against him? The most that can be said is that Iran is an ideological dictatorship, for the reason that only candidates with a certain set of beliefs are allowed to run for public office. --140.180.242.9 (talk) 01:41, 11 October 2012 (UTC)[reply]

In response to the OP question... It doesn't necessarily imply war. You'll have analysts saying, "Yeah, they're 100% prepared and ready to go to war or launch a strike if necessary, no doubt about it." Then you'll have other analysts saying, "There aren't any actual intentions to go to war, all of these are just rhetoric and threats to prevent a nuclear-Iran." The red line, in and of itself (which Netanyahu drew at 90% uranium enrichment at the United Nations), is meant to convey a message that that line would be the last straw. Go up to it, you're good. Concerned, but good. Pass it, and that's unacceptable. --Jethro B 23:39, 10 October 2012 (UTC)[reply]

Don't see anything there I would disagree with Jethro B, only thing is I would caution anyone from paying too much attention to "analysts" especially ones that publicize their findings/conclusions. Even the best CIA ones were still advising the Berlin Wall would not fall as it was on CNN falling, and then there was that whole Yellow Cake/nuclear devices thing with Iraq but those are discussions for another topic. Marketdiamond (talk) 00:03, 11 October 2012 (UTC)[reply]
Yes, I'm merely showing that there are 2 completely different ways of thinking - both possibilities seem like the only possibilities to me. If there are more, I am unaware of them. --Jethro B 00:58, 11 October 2012 (UTC)[reply]
The most interesting stuff has been that which has come out about internal Israeli defense thinking on the matter. There's considerable dissent within the ranks, which is itself an interesting datapoint. One of the reasons we know this is because whenever there is internal dissent, both parts of that argument start leaking things to the press. --Mr.98 (talk) 01:37, 11 October 2012 (UTC)[reply]
In democracies, people have opinions and that should be respected and taken into account when considering any action. Having a different opinion though isn't the same as dissent. --Jethro B 02:26, 11 October 2012 (UTC)[reply]
Huh? Dissent means "having a different opinion". As far as I know, it usually has a positive connotation: dissenting opinion (in the Supreme Court), suppression of dissent, etc. --140.180.242.9 (talk) 02:38, 11 October 2012 (UTC)[reply]

Could I buy Greek debt?

We hear a lot in the news these days about the amount of trouble Greece is in. A major issue is that they need to borrow huge amounts, but are also seen as unstable, so that the interest rates they must pay to borrow are extremely high. Say I, a private citizen from the UK, decided that I thought the risks were worth the rewards, and I wanted to lend Greece some money at the market rate. Could I, and if so how? Also, is there a minimum amount I'd have to 'invest' in this or is it very fluid? 86.166.186.159 (talk) 13:01, 10 October 2012 (UTC)[reply]

Sure, you can: you buy Greek bonds. This is basically the same as saying, "I will give you some money, and I expect to get it back, with some return, after a certain amount of time." Know, of course, that doing so entails significant risk — you may not get your money back. How you go about buying actual bonds as an individual, I don't know, but a financial advisor or broker can probably set it up. My understanding is that for many bonds there are minimum investments for it to be worth a broker's time to set up — on the order of a few thousand dollars USD, in the USA. I don't know about buying Greek bonds in the UK, though. But this is a fairly standard sort of transaction. --Mr.98 (talk) 13:27, 10 October 2012 (UTC)[reply]
This suggests that the vast majority of Greek debt is held by institutional buyers, including banks, insurance companies, pensions and mutual funds. You could find out which mutual funds have invested in it, and buy shares in them. Again, you'd want to talk to a broker about it, but it may or may not be possible for an individual to buy bonds on a small scale (but it might be, I don't really know). Buddy431 (talk) 20:13, 10 October 2012 (UTC)[reply]
Why not buy National Bank of Greece (NBG NYSE) or a like institution, although bonds offer more safety, at this point with Greece's economy bonds may be as unsafe as stocks (you may be first in line to be paid with bonds but if the country goes under no one is getting paid bondholders or stockholders). I doubt Greece will actually go under this or next year but the bonds may have as much risk as the stocks. Marketdiamond (talk) 22:56, 10 October 2012 (UTC)[reply]
The National Bank of Greece is not the same as the Greek state bank; that's the Bank of Greece. Andrew Gray (talk) 05:24, 13 October 2012 (UTC)[reply]

Federal Penitentiary Service (Russian prison) death rate

A lot of you probably have recieved an email asking to sign a petition to release Pussy Riot from prison and the one I got from care2 today says the Russian Prosecutor General has claimed that 2,000 people have died in or en route to Russian prisons in the last six months. I am not so interested in Pussy Riot (they do stuff like perform sex acts with dead chickens for publicity and I'd have give them two years in a mental home rather than a prison) but the Russian prison thing picqued my interest and lo and behold I searched the internet and found practically nothing that didn't say "Pussy Riot" on it or reword the email I got. So, I am thinking what a load of lies, but Russia is notoriously tough on its people so, is there anything to it? It could do with a note on the article here at least if it is true. Any references worthy enough for WP? ~ R.T.G 15:07, 10 October 2012 (UTC)[reply]

It wasn't a sex act with a chicken, it was stealing a chicken by shoving it up a woman's vagina. And it wasn't Pussy Riot, it was Voina. 109.99.71.97 (talk) 20:39, 10 October 2012 (UTC)[reply]
Technically it was a protester that was with Voina, kind of unclear if it was actually one of the groups members, though Voina seemingly celebrated the act. I would seriously doubt the 2,000 in last 6 months statistic. Just doing the quick math that seems to approach a Stalinst death count and the smell test would dictate we would be hearing about that tons more in the media today. Plus from what I've read of Putin he may govern in Stalinst ways but he is not a fan of Stalins gulag or killings, there have been stories where Putin and his ilk have done some very creative things to increase population and for lack of a better term "babymaking" with some observers remarking that the vast expanse of the eastern frontier is bordered by the exploding populations of China, India and the like . . . I'd doubt that thousands are dying under Russian incarceration every 6 months but then again Putin has never been known for his unified logic. Marketdiamond (talk) 22:54, 10 October 2012 (UTC)[reply]
Russia has about 700,000 people in prison. Assuming the age distribution of prisoners reflects that of the general public (it probably doesn't), you could reasonably expect about 10,000 of them to die of old age every year. --Carnildo (talk) 01:56, 11 October 2012 (UTC)[reply]
"В 2010 году в исправительных учреждениях умерли 4423 человека.... более 90 процентов из примерно 830 тысяч заключенных больны." - "In 2010, 4423 persons died in [Russia's] correctional institutions. ... Over 90% of some 830,000 prisoners are sick", according to the Federal Penitentiary Service's officials. -- Vmenkov (talk) 04:40, 12 October 2012 (UTC)[reply]

Who were the 2 Mass. House members who voted against Romney Care?

I have read your page on Mass. health care reform. Romney has said there were only 2 votes against it by the time they were done drafting the bill. I have been able to determine that those votes were in the house. I am curious who the nay votes belonged to, what party they were members of, and what their reasoning was for voting against the bill. I have looked for records on the Mass. house site but they don't seem to have vote records. I tried calling them but got no answer. Thanks for your help!71.54.146.92 (talk) 22:58, 10 October 2012 (UTC)[reply]

Did a search on the MA House site but only goes back to 2009, also did several different Google News searches around April 2006 and then Spring 2006, several sources do cite that "2" voted against never with names (NPR, Bloomberg, etc.) also searched just general google searches for "2 against Romneycare" and derivations, nothing. You may want to try Spring 2006 resources on the Wikipedia:WikiProject_Resource_Exchange/Resource_Request. Marketdiamond (talk) 00:45, 11 October 2012 (UTC)[reply]
According to this page, the two "Nay" votes were Jeff Perry of Barnstable and Daniel K. Webster of Plymouth. ☯.ZenSwashbuckler.☠ 16:50, 15 October 2012 (UTC)[reply]

Question about copyright notice wordings by publishers

Hope this won't be considered legal advice, as I'm merely asking for observations. I'm also not sure if this should belong here or in the language reference desk since this is a question about wording. However, I've noticed that there are several websites which provide web content which state that "reproduction of this material in any form is prohibited". This is normal. What is not normal is that it seems that they seemed to have intentionally left out either the words "unauthorized" or "without permission". Does this literally mean that they do not allow any copying of their material with or without permission, or do they assume that the readers already know that they can copy if they first seek permission? If it is the former, what is the rationale? Are there really people or media companies that do not want their published works to be copied for any use at all with or without permission even if it would be covered by fair use or fair dealing? Narutolovehinata5 tccsdnew 00:02, 11 October 2012 (UTC)[reply]

This is skirting the legal advice prohibition, but talking very generally when in doubt contact the source of the material directly. Each law firm has their own specially tailored way of writing notices for these clients and some states may have unique areas of law either directly about copyright or in areas that could be implicated by copyright laws or vice versa. Also a lot of this is because of court decisions and precedence, because at the end of the day copyright law is whatever the judge and especially the appeals judge says it is (or the lower courts instructions to the jury). Since there are 11 different precedence in U.S. Federal Courts (some even directly contradict each other) you may find copyright notices in say California very different then those in Boston because of the differences between not only CA and MA laws but the 9th and 1st U.S. District Court precedence. Marketdiamond (talk) 00:12, 11 October 2012 (UTC)[reply]
My reading of it is that such language is meant to be as strong as possible. It's stronger than the actual law allows, frankly — it doesn't say, "except as allowed by fair use," for example. But there's no disadvantage to them wording it as strongly as possible; they are not fined for it or anything like that. --Mr.98 (talk) 01:17, 11 October 2012 (UTC)[reply]
And if they ever "change their mind" in a particular situation, making an absolute statement like that does not prevent them from then contracting with another person to allow them to reproduce the material. --PalaceGuard008 (Talk) 09:39, 11 October 2012 (UTC)[reply]
One amusing/disturbing example of Mr.98's point that many of these notices claim prohibitions not supported by the actual law is the saga of Wendy Seltzer vs. the National Football League: [8], [9], [10] ☯.ZenSwashbuckler.☠ 17:03, 15 October 2012 (UTC)[reply]

proportional representation

Is there a website or a document of how open-list party-list proportional representation, closed-list party-list proportional representation works and mixed-member proportional representation works? — Preceding unsigned comment added by 70.53.230.254 (talk) 00:56, 11 October 2012 (UTC)[reply]

Wikipedia. See Category:Proportional representation electoral systems for some articles at Wikipedia that covers this topic. --Jayron32 01:08, 11 October 2012 (UTC)[reply]

looking for japanese communities in thailand, northern thailand and southern china and throughout china

im looking for significant numbers (1000 or more?) of japanese people in thailand and china-- besides bangkok--but also info on that area -- — Preceding unsigned comment added by Steve1mm (talkcontribs) 01:29, 11 October 2012 (UTC)[reply]

Hi (please remember to always sign your posts with four ~, thanks!) just browsing the old google machine brings up the wikipedia articles Japanese people in China and Japanese migration to Thailand and they seem pretty extensive articles, hope that helps! Marketdiamond (talk) 04:11, 11 October 2012 (UTC)[reply]

October 11

RMS Titanic

Hi!, how are you?, I'd like to know how many Canadians were on board the Titanic. Thank you and have a nice morning! Mark. Iowafromiowa (talk) 11:03, 11 October 2012 (UTC)[reply]

Our own Passengers of the RMS Titanic has all of them, but doesn't separate them by nationality (although it is possible to sort them and count them). The Encyclopedia Titanica however does list them - there were only 5 who were born in Canada, but 35 who lived there, and 81 who were travelling there. I suppose these numbers overlap though. You can use that site to look for other nationalities too. Adam Bishop (talk) 11:08, 11 October 2012 (UTC)[reply]

Thank you! Iowafromiowa (talk) 11:10, 11 October 2012 (UTC)[reply]

You can also press the CNTRL and F keys and put in "Canada" that way you can count off the key presses to retain accuracy (instead of trying to remember your placement on the screen). Marketdiamond (talk) 11:36, 11 October 2012 (UTC)[reply]

Calculate risk of variable-rate interest loan/mortgage

How do you calculate the risk of variable interests? How do you know the value of an asset (like a house) which has a liability like a mortgage of 25 years with variable-rate mortgage on it? For the bank it's clear that they can charge the basic index + a margin and cover their back. As a borrower, you know that the interest rate can be as high as in the past, but that the past doesn't have to determine the future (thus, the rate can also be much higher than in the past). So, how much reserves should someone plan to cover any contingency? OsmanRF34 (talk) 11:31, 11 October 2012 (UTC)[reply]

You need to make some assumptions about the probability distribution of the future interest rates. Coming up with good assumptions for that kind of thing is very difficult and there have been hundreds of books and papers written on the subject. For most purposes, people just value loans as the current amount outstanding, which essentially means you are using the interest rate on the loan as your discount rate. I don't think many people intentionally hold reserves again their mortage - it would be more efficient to just pay off part of the mortgage. What people actually do is make an estimate of how high their mortage payments could go and make sure their income would be able to cover them (so you don't borrow so much that an increase in the rate would mean you no longer had enough money each month). Those estimates aren't easy to make for the long-term, though. --Tango (talk) 12:09, 11 October 2012 (UTC)[reply]
Holding reserves can be a good idea, if the mortgage has low interests (which is the case at the moment) but you are investing the deposit in a higher rate fund or higher paying bond, although this extra margin implies some additional risks. OsmanRF34 (talk) 12:16, 11 October 2012 (UTC)[reply]
When interest rates are low, isn't that the ideal time to get a fixed interest rate loan ? StuRat (talk) 15:26, 11 October 2012 (UTC)[reply]
Since the decision fixed or variable depends on the expectation up or down I suppose that you are right. The interest cannot get lower, so go for the fixed interest. But in a real-life scenario that's not always the case. Some banks won't offer fixed interest, some people are already into the mortgage and some people still want to know how do we evaluate variable interest loans. OsmanRF34 (talk) 15:51, 11 October 2012 (UTC)[reply]
It will be rare to be able to find something giving low-risk investment returns higher than the mortage interest rate (unless it's a fixed rate mortgage and rates have gone up since you took it out, but obviously that isn't relevant to your question). All returns and interest rates are essentially the risk-free rate plus a premium. The premium on your mortage is going to include an allowance for the risk of you defaulting and a profit margin. To find an investment product with a higher premium, you would need to take on significant risk. --Tango (talk) 17:03, 11 October 2012 (UTC)[reply]
But if someone gets extra unexpected cash, like a bonus, he can decide: will I cancel a part of the mortgage or will I take the risk of some bond (corporate or governmental) and cancel the mortgage next year with the proceedings? But that's only possible if he had some rule of thumb to evaluate the risk of the variable-rate mortgage. For the gov and corporate bond you already have several ratings of credit worthiness.
But maybe the answer is as simple as 'it's just speculation. (a gamble).'OsmanRF34 (talk) 17:20, 11 October 2012 (UTC)[reply]
As I said at the start, you have to come up with some model of interest rates, which is a problem far too difficult to answer here. As I said, there are many books written on the subject. You will generally have to take on very high risk to get an investment return higher than mortgage rates, though, so I doubt it is really worth worrying about, especially with a time horizon as short as one year. Just pay off the mortgage. (I am, of course, talking in general terms. If you are actually trying to make a decision about your own finances, you should speak to an independant financial advisor - they will be able to take your personal circumstances into account, which I cannot do.) --Tango (talk) 11:13, 12 October 2012 (UTC)[reply]
You don't need to come up with a model, you could look at what others with models think. Market expectations of future interest rates can be gauged by looking at, for example, forward interest rate and interest rate swap.90.212.157.32 (talk) 15:57, 13 October 2012 (UTC)[reply]

Party registration in the United States

I was trying to find out information about the system of party registration (i.e., where a voter rather than an elected official is described as a "Registered Democrat/Republican"). In the UK the concept simply doesn't exist. You might choose to join a political party, paying dues and possibly having a say on internal matters, but that's a private matter between you and the party just like joining any other private club. There is no concept of "registering" as a supporter of a particular party - or even odder, registering as an Independent - in the way that there is in the US.

Unless I'm missing something, information on Wikipedia seems to be scarce. I found one paragraph here, which at least told me how this happens (at the point when you register to vote), and that it's not the same for every state. But I can't find much more about it. I'd just be intrigued to know stuff like how this system first came about; what the perceived benefit is; why it's used in some states but not others; is it unique to the US or do other countries work the same way; and so on. --OpenToppedBus - Talk to the driver 12:32, 11 October 2012 (UTC)[reply]

Well to answer for the U.S. it seems that this will answer how and where it came about and some additional musings from the academic here and here. The simple answer is that in many jurisdictions and states you can't vote in the Spring primaries without being registered with a party affiliation, big whoop you might say but for the 2012 Republican presidential field it was a big deal and in 2008 you had the primaries almost go into the summer with McCain v. Romney and Obama v. Clinton . . . not to mention some jurisdictions that are basically one party dominate so that the general election isn't really relevant but the party primary in Spring is the election (again in most you may not primary vote unless you are registered with that party). As far as registering Independent there are on occasion Independent candidates and even in the mega cities some sitting mayors have broken away from their party for one election cycle to run for the general election in the fall after losing a party primary or knowing they would lose it like here. Also some states allow Independents to vote in either party primary. Marketdiamond (talk) 12:50, 11 October 2012 (UTC)[reply]
it's not an all US thing, some states have this voter registration system and others don't. OsmanRF34 (talk) 13:06, 11 October 2012 (UTC)[reply]
The OP said that in their final sentence. AlexTiefling (talk) 13:14, 11 October 2012 (UTC)[reply]
(edit conflict) To add to some of that: Very broadly speaking, the point of registering in the U.S. is that it grants the registree (is that a word? I'm going with it) the right to vote in primary elections, and thus have a say in which candidates appear on the ballot in the general election. The primary elections are basically the time when the parties choose their candidates. When you get a ballot in a primary, there are multiple ballots. Each party has a ballot where you choose from among that party's candidates for various offices. When you vote, you're choosing which candidates from that party will run in the general election later in the year. The specific practice varies greatly from state-to-state. Some states have a closed primary, which means you MUST be pre-registered as a member of that party in order to vote on that party's slate of candidates. You can still vote on non-partisan races (many Judgeships and other offices are officially non-partisan) and ballot initiatives/referenda but you don't get to vote on the party lists if you are officially "unaffiliated". Some states have open primaries, which means that registration is entirely meaningless: you still only get to vote in one party's primary or the other, but you simply pick which party when you show up at the polling station, and they give you a ballot. Other states have a hybrid of the two: basically, if you are registered as "unaffiliated" you get to choose which ballot you want, if you are registered for a party, you get that party's ballot. And, there are states which don't use primary elections but rather operate on what is called the caucus model (they all used to do this a century or two ago, but I think there's only 3 or so that do it now), Iowa_caucuses#Process covers how the caucus usually runs. So you can see, it is a complex system. Ah, the joys of federalism. --Jayron32 13:14, 11 October 2012 (UTC)[reply]
OK, thanks folks. Some really interesting stuff there, especially the links that Marketdiamond posted to the history of how it all first came about in Crawford County, Pennsylvania (and it really ought to get a mention in that article). It looks as if the concept of a Primary election, open or closed, isn't unique to the US (though it may have started there. But possibly, the concept of party registration, as opposed to party membership, is. I certainly can't find any evidence of that happening anywhere else. --OpenToppedBus - Talk to the driver 15:17, 11 October 2012 (UTC)[reply]
No, in the U.S., party registration is a nearly perfect synonym of party membership. People generally need to be registered members of a party in order to get that party's sponsorship to run for office to represent party, or to participate in party conventions, or anything else like that. You don't have to pay dues to join a political party, all you have to do is register for it, but I think (and I am just speculating here) that if a party collected dues from its members it would amount to a Poll tax (United States) on voting in primaries (poll tax means a fee charged to vote in the U.S. It has a different meaning elsewhere). See Twenty-fourth Amendment to the United States Constitution, which expressly forbids the charging of fees for ballot access. If it cost money to join a party, then that would mean that the parties could control ballot access unfairly in primary elections. The parties already do control ballot access in the primaries, but how they do so is carefully designed to be equitable under the Constitution. Using a fee to control that access would be (IMHO) a clear violation of the 24th amendment. --Jayron32 15:44, 11 October 2012 (UTC)[reply]

Like the OP, I also am not quite understanding this concept. After reading this thread, I still don't quite understand what the reasoning is for the US Government, or local governmen,t being part of handling the membership rolls of the parties. In most other situations, the US takes separation of state and private organisations much further than most of Europe, but in this case it seems to be the opposite. Why aren't parties seen as any independent organizations that can handle their internal affairs as they like? (Such as the Boy Scouts of America, who are apparently allowed to exclude gay people just because they want to.) Since you are mentioning the 24th amendment, Jayron, are you implying that the dominance of the two current main parties means that you can't get on the ballot without them? Because if I am not mistaken, there are plenty of people running for offices without being part of any of them. Doesn't that mean anybody can get on any ballog? How could e.g. the Republican party (who aren't a government of any level, or an agency thereof) be violating a rule against a "tax"? Aren't they allowed to (like the BSA) accept or reject anybody they like, and charge whatever fees they like? /Coffeeshivers (talk) 19:52, 11 October 2012 (UTC)[reply]

  • "are you implying that the dominance of the two current main parties means that you can't get on the ballot without them?" I won't just imply it. I'll state it right out: Unless you belong to one of the two main parties, you stand almost no chance of getting on the ballot, and significantly less chance of being elected even if you do. The proof is in the pudding: In the United States, there have been three or four significant "third parties" that have made any national impact: the Constitution, Reform, Libertarian and Green parties. None of them has "automatic access" to run candidates in any state. And in all of history, do you know how many candidates from these three parties collectively have been elected to National Office (President, Senator, or House of Rep.)? Zero. I've never seen an election anywhere for any national office where someone from any of these parties polled in the double digits, and commonly, even where they are on the ballot, they get less than 1% of the vote. There are "independent" candidates that occasionally win elections, but in the past 30 years the only one who has done so is Bernie Sanders from Vermont, and that's only because it's Vermont. Sanders also has the broad endorsement of the National Democratic Party, and caucuses with them in the Senate. Other so-called "independent" candidates have all been ones who were originally elected with support of one of the two major parties, but "left the party" at some point after gaining national recognition (see Joe Lieberman). Unless you belong to either one of the two major parties, you couldn't be elected "Dog Catcher" anywhere in the U.S. Third_party_(United_States)#Barriers_to_third_party_success has some background on the myriad ways in which the two major parties maintain a stranglehold on elections in the U.S. But what it comes down to is that neither of the two major parties has anything to gain by making it easier for smaller parties to have access to getting candidates on the ballot, so they make it as hard as possible. --Jayron32 20:30, 11 October 2012 (UTC)[reply]
    More: List of third party performances in United States elections shows that, since 1990, "any elections where a candidate that wasn't a Republican or Democrat obtained at least 5.0% of the vote" and shows how completely rare it is for a third-party candidate to get even that small amount of votes. Since 1990, in any senatorial, gubernatorial, or presidential poll, someone from outside the two major parties has won their election 7 times:
    Lowell P. Weicker, Jr. was an ex-Republican who ran as an independent after gaining national fame as a Republican.
    Wally Hickel was the same.
    Angus King was a true indepedent, with no party affiliation
    Jesse Ventura was a member of the Reform party with no prior political experience
    Lincoln Chafee is an ex-Republican who later ran as an independent after gaining national exposure as a Republican
    Joe Lieberman is an ex-Democrat and one time Democratic Vice Presidential nominee.
    Bernie Sanders is a true Independent, with no prior party affiliation
    That's it: 5 governors and 2 senators and never since 1990 has a Presidential candidate from third party won a single Electoral College vote, meaning no such candidate has ever carried a state. And of those 5 governors and 2 senators, only 3 won their elections with no prior party affiliation, that is everyone else had to use the party system to build up enough exposure to strike out on their own. By my math, that's 302 gubernatorial elections, 300 senatorial elections, and 5 presidential elections over that time frame: so in 607 tries, a third party candidate with no prior party affiliation to work from has won an election 3 times, or 0.494% of the time. I can't think of another Western democracy where two parties have won 99.506% of every election of the past 18 years. --Jayron32 20:52, 11 October 2012 (UTC)[reply]
OK. Thanks for the info. However, while the two dominant parties de facto have a complete stronghold on high level politics, they don't have any de jure special status. Or do they? And if they don't, how can any membership fees or rules they would (hypothetically) make, be affected by a law against taxes? And how can the government at different levels be involved in their membership management? Maybe not expecting answers that I'll understand to that, a more concrete follow up question would be: Is Green Party, Libertarian Party etc. membership also done by voter registration? Or are those parties, dwarves in size compared to the main two, completely different? (And I don't mean in practice, that I know that they are, but from a legal perspective?) /Coffeeshivers (talk) 21:06, 11 October 2012 (UTC)[reply]
No, those parties do not run in the Primaries. They nominate their candidates at conventions/caucuses run by their parties in various state and national conventions/caucuses. So if you are a Libertarian, and want to help select the Libertarian Candidate for an office, you need to go to the local, state, or national Libertarian Convention where the candidate is chosen. The deal is that in the U.S., there is literally no national election. Even the President is technically elected by the Electoral College, the members of which are elected by the individual states on state-wide elections according to state laws, with no input from the national government. There are very few restrictions placed on how states choose to run their elections, so effectively the two parties needn't have any permission or legal allowances at the federal level to maintain their de jure and de facto stranglehold on national politics. The states are not subject to Federal laws except in a VERY limited number of cases, so machine politics can run rampant in the states and localities. The federal government just does not get involved in state election laws, so the Parties are allowed to enact election laws which not just set up de facto barriers to third parties, but which de jure deliberately exclude (or set such giant hurdles as to practically exclude) anyone except themselves. --Jayron32 21:13, 11 October 2012 (UTC)[reply]
I wouldn't want to see the case overstated. The Republican and Democratic Parties have no special legal status. If another party wants to have a primary ballot, I see no reason why they could not do it if they meet some minimal threshold of support as demonstrated by a petition. I'm no lawyer, but I find it implausible that any state could get away with treating another party according to different rules than those applying to the Democrats and Republicans (the federal courts would slap that down). The fact is that it's simply part of US culture that third parties are not generally all that popular.
The purpose of having the state government handle the list of party registrants is simply one of practicality: when you show up at the polling place on primary election day, the poll workers need to know from their records which ballot to give you (if only registered party members can vote in its primary) or which ballots you can choose from (if independents are allowed to vote in any party's primary).
Party registration is a concept that refers to registration with the polling authorities: when you register as a voter in most states, you state a party or none (="independent"). In some states this is meaningless, while in others it signifies what party's primary you can or cannot vote in, as explained above. But party membership is a vague concept in the US, seldom referred to. If someone says that he is a member of a particular party, maybe he means he is registered with the polling authorities that way, or maybe he means he's "a card-carrying member" of that party, probably implying he has contributed money to them and possibly received some sort of certificate in return. But "party membership" is just a vague and unimportant concept. (One exception: back in the 1950s during the paranoia about communism, Joe McCarthy used to haul people before Congress and ask them "Are you or have you ever been a card-carrying member of the Communist Party?" Duoduoduo (talk) 17:03, 12 October 2012 (UTC)[reply]
Interesting tidbit in this regard. From today's CNN obituary of politician Arlen Specter:
Even though he was a registered Democrat, Specter ran successfully for Philadelphia district attorney on the Republican ticket in 1965 and eventually registered as a Republican.
Duoduoduo (talk) 19:27, 14 October 2012 (UTC)[reply]

Can someone provide for me an example or illustrate for me what this quote means from this article secularization?

  • Complete Secularization: this definition is not limited to the partial definition, but exceeds it to "The separation between all (religion, moral, and human) values, and (not just the state) but also to (the human nature in its public and private sides), so that the holiness is removed from the world, and this world is transformed into a usable matter that can be employed for the sake of the strong".

What on earth does the author mean by that? 140.254.227.67 (talk) 14:22, 11 October 2012 (UTC)[reply]

I'm not entirely sure, but it sounds like the author of that quote, Abdel Wahab El-Messiri, is using terms that recall the philosophical tradition of nihilism, the idea that life has no inherent meaning or value. In his opinion, because "complete" secularization removes all external sources of value (religious, moral, etc.) from a person, they become vulnerable to being "usable matter that can be employed for the sake of the strong", meaning that they can easily be influenced by people who want to use them. I read him as saying "without some sort of externally-given values, people lack any means by which to resist becoming pawns of the powerful". I don't agree or disagree with what he is saying, but that seems to be what he is saying. --Jayron32 14:31, 11 October 2012 (UTC)[reply]
OK. I'll take it as a negative/pejorative connotation of secularization then. 140.254.227.67 (talk) 14:39, 11 October 2012 (UTC)[reply]
I think the brackets in that sentence don't make sense. Parenthetical material should be capable of being removed, but if you remove the contents of the sets of parentheses, there's nothing left of the second part of the 'separation'. I also noticed that the sentence prefacing this quotation in the article body refers to "the secularization term" instead of to "the term 'secularization'". I wonder if something has been lost in translation?
Nevertheless, the concept of complete secularization that El-Messiri is articulating here seems to be one of strict naturalism or materialism - a world-view in which not just religious concepts such as 'faith', 'god', 'dharma', 'tawhid', 'avatara' are meaningless, but also ethical concepts such as 'proper', 'good', 'lawful', 'cruel'. Whether anyone actually holds such a view is questionable, but it is clear he wishes to distinguish the simple separation of religion and nation from the abandonment of the full range of religious and philosophical inquiry and action. AlexTiefling (talk) 14:54, 11 October 2012 (UTC)[reply]
"He's a Nihilist" . . . "aw that must be exhausting!" Marketdiamond (talk) 15:11, 11 October 2012 (UTC)[reply]
So, that would imply that religious and philosophical inquiry and action have some purpose, do they? Religious and philosophical traditions do have value, do they? 140.254.227.67 (talk) 15:22, 11 October 2012 (UTC)[reply]
Yes, that's what he's saying. What he's doing in that quote appears to be a reductio ad absurdum argument. He's framing secularism as to its inevitable end: if people lack any sort of ethical values at all, then they have no means by which to judge for themselves which people to give their allegiance to, and which to not. Thus they are reduced from independent actors of their own to tools "be employed for the sake of the strong". It is a careful warning to not take secularism too strongly, because the sorts of values that religion and things like religion like philosophy and morality and ethicism gives us is itself important for us. He's not necessarily advocating for a religious world view, but he is also advocating against the sort of extreme nihilism that he equates "complete secularism" to. --Jayron32 15:33, 11 October 2012 (UTC)[reply]
That reminds me of continental philosophy. I once knew a continental philosopher/English teacher who would agree with that attitude, and I am pretty much influenced with that sort of thinking. She claimed she was a humanist, or specifically a Christian humanist, but it appears that she frames the pitfalls of secularism in its implications in King Lear or the Enlightenment. 140.254.227.67 (talk) 15:56, 11 October 2012 (UTC)[reply]
This strikes me as not unlike Heidegger's The Question Concerning Technology. Basically Heidegger argues that technology is not a thing but a way of mind, a way of rendering everything into utility and purpose, including (ultimately) man himself, who is in the end just seen as another form of reserve resources for exploitation by the technological worldview. I've never thought of it as a critique on "complete secularism" but there's definitely that to it. --Mr.98 (talk) 16:37, 11 October 2012 (UTC)[reply]
I don't think the quoted text makes any sense because it groups "moral values" and "human values" with "religion" when "moral values" and "human values" bear just as much affinity to the secular sphere as to the religious sphere. Bus stop (talk) 16:52, 11 October 2012 (UTC)[reply]
Well, it depends on what you mean by "secular". That's the distinction he's making, between the common definition of "secular" which just means "not religious" and a new term he's coining called "Complete secularism" which takes it an extreme end. --Jayron32 17:16, 11 October 2012 (UTC)[reply]
Right; the guy is arguing that secularism means the separation of all values from the activities of the state, which is not most people's definition of secularism. --Mr.98 (talk) 02:23, 12 October 2012 (UTC)[reply]

The creation of the Hebrew calendar

When was the Hebrew calendar created? I know it was a gradual process, but when did the main events in its creation occur? Is it true that it's the oldest calender known? Thanks, Oh, well (talk) 17:14, 11 October 2012 (UTC)[reply]

The history seems to be covered in some detail in the Change to a calculated calendar section of the article that you linked. Is there something in particular you wanted to know that's not covered there? Regards, TransporterMan (TALK) 17:29, 11 October 2012 (UTC)[reply]
Yes – I want to compare its creation date with other calenders to see it it's really the oldest, but the article focuses on changes to different versions of the Hebrew calender rather than prospects to the title "the first version of the Hebrew calender". I wish it had explained in more details its originations, especially with comparisons to other calenders that may have influenced it (similarly to how the Julian calendar influenced the Gregorian one). Oh, well (talk) 19:05, 11 October 2012 (UTC)[reply]
You've just about exhausted my ability to give any substantial help, but I would note that in looking into this it occurs to me that you really cannot answer the question you're answering without determining a set of criteria by which to determine what "creation" means, so as to distinguish between minor changes to a calendar scheme, which leave it being the same calendar, and more major changes which would cause it to be the creation of a new calendar. If you set those criteria as being the first time humans began to group days into larger and smaller cycles, then calendars which evolved in the same region are all likely to be of the same age. If on the other hand you set those criteria by choosing some particular subset of:
  • number of days,
  • number of months,
  • existence and number of days in the week,
  • number of months,
  • names of days,
  • names of months,
  • whether months have fixed numbers of days or correspond to the lunar month,
  • whether the calendar is a solar annual calendar always beginning on the same solar day or a lunar calendar which is or is not occasionally corrected to match the solar month,
  • how and if years are numbered,
  • and I could probably think of additional criteria if I tried,
then the particular criteria that you choose and the importance that you assign to them are going to determine whether a particular change in a calendar creates a new calendar or merely a new version of the same calendar. If you're playing a game of "mine's the oldest" (and I'm not saying that you are, but if you were) then that game is inevitably destined to devolve into a argument over whose criteria are the right ones rather than a real determination of which calendar is older and the oldest, like beauty, will largely be in the eye of the beholder. Regards, TransporterMan (TALK) 20:29, 11 October 2012 (UTC)[reply]

Yes, "creation" is a bit vague. For example, during certain periods of ancient history, Jews in Israel would assign special watchers who at the end of every month would observe a full moon in the sky and light a bonfire on a hill. This bonfire would be seen by people on other hills, who would light a bonfire as well. The bonfire would spread. This was the form of communication - the way to say "it's a new month." So that essentially was a way to keep track of time and when a new month was, even without setting it in stone. Indeed, the same is true for holidays during the year, as we know that they had (have) set holidays on specific dates to be observed then. There were many calculations involved, and it's likely there was some form of a written calendar, even if not like the final one today. Certainly, the version of their calendar used (and still used) is a lunar calendar, not a solar calendar. --Jethro B 23:47, 11 October 2012 (UTC)[reply]

The full moon does not occur at the start, but halfway through a Jewish month. You probably mean the observation of a new moon. -09:07, 12 October 2012 (UTC)
"Sound the trumpet at the new moon." 86.159.77.170 (talk) 10:30, 12 October 2012 (UTC)[reply]

The creation of the Hebrew calendar is traditionally ascribed by Jews to the moment that this was said. If you want to know when that was, you'll need to take a deep breath and dive into the cesspit of Exodus dating, because it took place about a fortnight before the Exodus. Modern scholarly consensus seems to place it at earlier than 1440 BCE. --Dweller (talk) 08:35, 12 October 2012 (UTC)[reply]

Why isn't he categorized as "Necrophiles" and "Canadian cannibals"?. I can't edit the article because I'm not yet autoconfirmed, but for those who saw the video, it's clear that he practises both things. Thank you. Iowafromiowa (talk) 18:10, 11 October 2012 (UTC)[reply]

You should be able to ask that question at the article's talk page. Be careful not to call him names or assert that he is indeed guilty of certain crimes, see WP:BLP for policy. μηδείς (talk) 18:50, 11 October 2012 (UTC)[reply]
Yes. Until he's convicted of something, such labels are BLP violations and are subject to immediate removal. ←Baseball Bugs What's up, Doc? carrots22:52, 11 October 2012 (UTC)[reply]

Luxury retailers profit protection

I was in a Chanel boutique the other day with my sister and we were browsing the suits, handbags, gowns, jewellery etc. While there I couldn't help but wonder how they protected their merchandise, it seemed very exposed. There was one of each handbag displayed impeccably, with seemingly nothing to stop you from just walking out with it; no security tag or anything. Fair enough, there was a security guard but if the store was ambushed by several thieves, doesn't look as though it'd be particularly effective. --Jewboy 20:15, 11 October 2012 (UTC)[reply]

The items usually have a significant markup (>3x) so the brand is worth more than the physical bag. Additionally, such venues usually don't attract theft.Smallman12q (talk) 23:21, 11 October 2012 (UTC)[reply]
Probably not a professional thief who wants to quickly turn around stolen items for maximum profit at minimal risk. A kleptomaniac, on the other hand, might steal it. StuRat (talk) 04:03, 12 October 2012 (UTC)[reply]
There will have been security cameras as well, so the police can identify the thief and get the product back. And are you sure there were no tags? They may well be hidden inside. --Tango (talk) 11:25, 12 October 2012 (UTC)[reply]

Danish throne in 1863

Why didn't Prince Frederick of Schleswig-Holstein-Sonderburg-Augustenburg contend for the Danish throne in 1863? He was the senior male in the Oldenburg line and was therefore the first in line heir to the Duchies of Schleswig and Holstein. He was also an heir according to agnatic primogeniture to King Frederick III of Denmark, the first absolute monarch in Denmark, that is after the childless daughters of Frederick VI, Princess Caroline of Denmark, Princess Vilhelmine Marie of Denmark and his elderly father (agnatic primogeniture was the alternative option to the Salic law after the end of Frederick III's line in 1863). Also most of his ancestors were Danish, which in an age of nationalism should count for something. --The Emperor's New Spy (talk) 20:31, 11 October 2012 (UTC)[reply]

His father's cession of claims associated with the London Protocol of 1852 seem to have taken him and his descendants out of the running for the Danish throne.--Cam (talk) 23:40, 11 October 2012 (UTC)[reply]
But that renunciation doesn't apply to Frederick, or so he claims by the time of the Second Schleswig War in 1864.--The Emperor's New Spy (talk) 20:02, 12 October 2012 (UTC)[reply]

Why is marginalism such a big improvement over subjectivism in economics?

It is my understanding that the subjective theory of value, has a long history before Adam Smith that can be traced from Aristotle to certain Scholastics and then Turgot and the physiocrats. So the idea that subjective utility, psychological preferences and supply and demand determines prices was advocated long before the Marginal Revolution of Jevons, Walras, Menger.

So my question is what was the big deal about marginal utility? From the wikipedia article it seems that the main thing is that you look at the individual units at the margins instead of the product as a whole. Why is this so much more useful than just saying subjective utility determines prices? Whats the big deal about the "marginal unit"? And why did this change economics so much?

Well, I notice economics becomes much more mathematical after Jevons, so is the main thing that it becomes easier to apply concrete mathematical calculus and quantification to economics?

Can someone explain why subjective marginal utility of the late 1800s was so revolutionary and transformative to the field of economics, compared to old fashioned subjectivism? --Gary123 (talk) 02:38, 12 October 2012 (UTC)[reply]

Marginal utility doesn't discuss the problem of value and subjective preference for utility, but discusses the problem of price through the price of the final unit realisable in an effective demand. It had immediate instrumental benefits for attempts to price commodities in finance capital and was widely adopted. Fifelfoo (talk) 03:53, 12 October 2012 (UTC)[reply]
As Joseph Schumpeter noted in his History of Economic Analysis:

It is not too much to say that analytic economics took a century to get where it could have got in twenty years after the publication of Turgot's treatise had its content been properly understood and absorbed by an alert profession.

The greater influence of the marginal revolution has to do with the paradox of value. The basic gist of the argument is as follows: Since you cannot live long without water, but could go a lifetime without diamonds, surely the subjective value of water should be much greater than the subjective value of diamonds in the mind of any reasonable person. But in the typical marketplace, a bucket of diamonds will carry a greater price tag than a bucket of water. This empirical fact is difficult to reconcile with a naive formulation of the subjective theory of value. So this is where the "subjective marginal utility" enters the explanation. The subjective theory of value is more readily accepted when we frame the above situation in terms of "the subjective value of one additional bucket of water" versus "the subjective value of one additional bucket of diamonds", rather than simply "the subjective value of a bucket of water". Since the typical marketplace actor will have an adequate water supply, but perhaps not a great deal of diamonds, the subjective value of more diamonds could well be greater to him than that of more water, thus explaining why diamonds are more expensive than water. Gabbe (talk) 13:57, 12 October 2012 (UTC)[reply]

American railroads in WWII

Besides Union Pacific, what other railroads were involved in the war effort during WWII?142.255.103.121 (talk) 03:54, 12 October 2012 (UTC)[reply]

Can you clarify your Q please ? I doubt if there was any American railroad which didn't contribute to the war effort by delivering troops, ammunition, weapons, vehicles, and/or supplies. StuRat (talk) 04:00, 12 October 2012 (UTC)[reply]
One of the largest corporations on earth up until the early 1970's was the Pennsylvania Railroad and they were the dominant transport for the "arsenal of democracy" Pittsburgh and among the top in the other "arsenal of democracy" Detroit. But StuRat is correct everything from the Santa Fe to the Atlantic Coast Line in Florida played major roles in the war efforts. Marketdiamond (talk) 06:22, 12 October 2012 (UTC)[reply]

Alright. Which other railroads, besides Union Pacific and Pennsylvania Railroad, were involved the the war effort?142.255.103.121 (talk) 17:20, 12 October 2012 (UTC)[reply]

Start with List of common carrier freight railroads in the United States and see which ones existed at the time. I picked Duluth, Missabe and Iron Range Railway at random; it states "Ore movement was nearly 45 million tons in 1942 and the War Production Board allowed the Missabe to order ten more Yellowstones, delivered in 1943." Clarityfiend (talk) 22:13, 12 October 2012 (UTC)[reply]
Every single solitary one in existence at the time. --NellieBlyMobile (talk) 21:18, 13 October 2012 (UTC)[reply]

October 12

RMS Titanic and American Civil War veterans question

I've seen that there were several Titanic's passengers who were in a way or another, involved in the American Civil War. Can anybody tell me if they were all Confederates or Unionists? Thank you. For instance, Isidor Straus was Confederate. And how old did they have to be to lie within the range of born before or during the Civil War? Thank you. Iowafromiowa (talk) 10:46, 12 October 2012 (UTC)[reply]

Well, the Titanic sank in 1912, which is 47 years after the end of the US Civil War (1861-1865). If you figure they had to be, say, 15-65 to fight in it, that means they would have to be 62-112 on the Titanic. People born before the Civil War would be approximately 51 or older, and those born during the Civil War would be approximately 47-51 (I say approximately because those events didn't all occur on the same day of the year, but they were all in April or May, so it's close enough for our purposes). StuRat (talk) 10:52, 12 October 2012 (UTC)[reply]

I understand it now. Thank you StuRat. Isidor Straus was 14 by 1859, so he fought in the Civil War for the Confederates. Francis Davis Millet was also involved in the American Civil War by helping his father and taking part himself in the war. I was surprised to find so many US Civil War veterans on board the Titanic. I didn't know there were so many on board. Thank you again. Iowafromiowa (talk) 11:01, 12 October 2012 (UTC)[reply]

It would have been quite unlikely if there weren't both Union and Confederate veterans, due to the large number of Americans in that age group on board. May I mark this Q resolved ? StuRat (talk) 11:03, 12 October 2012 (UTC)[reply]

You mean I shouldn't have been surprised to find American Civil War veterans on board? hahaha. Thank you. Iowafromiowa (talk) 11:05, 12 October 2012 (UTC)[reply]

Consider the case of John Clem, the "Drummer Boy of Chickamauga" who was promoted to Sergeant in 1863 at the age of 12. He would have been 60 when the Titanic sank. "Over 100,000 boys younger than 15 enlisted in the Union Army, there were even 300 boys younger than 13."[11] Alansplodge (talk) 17:29, 12 October 2012 (UTC)[reply]

big face rock

I want to know the name of a location (a city) that has many big rocks with the face shapes. I think it is somewhere in America continent but not sure. Thanks!Pendragon5 (talk) 12:43, 12 October 2012 (UTC)[reply]

Do you mean man-made faces like Mount Rushmore or natural formations that some claim look like faces like the (now deceased) Old Man of the Mountains? --Jayron32 12:56, 12 October 2012 (UTC)[reply]
Rushmore is the most famous/visited but there are many in America and the world, you may want to check this out (all world locations) Category:Mountain monuments and memorials. Marketdiamond (talk) 14:11, 12 October 2012 (UTC)[reply]
It seems that of all the memorials on that list only the Crazy Horse Memorial features a huge carved face like Mount Rushmore. The rest are either full statues or various kinds of structures. --Saddhiyama (talk) 15:05, 12 October 2012 (UTC)[reply]
Are you sure it's on the American continent? The moai of Eastern Island would fit your description. 2001:18E8:2:1020:749C:5B76:1D8E:3D22 (talk) 19:22, 12 October 2012 (UTC)[reply]
Moai, that's it! That's what I'm looking for. Thanks everyone for suggestions!Pendragon5 (talk) 22:20, 12 October 2012 (UTC)[reply]
Those are the carved statues. The location you asked for is Easter Island (not Eastern Island). -- Jack of Oz [Talk] 22:59, 12 October 2012 (UTC)[reply]
Re:Saddhiyama actually found that category by going to Stone Mountain so that's also on the list. Marketdiamond (talk) 16:54, 13 October 2012 (UTC)[reply]

History:The recall of Colonel Amherst and smallpox

Hi, I'm wondering whether the reason Amherst was removed from command was because the Brits found out about his plan for killing Native Americans using smallpox.--I'd like to think that at least some of the higher ups had scruples, and "fired" Amherst for that reason.Thanks198.189.194.129 (talk) 17:35, 12 October 2012 (UTC)[reply]

Not sure if it was considered here, but even if one has no concern about the genocide of Native Americans, spreading smallpox among them may very well lead to the disease jumping from them to people they did care about. StuRat (talk) 18:21, 12 October 2012 (UTC)[reply]
Our article on Amherst says that he was recalled because his policies provoked the war that the smallpox gambit was intended to stop. If part of the criticism levied against him was his attempt at genocide, the honors that he subsequently received very shortly after his return to England and continued to receive thereafter would suggest that the objection certainly wasn't very strong. Regards, TransporterMan (TALK) 18:29, 12 October 2012 (UTC)[reply]

Not sure whether it's really known that Amherst in fact distributed smallpox-tainted blankets -- in several letters he said it would be a good thing to do, but didn't say that he had done it. AnonMoos (talk) 22:30, 12 October 2012 (UTC)[reply]

Soldiers in Amherst's army did try the smallpox-blanket tactic at Fort Pitt, probably without success, but the surviving documents indicate that they thought of it and tried it before Amherst even mentioned it. This is such a striking coincidence that it might mean the idea had been bandied about before then. I've never seen any evidence that Amherst's superiors learned about the idea. Some would have no doubt been horrified, though in the next war, the American colonists were sure the Bristish were deliberately trying to infect them, and in 1777, a British officer once again suggesting doing just that. See http://www.history.org/foundation/journal/spring04/warfare.cfm. —Kevin Myers 01:33, 13 October 2012 (UTC)[reply]

Kevin Myers is correct for the location, and just for clarity that is Fort Pitt (Pennsylvania) since naming things after Prime Minister (and State Secretary) Pitt was actually quite popular in Pre-Revolutionary North America and quite a number of forts and towns were named Pitt or derivations.

  • June 24, 1763: Fort Pitt: Captain Simeon Ecuyer gifts the Delawares two blankets and a handkerchief from the small pox ward "out of regard to them" after the tribe pledges to renew their friendship. While the exact meaning of his phrase is unclear, a later invoice appears to establish the purpose was transmittal of small pox, the world's first use of bio weapons.
  • July 13, 1763: Col. Bouquet writes back to Amherst: "I will try to inoculate the bastards with some blankets that may fall into their hands, and take care not to get the disease myself."
  • July 16, 1763: Amherst replys to Bouquet: "You will do well to inoculate the Indians by means of blankets, as well as every other method that can serve to extirpate this execrable race."
  • Not my words and also remember that the Siege of Fort Pitt was at its height with nothing less than the American continent at stake. Marketdiamond (talk) 16:49, 13 October 2012 (UTC)[reply]

"remember that the Siege of Fort Pitt was at its height with nothing less than the American continent at stake" What do you mean? Do you mean that the Siege of Fort Pitt was a battle the Native Americans had to win? Best wishes, Rich Peterson216.86.177.36 (talk) 00:28, 14 October 2012 (UTC)[reply]

Not the first use - I think I remember reading the Mongols catapulted bubonic plague corpses before the great European outbreak. (The article says that also but doesn't give a source) Wnt (talk) 20:03, 13 October 2012 (UTC)[reply]

EU Nobel Peace Prize

What did the EU do that they got the Nobel Peace Prize this year? --Jethro B 18:27, 12 October 2012 (UTC)[reply]

Basically, it came into being after World War 2 and there hasn't been a war in Europe since. Here's the award citation. Why this year in particular? Maybe the Nobel committee wanted to try and remind us Europeans what we have achieved before we tear ourselves apart ;) --TammyMoet (talk) 18:36, 12 October 2012 (UTC)[reply]
There hasn't been a war on the North American continent since WWII either - are Canada, America, and Mexico entitled to the Nobel Peace Prize as well?? The EU deals with other regions as well and gets involved in some conflicts, but a lot of these conflicts have not been solved and still remain. I don't have anything against the EU, I'm just trying to understand why they got the Nobel Peace Prize, as it's not so clear to me. --Jethro B 18:58, 12 October 2012 (UTC)[reply]
I think a better way of looking at it would be that Europe has been in a state of intermittent war between nation-states more or less continually for the past fifteen hundred years (or more). It's not simply that the EU hasn't had a war among its member states for sixty years, but that they haven't even come close - this system has apparently managed to break a cycle that lasted for most of recorded history, which is quite a feat. The North American context isn't quite the same...
It's also worth noting that "peace" isn't simply, in the Nobel context, a matter of not having wars. The award talks about the EU promoting "peace and reconciliation, democracy and human rights" - it's remarkable the effect that the carrot of EU membership has had on some states in ensuring the Copenhagen criteria are adhered to. Andrew Gray (talk) 22:56, 12 October 2012 (UTC)[reply]
And there was in fact a major genocide in Europe about two decades ago... --Jethro B 18:59, 12 October 2012 (UTC)[reply]
Not in the EU, however. Even today, from the successor states of Yugoslavia, only Slovenia has joined the EU, and Slovenia had peacefully seceded.--Stephan Schulz (talk) 20:09, 12 October 2012 (UTC)[reply]
That is not correct, Stephan. Slovenia'a secession was not "peaceful", see Ten-Day War. Only the Serbes could not afford to do to much to them, as they had to deal with the Croatian right away after the Slovenian secession (and Serbia has a border with Croatia, which made it more urgent). --Lgriot (talk) 13:54, 13 October 2012 (UTC)[reply]
Ooops. I did check before I wrote that, but apparently not carefully enough. I've struck it. Thanks! --Stephan Schulz (talk) 19:06, 13 October 2012 (UTC)[reply]
I agree with you, the Kosovo War is only an example of why the prize was wrongly awarded to the European Union. Iowafromiowa (talk) 19:02, 12 October 2012 (UTC)[reply]
It's all a matter of the politicization of the Nobel committee. Look at the 2009 Nobel Peace Prize. I am not sure what further references we can provide here, other than debate and opinion. μηδείς (talk) 19:09, 12 October 2012 (UTC)[reply]
(ec) I've noticed that the way to get a Nobel Peace Prize seems to be to kill lots of people, then kill fewer people. Thus, since the death toll relative to WW1 and WW2 has been considerably reduced, they are as worthy of the prize as Arafat was for reducing the number of civilians he had murdered, in his later years. Then there was the case of Obama getting one for "not starting the US invasion of Iraq". StuRat (talk) 19:11, 12 October 2012 (UTC)[reply]
"Political satire became obsolete when Henry Kissinger was awarded the Nobel peace prize." [12] AndyTheGrump (talk) 19:16, 12 October 2012 (UTC)[reply]
According to the Nobel Peace Prize page, "Unlike the scientific and literary Nobel Prizes, usually issued in retrospect often two or three decades after the awarded achievement, the Peace Prize has been awarded for more recent or immediate achievements, or with the intention of encouraging future achievements." Of course right after that is [citation needed]. Livewireo (talk) 19:23, 12 October 2012 (UTC)[reply]
Then why not award the Nobel Prize in Chemistry to chemists who've never done anything noteworthy, to encourage them to do so ? :-) StuRat (talk) 19:26, 12 October 2012 (UTC) [reply]
I don't know about you, but I wouldn't call 67 years of peace between the members a "recent or immediate" achievement. And in any case, I am French living in Britain, and I assure you that I very thankful to the EU that in all the 40 years of my life I never had to travel to a frontline for the defense of the homeland against an invading army, unlike my grandfather. --Lgriot (talk) 14:03, 13 October 2012 (UTC)[reply]
67 years of peace happened very recently. 66 years of peace was a little less recent, but apparently that wasn't enough! --Tango (talk) 19:42, 13 October 2012 (UTC)[reply]
The European Union is not 67 years old, so you cannot prize them for that. Furthermore: the EU is not the only European pan-national institution, the NATO seems more responsible for the peace in Europe, both among NATO members and between West/Central Europe and Eastern Block. OsmanRF34 (talk) 20:04, 13 October 2012 (UTC)[reply]
And not forgetting the Council of Europe (of which the UK was a founder member in 1949). The EU has even stolen their flag! Alansplodge (talk) 20:40, 14 October 2012 (UTC)[reply]
I get the impression the Nobel Peace Prize Committee is about as liberal as you can get, so aren't about to buy into the idea that a strong defense ensures peace. For them, unilateral disarmament is more likely to be awarded with a prize. StuRat (talk) 05:17, 15 October 2012 (UTC)[reply]
I'm late to this discussion, so if I don't get any replies here I may post a new question. What interests me is why the Nobel committee has awarded the Peace Prize to organizations 20 times when Nobel's will expressly stated that it should be given to "the person [my italics] who shall have done the most or the best work for fraternity between nations, for the abolition or reduction of standing armies and for the holding and promotion of peace congresses." --Viennese Waltz 08:42, 15 October 2012 (UTC)[reply]
His will also said that awards may not be made posthumously. No ifs or buts. But there have been half a dozen posthumous awards, the latest as recently as 2011. -- Jack of Oz [Talk] 10:52, 15 October 2012 (UTC)[reply]
Not doubting your word, Jack, but I can't see where in the will it says that. --Viennese Waltz 11:39, 15 October 2012 (UTC)[reply]
Seems I assumed too much. I read about these rules in "Posthumous nominations" and assumed they came from Nobel himself. That sort of makes me wonder what sorts of other conditions (or, indeed, freedoms) the Nobel Committee can impose. -- Jack of Oz [Talk] 19:23, 15 October 2012 (UTC)[reply]

American Civil War veterans' families benefits

I'm quite interested in the American Civil War because my grandfather served in the Union Army (yes, I'm old and my father was already old when he had me born) and I would like to know if there are further benefits for veterans' families now and if my grandfather can be taken to the Arlington Cemetery. Thank you. Iowafromiowa (talk) 18:54, 12 October 2012 (UTC)[reply]

My grandfather (1845 – 1940). My father (1895 – 1955). Iowafromiowa (talk) 19:45, 12 October 2012 (UTC)[reply]

I found this [13] at the Department of Veteran Affairs. There is a pdf you may want to read on survivor's benefits. The social security administration pays out benefits to surviving family, but it has a list of who qualifies, and grandchildren are not eligible. There is a contact link at the bottom of the page if you wish to contact the VA, I would definitely suggest trying that. Livewireo (talk) 20:18, 12 October 2012 (UTC)[reply]

Thank you for helping me, I will read that. Iowafromiowa (talk) 20:25, 12 October 2012 (UTC)[reply]

Living children of both Union and Confederate soldiers

This is my third question regarding the American Civil War, hope you understand since my grandfather fought for the Union Army. But I've come to know that there are living CHILDREN and WIDOWS of Civil War veterans. I would like to know how many children and widows of veterans from both sides are still alive today. Thank you indeed. Iowafromiowa (talk) 22:03, 12 October 2012 (UTC)[reply]

The last widow of a Civil War veteran died in either 2004 or 2008, depending on how you count it. this story and Alberta Martin for the 2004 death, and this story and Maudie Hopkins for the 2008 death. For living Children of civil war veterans here is the list for the Union, last updated yesterday. I can't find a similarly comprehensive list for confederate veterans, but this google search turns up several stories of living children of Confederate veterans. --Jayron32 23:35, 12 October 2012 (UTC)[reply]

Peerage: pregnant widow of a childless peer

Has it ever happened that a British peer died childless leaving a pregnant widow or leaving a daughter and a pregnant widow? Did the title descend to the next-in-line (the daughter or a brother of the peer) and then revert to the posthumous child (son)? Surtsicna (talk) 19:56, 12 October 2012 (UTC)[reply]

It happened to Edmund Tudor, 1st Earl of Richmond, who died several months before the birth of his son the future Henry VII of England. I'm not exactly sure what happened to Richmond in the meantime, but it doesn't seem to have passed to anyone else. Adam Bishop (talk) 21:00, 12 October 2012 (UTC)[reply]
Not a good example, as Edmund Tudor was a prisoner at the time of his death, and his peerage was forfeit. But our article on posthumous children lists several British peers. I believe (although I haven't a reference to hand) that peerages are kept pending the birth of any child en ventre sa mere; the succession would be backdated to the death of the previous holder if the child was female (if thus ineligible to inherit), or conferred immediately on the child (if eligible). AlexTiefling (talk) 21:16, 12 October 2012 (UTC)[reply]
Unfortunately, none of the posthumously born peers listed there inherited the title from their father. I assumed that succession is automatic and that the next-in-line would succeed, with the title devolving upon the posthumous child if born alive and eligible to succeed. So, the 4th Marquess of London dies leaving the Marchioness pregnant and his brother becomes the 5th Marquess; however, the widowed marchioness gives birth to a son who immediately becomes the 6th Marquess. Similarly, the 8th Baron Windsor dies leaving a daughter (or two) and a pregnant widow; the daughter becomes the 9th Baroness (or the title goes into abeyancy if she's got a sister), but a brother is born and becomes the 10th (or, in case of abeyancy, the 9th) Baron. The only basis for this assumption is the arrangement made for Victoria's accession to the throne (Victoria would succeed William IV even if Adelaide was left pregnant, but the posthumous child would ascend upon birth as if Victoria had died, as the throne can never be vacant). Surtsicna (talk) 22:30, 12 October 2012 (UTC)[reply]
An actual example occurs in the succession of the Petre Barony. The 7th Baron, Robert Petre, died in 1713, leaving a pregnant widow (Catherine, née Walmsley), who gave birth to the 8th Baron three months later. The Complete Peerage says of such sons that they "succeeded to the peerage at their birth". The 7th Baron inspired Pope's Rape of the Lock- Nunh-huh 03:14, 13 October 2012 (UTC)[reply]
Thank you, that's a very good example! However, upon the 7th Baron's death, there was nobody in line to succeed to the barony; had the child been female, the title would've become extinct. Therefore, the only option in that case was to wait for the child to be born, as there was nobody to hold the title in meantime. Surtsicna (talk) 07:32, 13 October 2012 (UTC)[reply]
Point taken. Here's a somewhat more apposite example.
When Edward Ward, the 8th Baron Dudley and 3th Baron Ward died of smallpox in March 1704 he left a pregnant wife (née Diana Howard) and no other progeny. He was succeeded by his posthumous son Edward Ward at his birth in June 1704. The 9th/4th Baron died unmarried and childless, and was succeeded by his uncle, the brother of the 8th/3rd Baron.
So we can see that when the 8th/3rd Baron died, he had—in addition to his unborn son—a brother who would have inherited his titles on his death, had Diana not been pregnant.
The Dudley barony was a barony created by writ in 1440 in the English Peerage. The Ward barony was a barony created by letters patent in 1644 in the English Peerage with a remainder to heirs male. So the titles were separated on the death of William Ward, the 10th Baron Dudley/5th Baron Ward, as only the Dudley barony could pass through a female line. The Ward title passed to John Ward of Willingsworth Hall, heir male of the body of Humble, the first Lord Ward, while the Dudley title devolved to the heir general, Ferdinando Dudley Lea of Halesowen Grange. - Nunh-huh 19:32, 14 October 2012 (UTC)[reply]
Anyway, this all suggests that during the period between the death of an otherwise childless peer leaving a pregnant widow and the birth of that possible heir, the title is considered at least conceptually to be dormant, that is, one for which the rightful holder, if any, is unknown, with the dormancy being sorted out once the child is born, rather than the solution you outline (provisional accession to the title, being rescinded if needed). However, I can't say I've ever actually seen the word dormancy applied in this exact situation, but it seems to capture the basic concept. Similarly this would apply for an heir to a barony by writ with only daughters and a bun in the oven: dormant until birth, then devolving to the son (if male) or entering abeyancy between the daughters (if female, or stillborn). - Nunh-huh 05:13, 15 October 2012 (UTC)[reply]
And the appropriately named Ladislaus the Posthumous, who succeeded to some, but not all, of his father's titles. Adam Bishop (talk) 16:48, 13 October 2012 (UTC)[reply]
Queen Victoria's youngest son Prince Leopold, Duke of Albany died in 1884, leaving a pregnant wife and daughter. The son succeeded to the title at birth. (Coincidentally, I just read that article a few minutes ago.) --NellieBlyMobile (talk) 21:12, 13 October 2012 (UTC)[reply]

Did all of Europe seriously abandoned Denmark to fight out the Second Schleswig War by themselves against Prussia and Austria? In the background section of the article it states the reason why the other nations of Europe didn't help and Sweden promised to help but never came with reinforcement, but did no nation in Europe protest the seizure of 40% of Denmark's territories and the enlargement of the Prussia threat in Europe after the war. --The Emperor's New Spy (talk) 20:06, 12 October 2012 (UTC)[reply]

Part of the problem, I believe, isn't that the rest of Europe merrily allowed Prussia to seize what had been unambiguous Danish National Land since time immemorial. You're looking at the situation through the modern understanding of the nation state and of sovereignty, two concepts that really didn't reach maturity until well into the 20th century. One thing to remember here is the complexity of the Schleswig-Holstein Question, which came to a head in that series of wars in the 19th century, but which had really existed for hundreds of years. The lands of Schelswig and Holstein had a sui generis sort of status for much of history, being simultaneously lands that were part of the Holy Roman Empire with imperial immediacy, and the hereditary homeland of the Danish kings since Christian I. The issue was simply which was more important: the relationship of the lands to the King of Denmark (as hereditary Danish lands) or to the German Confederation (as the successor state to the Holy Roman Empire). The same sort of confusing status had existed in the United Kingdom until the ascension of Queen Victoria allowed the Kingdom of Hanover to split off under Ernest Augustus I. The Schleswig-Holstein question stubbornly refused to resolve itself with a convenient succession crisis, and so as the various nation-states of Europe began to coalesce into their "natural" and "legal" limits in the 19th century, the Schleswig-Holstein question flared up into war. The rest of Europe didn't see the issue as one of Prussia violating Danish sovereignty, they saw it as a convoluted clusterfuck of a mess that they didn't want to get involved in. And I quote "Only three people...have ever really understood the Schleswig-Holstein business—the Prince Consort, who is dead—a German professor, who has gone mad—and I, who have forgotten all about it." --Jayron32 20:30, 12 October 2012 (UTC)[reply]
I understand Schleswig-Holstein Question, but the rest of Europe saw the First Schleswig War as Prussia's violation of Danish sovereignty and the status quo of the region in 1848, during the beginning of the whole nationalism spirit of the Revolutions of 1848. In the first war, Britain was openly against Prussia's invasion and was going to send her Navy to assist Denmark but the Danish refused at the time, Austria, France and Russia were all against Prussia in the first war and spoke out against the war, although no one actually send any forces to assist since Denmark could deal with it at the time. The reasons why these countries didn't support Denmark militarily in the second war like they tried to in the first war is explained, but what prevented them from protesting Prussia's takeover, and the fact that no nation came the aid of Denmark is really awkward since so many European wars involved the intervention of multiple nations. It doesn't make sense that no opposition to Prussia's takeover was ever made by the other nations after the war especially since no new independent Schleswig-Hostein state was ever created and Prussia just gain more territory when the purpose of the first war and the second war was to liberate the people of the region from Danish oppression. Also saying Prussia was somehow supporting nationalism and the whole idea that later became known as the self-determination, that Germans should rule Germans, is a bit ironic since a large amount of Prussia's population was Polish. --The Emperor's New Spy (talk) 20:58, 12 October 2012 (UTC)[reply]
The Danish government did actually hope that France or Great Britain would eventually intervene in favour of their cause as they had done in 1850. However it does not excuse the actions of the Danish government in this conflict in any way, since this hope was completely spurious. They actually acted on largely unfounded and unrealistic premises, and played right into Bismarcks hand. --Saddhiyama (talk) 21:35, 12 October 2012 (UTC)[reply]
Regarding the first Schelswig war, I'm pretty sure that the intervention of other European powers was not necessarily to defend Danish sovereignty, but rather to preserve the Balance of power in Europe; that and NOT issues of sovereignty of nation-states, was the main engine of diplomacy and international relations in Europe throughout the 19th century. Indeed, on the issue of Sovereignty, the majority of residents of the areas seemed to favor unity with Germany rather than Denmark, as they were ethnic Germans. The issue is confounded by the Revolutions of 1848, not explained by it. That complicates, rather than simplifies, any understanding of the causes of that war. --Jayron32 23:47, 12 October 2012 (UTC)[reply]

Was Britain even pro-German in 1864? The article Second Schleswig War said Queen Victoria was because Prince Albert died, but wasn't Queen Victoria: A. a figurehead with no actual power over the foreign and military affair of the country, and B. didn't she withdraw from politics and court completely after Prince Albert's death and didn't come out of her mourning until the Golden Jubilee. --The Emperor's New Spy (talk) 20:58, 12 October 2012 (UTC)[reply]

Britain was pro-Danish in this, but couldn't agree to act on the issue; see Henry_John_Temple,_3rd_Viscount_Palmerston#Denmark. Palmerston is sometimes blamed for provoking the war; "In actuality, Palmerston’s attitude during the Second Schleswig War in 1864 considerably helped the German decisive victory in that war, by letting the Danes get the wrong impression that Britain would fight on their side and thus emboldening them to embark on a war they had no chance of winning alone."[14] Alansplodge (talk) 21:49, 12 October 2012 (UTC)[reply]

What was Christian IX's view on his daughter Alexandra's position in this, was their any strain in the two's relationship? Was there some hostility between the Danish Royal Family and the British Royal Family for not coming to Denmark's aid in 1864?--The Emperor's New Spy (talk) 21:17, 12 October 2012 (UTC)[reply]

The U.K. couldn't save Hannover (which had been under the UK monarch from 1714-1837) in the Austro-Prussian war, so why should it have been expected to save Schleswig? (And Holstein was inhabited by Germans.) AnonMoos (talk) 22:14, 12 October 2012 (UTC)[reply]
A brief reading of our article on Lord Palmerston and the link posted above, suggests that Palmerston thought that the appearance of a huge British fleet off the coast of Jutland might make the Prussians have second thoughts. I suspect that (rather like the Americans in Kosovo), he was anxious to avoid getting involved in a land war, especially in the light of British failings in the Crimea, and thought that a show of strategic power projection by the RN would do the trick. When push-came-to-shove, he couldn't get Cabinet approval to send the fleet and so nothing was done. An alliance with France, who had a far larger army, might have done the trick; but at the time, Palmerston's government was furiously building dozens of massive forts around the British coast ("Palmerston's Follies"), because he expected Napoleon III to invade. Alansplodge (talk) 20:31, 14 October 2012 (UTC)[reply]

Conflation of totalitarianism with collectivism.

If I have a midlife crisis, then it will be around philosophy. Until I had Anthem in tenth grade, I never saw the individualism-collectivism scale as having a correlation with freed-totalitarianism. Objectivism has done an effective job at making me doubt things. Yet, I cannot help but think that ¬(◻(collectivism⇔totalitarianism)), ¬(◻(individualism⇔freedom)) and that ¬(◻(collectivism⇔freedom)) or ¬(◻(individualism⇔totalitarianism)). I am not sure what sort of question I should ask. Can a totalitarian society not be collectivist? --Melab±1 22:28, 12 October 2012 (UTC)[reply]

I think of collectivism as being related to socialism and communism, with individualism being related to capitalism. You can get totalitarianism at either end of the spectrum, from Stalin to the French kings before the French Revolution. StuRat (talk) 22:51, 12 October 2012 (UTC)[reply]
What in the world did the French kings have to do with individualism? Or capitalism, for that matter? --Trovatore (talk) 02:08, 13 October 2012 (UTC)[reply]
See bourgeoisie. That they allowed such a class to exist shows they allowed a certain degree of individualism and capitalism. StuRat (talk) 02:17, 13 October 2012 (UTC)[reply]
"Allowed to exist" does not show that there's any relation between that and their, what shall we say, "partial totalitarianism"? It was a check on their power, not an aspect of it. --Trovatore (talk) 19:36, 13 October 2012 (UTC)[reply]
When I was younger and smarter, I was wholly convinced of the near perfectness of Ayn Rand's philosophy on the world and its stress on individuality over collectivism. I was an earnest disciple, and bought in to every word she said. As I have gotten older and stupider, I have come to see more nuance in the world, and have adopted a more pragmatic view on humans with regard to the issue of collectivization vs. individuality. I think Rand's conflation of the difference between power relationships with free association is a key flaw in her philosophy. The issue is not whether people are dealt with as groups or as individuals, it is whether or not any aspect of their behavior is controlled by coersion or free choice. The great paradox of Rand's philosophy is that it denies the possibility that free-thinking individuals would, of their own choice and volition, establish collectivities for their own interests, and that these collectivities can be benign, or from the other direction that an individual, acting in their own rational self interest, could oppress others. Experience in the breadth of history clearly shows that both happen all the time, so the key issue shouldn't be whether people exist as self-interested individuals or in collectives (which are not mutually exclusive states) but rather on the level of control people have over the course of their own lives, and on how they relate to the power structures in their lives. In simpler terms: Rand's conflation of totalitarianism with collectivism breaks down in the face of actual historical examples. The world does not exist on a strictly two-dimensional axis, with all things evil on one pole and all things good on the other. --Jayron32 23:26, 12 October 2012 (UTC)[reply]
For me, what I always wondered about is why collectivism doesn't seem to scale up. We can have small groups who share everything equally, and everything works well. But, when nations try to do that, we end up with Stalin or Mao in charge. Perhaps it has to do with the fact that, on the small scale, if somebody tries to take over and run things for his own benefit, everyone can just leave and form a new group without that leader. Not so on a national scale. StuRat (talk) 01:37, 13 October 2012 (UTC)[reply]
I can't find it now, but look into something called the "rule of 150". I heard someone talk on it once, and it made a lot of sense. 150 is about the maximum number of people that can exist in a collective before everyone can't fully "keep track of" everyone else. In small enough groups, everyone knows everyone else well enough that we can interact as equals, and we can all take active ownership in the group because we all know exactly how each individual group member relates to each other group member and the group as a whole. So that's why small groups work. In larger groups, more than about 150 people, it becomes impossible for the human mind to make enough rational connections to keep the group all together in our heads, so the small group dynamic breaks down, and it becomes a VERY different entity; people stop interacting with each other and the group in the same way, and that's why big groups don't work like small groups. --Jayron32 01:46, 13 October 2012 (UTC)[reply]
Here it is: Dunbar's number. --Jayron32 01:47, 13 October 2012 (UTC)[reply]
Interesting. But it seems to me that collectivism can work on somewhat larger scales than that. My limit of "when people are free to leave if they want to" could be highly variable in terms of size. Even a tiny cult might threaten people who attempt to leave, while an entire nation could allow dissatisfied people to leave (and many do), but the particular difficulties of finding a new nation willing to accept you, selling all your possessions, moving thousands of miles, trying to find new employment, and leaving friends and family behind, all make it unlikely that everybody in a seriously mismanaged nation will just pick-up and leave. Too bad, it would make for an effective way to get rid of dictators (there would be little point in remaining a dictator, with no subjects to tax and abuse). StuRat (talk) 02:27, 13 October 2012 (UTC)[reply]
Hm, ditto interesting. I wonder if there's even been a place where the policy was "If you want to leave, not only are you welcome to do so, but the state will happily pay for all your expenses, because we don't want anyone here who doesn't want to be here, and it's in the national interest to ensure that those who want to leave do so as quickly and painlessly as possible". Probably not. -- Jack of Oz [Talk] 03:24, 13 October 2012 (UTC)[reply]


If you're grappling with the theoretical concept of totalitarianism you could do worse than to read Hannah Arendt on totalitarianism. Fifelfoo (talk) 20:23, 14 October 2012 (UTC)[reply]

October 13

Does the Queen own Canada?

In a recent issue of Fortune magazine, there is an article on a man who owns the most amount of land in the United States. In that article there is this sentence: "The largest landowner in the world is Queen Elizabeth II, because technically she owns places like Australia and Canada". Is this true? Does the Queen own entire countries? (The article is in the October 8 issue, volume 166, No 6. It doesn't seem to be online yet) RudolfRed (talk) 01:30, 13 October 2012 (UTC)[reply]

Not in any real way. If she tried to sell it, for example, that wouldn't happen. StuRat (talk) 01:32, 13 October 2012 (UTC)[reply]
Land tenure is a relevent article here for the OP to read. --Jayron32 01:39, 13 October 2012 (UTC)[reply]
Also Fee simple: "In English common law, the Crown has radical title or the allodium of all land in England, meaning that it is the ultimate "owner" of all land." Not sure how this relates to states of the Crown other than England directly, or even how it relates even practically to England. --Jayron32 01:41, 13 October 2012 (UTC)[reply]
The Tenure Abolition Act 1660 discontinued various forms of tenure and after the Law of Property Act 1925, almost all vestiges of this system had been removed. Although there are a few extant property rights relating to the Crown that have been imputed to the doctrine of tenure, e.g. the reversion of land to the state when no owner can be identified, the connection is vague and there are usually other sources to explain them. The main practical impact of tenure has been the development of the doctrine of estates (such as fee simple), a natural consequence of the tenure feudal system in which tenants owned an interest in the land as opposed to owning the land itself which all belonged to the Crown. Ankh.Morpork 19:57, 13 October 2012 (UTC)[reply]
A much smaller sub-set of all the land in England is the Crown Estate. It "belongs to" the monarch but is not "the private property of the monarch and cannot be sold by him/her". If she doesn't even own the Crown Estate, there's no way she owns all land in England, let alone elsewhere.
The queen does indeed have personal property that she can sell or bequeath to whomsoever she likes, just like any other person. Such as Balmoral Castle. This has been personally owned by members of the Royal Family since being purchased by Queen Victoria and Prince Albert. I guess the general expectation is that she would keep it in the Family and will it to her heir, Prince Charles, but I don’t think she’s under any legal restriction.
She may also own personal property in Australia, Canada and other places, but as for owning the entire nations, forget it. In some abstruse arcane legal sense the Crown may own all the land, but the Queen is not the Crown. She is the embodiment of the Crown. -- Jack of Oz [Talk] 02:20, 13 October 2012 (UTC)[reply]
Thanks, everyone.
Resolved
RudolfRed (talk) 03:44, 13 October 2012 (UTC)[reply]

Comments despite the "resolved" tag: There's more at Crown land, which begins by baldly stating "Crown land is an area belonging to the monarch", but goes on to say it is entailed and cannot be alienated from the monarchy. This makes is sound like the Queen does "technically" own vast amounts of land, but not in the fee simple sense of "own". As for Canada there is Crown land#Canada. This section likewise says that "the Canadian monarch owns all Crown Land in the country", but not in the normal sense of "owns". Since 89% of Canada is Crown Land you could say that the Queen owns 8,886,356 km² of land in Canada alone, if you're willing to stretch the normal sense of the word "own". Still, the statements in the Crown land article are pretty clear in saying that the monarch—the Queen—"owns" all the Crown Land; that the Crown Land "belongs" to "the monarch". The OP's quote reads "The largest landowner in the world is Queen Elizabeth II, because technically she owns places like Australia and Canada". One may say "yes but not really", but then the quote does say "technically", which implies "not really but yes". No? Pfly (talk) 05:12, 13 October 2012 (UTC)[reply]

Imho, the opening sentence of Crown land is inaccurate in equating the Crown with the monarch. From The Crown:
  • "The Crown is a corporation sole that in the Commonwealth realms and any provincial or state sub-divisions thereof represents the legal embodiment of governance, whether executive, legislative, or judicial. It evolved first in the United Kingdom as a separation of the literal crown and property of the nation state from the person and personal property of the monarch, a concept which then spread via British colonisation and is now rooted in the legal lexicon of the other 15 independent realms." (my highlighting).
This supports what I said above: Crown land is owned by the Crown, not by the Queen. -- Jack of Oz [Talk] 05:24, 13 October 2012 (UTC)[reply]
Hmm, maybe the Crown land page ought to be copyedited and improved.... Pfly (talk) 05:36, 13 October 2012 (UTC)[reply]
Yes, that would be my strong suggestion. -- Jack of Oz [Talk] 05:54, 13 October 2012 (UTC)[reply]
The US equivalent of Canada's Crown land is Federal lands - which is not the personal property of Barack Obama, but the land is under the control of various departments and agencies which are ultimately the responsibility of the president. Roger (talk) 14:32, 13 October 2012 (UTC)[reply]
The Queen and other Royals also own several properties in the United States and have a few "ventures" here as well. Marketdiamond (talk) 16:37, 13 October 2012 (UTC)[reply]
See, for example, [15] which reads:
'"Crown" means Her Majesty the Queen in right of the province; (« Couronne »)
"Crown lands" includes land, whether within or without the province, vested in the Crown, and includes "provincial lands" wherever that expression is used in an Act of the Legislature; (« terres domaniales »)'
I would say the distinction to be made is that she does not own it as personal property but she does own it in the sense of it being hers while under the constraints of the constitution and other legislation. In a similar fashion she is the head of state, i.e, she is the head of state as proscribed by the relevant legislation, i.e, de jure but not de facto. So she does not own the land at all in the sense that John Malone or Ted Turner own their land. First of all, they are not even in the same jurisdictions, so they are not entitled according to the same legislation or policies, but they are not even entitled according to analogous legislation or rights. --Atethnekos (DiscussionContributions) 19:44, 13 October 2012 (UTC)[reply]
There must be some standard for ownership that applies to entailed land, which is clearer. For example, if the Queen decides that she doesn't like cigarette smoking, what lands can she ban people from doing it on as a condition of their visit, rent, or conduct as a customer? Wnt (talk) 19:58, 13 October 2012 (UTC)[reply]
Balmoral Castle, for one. Assuming the law permits private land owners to ban smoking on their land - and I'm not sure that it does - she could ban smoking there, because she owns it personally. WHAAOE, but I don't know whether we have a list of all land and other property personally owned by the Queen. -- Jack of Oz [Talk] 21:31, 13 October 2012 (UTC)[reply]
She could, but I doubt she'd be so crass as to make it By Order of Her Majesty The Queen. Word would trickle down backstairs and it would be done administratively at some level. If I recall correctly, Balmoral was one of the things that had to be purchased from the former Edward VIII, Sandringham as well.--Wehwalt (talk) 17:53, 15 October 2012 (UTC)[reply]
Yes, that's right. The Duke of Windsor had never shown much interest in them as king, and was apparently happy to sell them to his brother George VI. But he could have held on to them and bequeathed them to the Duchess of Windsor, who could have bequeathed them on her death to her French nurse, housekeeper and gardener. -- Jack of Oz [Talk] 22:06, 15 October 2012 (UTC)[reply]

People on the image

As seen on the picture, there's several soldiers/men-at-arms etc wearing these curious coat-of-arms i havn't found anywhere(!)

1)Are the persons all (or atleast some) known by name, where they came from, like "duchy of Burgundy" etc or is it jus an image of a completely or partly fictional ppl/coat of arms?

2) If they are known, any idea where those black/white & black/yellow "rampart lion" coat of arms and the Red/white fleur-de-lis coat of arms are from?

Im guessing the White/Black coat of arm is from somewhere in Duchy of Brabant or Duchy of Jülich, but that's just a guessimate.

allsow, why isn't this feference-desk page archived more often? It's huge and laggs allot :(

--Byzantios (talk) 03:21, 13 October 2012 (UTC)[reply]

Did you look at the Battle of Worringen article? The infobox lists the belligerents and shows the coat of arms for each of them. (Regarding archiving, the system is broken at the moment, but people are working on it.) Looie496 (talk) 03:35, 13 October 2012 (UTC)[reply]
left: ?, mid: Duke John of Brabant, right: knights of Duke John of Brabant (Zangemeister 1892, p.2)
(thru ec) See for a suggestion of where the red/silver and gold/black lions rampants might come from (John I, Duke of Brabant) - Nunh-huh 03:37, 13 October 2012 (UTC)[reply]
BTW, what's with that border on the 2nd illustration ? It looks like it was colored by a kindergartner having trouble staying inside the lines. Is this another case of an incompetent art restorer ? StuRat (talk) 04:03, 13 October 2012 (UTC) [reply]
I wouldn't expect to recognize all the arms in such a scene; the armigers are likely to include dukes and obscure knights. —Tamfang (talk) 05:35, 13 October 2012 (UTC)[reply]
The participants and their coats of arms are identified in Werner Schäfke (ed.): "Der Name der Freiheit 1288-1988", Cologne 1988. --Pp.paul.4 (talk) 11:56, 13 October 2012 (UTC)[reply]
You might also be interested in my new article de:Brabantsche Yeesten which calls the images realistic and links to the complete manuscript online and in the fact that in John I, Duke of Brabant the image shown is not that of Duke John but of the mythical king Gambrinus, inventor of beer brewing. --Pp.paul.4 (talk) 20:36, 14 October 2012 (UTC)[reply]

Duchy of Cornwall and changes to the British succession law

What will happen to the Duchy of Cornwall if male-preferance primogeniture is replaced with absolute primogeniture? Per Edward III's royal charter of 1421, which created the duchy, it can only pass to the Sovereign's eldest son. What if the succession law is changed? What if William has a daughter first and then a son? The daughter cannot possibly be Duchess of Cornwall, as the title is not conferred to the heir apparent, but strictly to the eldest son. Would the royal family then consist of "HM The King, HM The Queen, HRH The Princess of Wales and HRH The Duke of Cornwall"? If so, what would happen when that Princess of Wales ascends the throne and has a son of her own while her brother is still alive? This is intriguing because the duchy is not merely a title, but also a significant source of the heir apparent's income. Surtsicna (talk) 10:32, 13 October 2012 (UTC)[reply]

That's only because the monarch's eldest son, if a son exists, and the heir apparent have always been one and the same person. The whole point of the proposed change is to no longer have that automatic linkage. There's no reason why the eldest son couldn't still become the Duke of Cornwall, regardless of whether he's also the heir apparent. Is there? Or, the Queen could issue new Letters Patent in respect of the Duchy of Cornwall. There'll need to be a very large amount of paperwork to put this conceptually very simple change into effect, because it's these sorts of ramifications that will be the really tricky bits. That's why they're taking their time to consider all these issues in great detail before rushing in.
I think you're assuming a lot when you refer to the eldest-born daughter as the Princess of Wales. True, the title Prince of Wales is only bestowed on the heir apparent, that is, someone who cannot be displaced in the line of succession by any later births; and after the proposed changes go through, an eldest-born daughter will be the heiress apparent, not merely heiress presumptive. But the title has only ever been given to males, and their wives are called Princess of Wales. There have been no announcements about how this aspect of the set-up will be organised in future.
On your last point: The Queen was never Princess of Wales or Duchess of Cornwall, but she seems to be getting on ok finacially. -- Jack of Oz [Talk] 10:57, 13 October 2012 (UTC)[reply]
As I read it, we already have the answer. From our Duchy of Cornwall article, If the monarch has no son, the estates of the duchy are held by the crown, and there is no duke and Since the passing into law of the Sovereign Grant Act 2011, revenues of the Duchy of Cornwall will pass to the heir to the throne, regardless of whether that heir is the Duke of Cornwall. In the event that the heir is a minor, 10% of the revenues will pass to the heir, with the balance passing to the Crown. To me, this seems to cover the eventualities you're asking about. If Wills has a daughter whilst the Queen is still on the throne, on William's accession the daughter will NOT become Duchess (or, indeed, Duke) of Cornwall, but she WILL get the benefit of the estate. The duchy itself will be vested in the Crown. Once a further male heir apparent to the throne is available, he will get both the title and the money. - Cucumber Mike (talk) 11:04, 13 October 2012 (UTC)[reply]
JackofoZ, the eldest son and the heir apparent have not always been the same person. King George III was Prince of Wales prior to his accession but never Duke of Cornwall because he was heir apparent to his grandfather, King George II, his father Frederick having predeceased George II. There is a reason why the eldest son couldn't still become Duke of Cornwall if his older sister is heiress apparent - if she were to succeed to the throne during his lifetime, as would be expected, her son would have to become Duke of Cornwall according to the charter, but two people cannot possibly hold the same title at the same time and enjoy the duchy's revenues. Furthermore, I am not sure letters patent could amend the charter; is it possible? I also believe that you underestimate the importance of the Duchy of Cornwall, ignoring both its financial ([16], [17]) and political significance. For example, the Duke of Cornwall has a right to veto government bills (as Charles himself has done 12 times). Surtsicna (talk) 11:09, 13 October 2012 (UTC)[reply]
As I said, "it's these sorts of ramifications that will be the really tricky bits".  :) -- Jack of Oz [Talk] 11:21, 13 October 2012 (UTC)[reply]
Evidently. It's just that I don't see how exactly they'll sort this out. Surtsicna (talk) 11:26, 13 October 2012 (UTC)[reply]
"New Zealand will chair a working group to consider the best way of accomplishing this reform in all the countries concerned. In the United Kingdom, the government is examining what legislation needs to be amended."
That last sentence encompasses this issue and, I'm sure, many others. We could speculate all day about what they might do to resolve the issue; but what they actually decide to do is all that matters, and nobody knows that yet. We may be pleasantly surprised by their creativity. -- Jack of Oz [Talk] 21:25, 13 October 2012 (UTC)[reply]
Cucumber Mike, that's an interesting observation. However, the charter does not say that the duchy is held by the "eldest son who is also heir apparent". It only says that it's held by the eldest son. Some evidence suggests that James Francis Edward Stuart remained Duke of Cornwall after ceasing to be heir apparent; his half-sister Anne was heiress apparent between 1694 (death of her sister Mary II) and 1702 (death of her brother-in-law William III), the same year James finally lost the duchy having been attainted. That's according to our article Duke of Cornwall and some sources I came cross after a quick search (e.g. [18]). Surtsicna (talk) 11:19, 13 October 2012 (UTC)[reply]
OK, so you're saying that if the heir apparent was a girl with a younger brother, the brother would be Duke? Yes, I can see how that would work. Note, though, that it would be the heir apparent who would be getting the money, as per the Succession Act referred to above. I would strongly suspect that, were this actually to come about, some piece of legislation would be drawn up sharpish to remove the anomaly. - Cucumber Mike (talk) 12:03, 13 October 2012 (UTC)[reply]
I too can see how that would work until that girl ascends the throne. On her accession, her own eldest son (if any) would have to become Duke of Cornwall though her brother, already Duke of Cornwall, is still alive. Two persons cannot hold the same title and enjoy all its privileges. What occured to me is that the brother would cease being Duke of Cornwall, now being brother of the Sovereign rather than eldest son, but there's an unofficial rule that says that individuals cannot lose their title due to something that isn't their own fault (i.e. they can normally lose the title only if they get attainted). That's why heirs presumptive are not created Princes of Wales; that's why the eldest daughter of the Sovereign is not created Princess Royal if her aunt already holds the title (e.g. Princess Anne and Mary, Countess of Harewood), etc. Surtsicna (talk) 13:01, 13 October 2012 (UTC)[reply]
No-one's mentioned WP:WHAAOE: 2011 proposals to change the rules of royal succession in the Commonwealth realms. Ghmyrtle (talk) 12:44, 13 October 2012 (UTC)[reply]
Unfortunately, the article doesn't seem to mention this issue. Surtsicna (talk) 13:01, 13 October 2012 (UTC)[reply]
Quite possibly because no-one but a Wikipedia editor would have thought of it. Ghmyrtle (talk) 13:23, 13 October 2012 (UTC)[reply]
If only people would listen to us! Perhaps WP:OR should be amended... Surtsicna (talk) 17:30, 13 October 2012 (UTC)[reply]
It's a very simple question to answer. What will happen is whatever it says will happen in whatever new law changes the order of succession (and any associated letters patent which may be issued at the same time to tidy up the details). The reference desk's repeated requisitions for a new crystal ball have all been refused, however, so we have no way to know what that will be. --Tango (talk) 19:54, 13 October 2012 (UTC)[reply]

Incomplete information

This article: http://en.wikipedia.org/wiki/List_of_members_of_the_United_States_Congress_by_longevity_of_service

Tells us nothing about currency. When did this actually occur?... between xx/xx/xx and xx/xx/xx or from xx/xx/xx or ???

You cannot combine this type of information and expect it to have meaning. What you have done is similar to telling me how many Bronze, Silver and Gold Olympic Medals have been won by a particular country without the years or span of years in which they were won.

How about completing the information ?

Regards,

Connolly — Preceding unsigned comment added by 78.121.151.38 (talk) 16:37, 13 October 2012 (UTC)[reply]

If you have suggestions for improving an article, you can raise it on the article's talk page. Or you can be bold and start working on it. RudolfRed (talk) 17:01, 13 October 2012 (UTC)[reply]
Hi there, as you may already know we are all, like you -- unpaid volunteers -- and although the article is much more interactive and descriptive than other unnamed online encyclopedias because of the uniqueness of allowing all to edit it also empowers us all, including you and me to encyclopedically improve wikiarticles! If you have information you feel is important but are not comfortable with how to format it into a Wikipedia article you can always seek assistance at the Village Pump for technical help, or WP:HELPDESK, happy editing! Marketdiamond (talk) 20:15, 13 October 2012 (UTC)[reply]

October 14

Electoral College - United States

In the a article Electoral college it says, "The United States is the only current example of an indirectly elected executive president, with an electoral college comprising electors representing the 50 states and the federal district". My question is how is this true? I look at systems in Pakistan and India for example and it seems to be the same to me. People vote for electors and then those electors choose the president. The only difference I see is that the electors in the American, ONLY vote for the president and then their job is done. In India and Pakistan you have the two houses and others choosing the president, in addition to other various duties. But it still is indirect, isn't it? An average citizen in Pakistan doesn't choose who the president will be. He chooses people who will choose on his behalf.75.166.140.82 (talk) 04:00, 14 October 2012 (UTC)[reply]

Just a thought: In the US, some members of the Electoral College can vote as they please (although, for various reasons, unfaithful electors are rare). Is this also the case in those other nations ? StuRat (talk) 04:11, 14 October 2012 (UTC)[reply]
An unfaithful elector, however, has never impacted the results of an election. --Jethro B 04:20, 14 October 2012 (UTC)[reply]
But, theoretically, they could. StuRat (talk) 04:22, 14 October 2012 (UTC)[reply]
Even if that would technically be true, India and Pakistan's presidents are only figureheads. Obama is both head of state and head of government. So it's possible that the US is the only example, or one of the few examples, of a country with a Presidential system with an indirectly elected president. And even then, the results of the popular vote are usually the same as the electoral college vote, with a handful of notable exceptions. Narutolovehinata5 tccsdnew 05:07, 14 October 2012 (UTC)[reply]
If I recall correctly, even the "only example of an indirectly elected executive president" part is flawed, since I think the presidents of Suriname and Burundi are indirectly elected as well (elected by parliament), and at least in Burundi they are a presidential system. Indonesia's president was also elected by parliament until about 10 years ago when they switched to direct elections. In my country, the Philippines, the President has always been directly elected from the start. South Africa also has a system where the President is elected by parliament and is technically a prime minister but is both head of state and head of government. The same system is used in Botswana and the Marshall Islands. Correct me if I'm wrong, but to my knowledge, the electoral college system where people vote for electors who elect the President is unique to the United States; the closest equivalent would probably be Hong Kong's Election Committee, and even then I'm not sure if it is a true parallel to the Electoral College. But it's true that almost all countries with a presidential system directly elect their president. Narutolovehinata5 tccsdnew 05:04, 14 October 2012 (UTC)[reply]

Ok, America may be the only presidential system that uses an electoral college to elect its president. But America is not the only system that uses an electoral college to elect its president. Correct? So now we are just haggling over what the president does in various systems. But this process of electing a president is not unique to the USA. The articles Electoral_College_(Pakistan) and Electoral_College_(India) say clearly that the president of Pakistan/India is chosen by an electoral college. Am I the only who is confused here? In addition, even if you forget the president and consider the prime ministers, Pakistani prime minister is still elected by the Pakistani National Assembly which is popularly elected so we have an electoral college. In India, the president is not just a figure head (in Pakistan the president is) and the prime minister is appointed by the president which is like two levels of electorals.

So in India, people elect the parliament, The parliament elects the president. The president appoints the prime minister. The president is elected by a college. The prime minister is elected through two colleges.

In Pakistan, people elect the parliament (and the provincial assemblies). Both houses of the parliament and the provincial assemblies elect the president. The prime minister is chosen by the national assembly (the lower house of the parliament). Both the president and the prime minister are elected by a college. The colleges are not the same but a college is used nonetheless. True? 75.166.140.82 (talk) 06:23, 14 October 2012 (UTC)[reply]

The statement in the article is unsourced, so just amend it so it says "an example" instead of "the only example". In France, the Senate is elected by people serving as local councillors, etc., but I don't know if they are referred to as an electoral college in English. Itsmejudith (talk) 08:53, 14 October 2012 (UTC)[reply]

Wild Lands Project Corridor system

I have been reading up on the wild lands project and I have some questions about some of their planning. From what I have been able to find out, they want to implement a corridor system in the U.S. that would restrict certain areas of land from certain types of human development, but I haven't been able to find any documents put out by the Wild Lands Project on exactly what kinds of human development would be restricted in the corridors they recommend implementing. Can anyone give me any links to documents explaining what they plan on restricting? Jjhodgson (talk) 04:53, 14 October 2012 (UTC)[reply]

I don't know about this particular proposal, but the general idea of wildlife corridors is to allow animals to move between wilderness areas, so they can migrate, find food, find mates, etc. Therefore, anything which blocks such movement is a problem. In some cases the animals can be accommodated by making special provisions for them. So, for example, if a highway separates two wilderness areas, an occasional tunnel underneath can be constructed to allow them to cross under it safely. StuRat (talk) 05:19, 14 October 2012 (UTC)[reply]
Are you talking about Wildlife Trust of India#Wild Lands Project? Alansplodge (talk) 20:14, 14 October 2012 (UTC)[reply]
Or Wildlands Project? They have some rather grand ideas about "corridors" they call "wildways", [19]. If them, then browse their website. I think they are a small organization and not about to do anything particularly large scale. Pfly (talk) 01:43, 15 October 2012 (UTC)[reply]
"If them" ? StuRat (talk) 05:11, 15 October 2012 (UTC) [reply]
"If [you, the OP, wants to know more about] them..." Ok, maybe not the most elegant English... Pfly (talk) 08:25, 15 October 2012 (UTC)[reply]
No apology needed, you've finally found a way to write without using all those pesky words. :-) StuRat (talk) 08:34, 15 October 2012 (UTC) [reply]
srsly Pfly (talk) 01:03, 16 October 2012 (UTC) [reply]

"Hate crime" laws and crimes against women

My question is regarding Hate crime laws.

A significant number (though far from all) violent crimes against women are motivated by a hatred of the female half of humanity. An example which springs to mind in my part of the world is Peter Dupas. The sentencing judge, in one of the murder cases, referred to a " a perverted and sadistic hatred of women" as a key motive in his crimes, which included some gruesome stabbings, murders and rapes. I am sure he is far from unique, by a long stretch.

I am not suggesting that every rapist is a hate-criminal. But my question is, in dealing with those such as Dupas who verifiably are, is there any jurisdiction which, in practice, routinely uses hate-crime laws in charging such individuals? And if not, why not? 203.45.95.236 (talk) 10:37, 14 October 2012 (UTC)[reply]

Polarization in US politics

What reasons have been suggested for the US political polarization and the lack of a significant middle ground which confounds a normal distribution model? Ankh.Morpork 10:46, 14 October 2012 (UTC)[reply]

It's often suggested that closed primary elections cause more extreme candidates to appear on the ballot. See [20] for example. The idea: closed primary voters tend to be the most passionate about their party, i.e. extremists. They will support the most extreme primary candidate, so the only candidates to make it through the primaries will be extreme ones. Staecker (talk) 11:54, 14 October 2012 (UTC)[reply]
While it may well be the case that closed primaries promote political partisanship, I am curious whether this is not just the result of the idiosyncrasies of a particular political mechanism but also a manifestation of wider social divergences that have effectuated this schism. Ankh.Morpork 18:16, 14 October 2012 (UTC)[reply]
You don't necessarily need lots of extremists voting in primaries to get that effect. A first-past-the-post system tends to result in a race to the middle. With a closed primary, "the middle" is the middle of that party, rather than middle of the nation, which results in the actual presidential election being a right vs left thing rather than the centre-right vs centre-left election you would get in a standard first-past-the-post system. (Voters in primaries do take electability into account to some extent, which means you do end up with someone closer to the centre than most of the party, but they are still a long way form the centre.) --Tango (talk) 19:37, 14 October 2012 (UTC)[reply]
Whatever model or explanation is used, it will have to account for the fact that the Democratic party remains pretty much what it's been for the last 40 years, while the Republican party has become more rigid and ideologically extreme (the "Rockefeller Republicans" used to be a significant wing of the party, but after this November's elections, Susan Collins will be just about the only remaining "Rockefeller Republican" officeholder). From Jim de Mint's "Waterloo strategy" of not cooperating with any measures for economic recovery etc. if they might redound to the political benefit of Obama (for some reason not mentioned on the Jim de Mint article) to the fanatical and dogged determination to retain tax policies which greatly favor the ultra-wealthy (seen in the debt ceiling crisis and "fiscal cliff"), all Republican economic measures seem to be guided by extremist Ayn Rand / Grover Norquist ideologies, and a concern for the well-being of the ultra-wealthy far above all others... AnonMoos (talk) 13:58, 14 October 2012 (UTC)[reply]
Halite crystals
Halite crystals
Here are a few grains of salt to go along with AnonMoos's left wing view of an objective description of American politics.
Consider that Romney, who socialized medicine in Massachusetts, was the most "moderate" of the candidates to chart in the Republican primary, and is despised by a large portion of the Republican base as a tool of the moderate establishment. Note that Romney wants to cut taxes across the board, and limit deductions, which allow the rich to avoid taxes, at $17,000. As for who's moved where, compare "New Democrat" Bill Clinton, who balanced the budget and ended welfare as a lifetime entitlement, with Barack "Where'd that $6,000,000,000,000.00 go?" Obama, who's repealed the 1994 welfare reform, seized General Motors, and socialized 1/6th of the economy. But yeah, blame it all on a 30-years dead Russian Jew if you need a handy boogity man. P.S. Atlas Shrugged Part II is in theaters if you need a good scare. μηδείς (talk) 17:24, 14 October 2012 (UTC)[reply]
I had assumed that AnonMoos' post was a satire of leftist rhetoric, meant to illustrate the issue by example, i.e. that Republicans who object to an ever-expanding welfare state are "extremists". When Democrats insist that not raising taxes is "extremist", they've forsaken the middle ground. Romney tosses red meat to his right, but he's more-or-less a moderate like Bill Clinton ended up being. If the Democrats take another beating like they did in 2010, they may find it necessary to move back to the center. 24.209.99.109 (talk) 17:59, 14 October 2012 (UTC)[reply]
In the current circumstances, with the ultra-wealthy paying a smaller fraction of US taxes than in many decades, while middle-class take-home pay is stagnant and student-loan debts pile up, placing rigid inflexible dogmatic adherence to Grover Norquist pledges as your highest priority above all other priorities (such as avoiding drastic and draconian cuts to all federal govt. non-military spending, or preventing a further U.S. govt credit rating downgrade) is far out of the traditional mainstream, and appears to be motivated by a fanatical devotion to extremist Ayn Rand ideology and/or a concern only for the well-being of ultra-wealthy combined with a callous indifference to the well-being of everybody else. Romney has (or had) some moderate instincts, but to get ahead in today's GOP, he's had to present himself as being more ultra than the ultras (and he doubled down on that by choosing Ayn-Randite Paul Ryan as for his VP), so it would be foolish to vote for Romney based on trust in his "moderation". And I notice you didn't try to dispute my point about the Democratic party being not too different from what it was about 40 years ago -- while the GOP has radically changed since then... AnonMoos (talk) 19:26, 14 October 2012 (UTC)[reply]
Generally agree, but not on the last. Both parties have galloped way to the right since 1972: Nixon vs. McGovern (probably a little left of today's Jill Stein), Nixon sometimes, but not always, a little to her right, except on foreign intervention, where far to the right. Today's Dems are far to the right of the 1972 Republicans. What's the world come to, when you, AnonMoos, are the left & I have to cover your flank? :-) Have others ever seen a real leftie?John Z (talk) 22:41, 14 October 2012 (UTC)[reply]
Obviously many details and surrounding circumstances have changed over the course of 40 or so years, and certain slogans (such as "full employment") which didn't have much of a chance of being implemented have been abandoned. However, I think it's basically true that the Democratic party occupies somewhat the same overall position on the political spectrum that it has done for the last 40 years or so, while the ideological center-of-gravity of the Republican party has shifted radically during that same time. If the Democratic party has remained pretty much where it was (or even drifted slightly rightwards), while the Republican party has undergone a radical transformation, then it would appear to me that almost all the "polarization" has been on one side, and it's pointless and obfuscatory to pretend that there's been a mutual or "balanced" polarization. (Note that I don't include under "polarization" a party seeking to consolidate its power through legislative committee assignments, gerrymandering, etc. -- this is tactical maneuvering, not ideology.) AnonMoos (talk) 14:51, 15 October 2012 (UTC)[reply]
No, the biggest shift might have been with the Dems. They've both moved far to the right. Both 2012 party positions would have been thought mentally & morally deranged in 1972. Because they are. For full-throated unabashed lunacy, yes, you have to go to the Repubs, but the Dems are the same under a thin veneer, and can be more effective enemies of the 99% because of this. "Full employment" was not a slogan, but something which was consciously abandoned. It is the easiest thing in the world to achieve full employment. No nation which has ever tried has failed. The postwar era was the most prosperous in history because just about every state in the world decided to have full employment. And then nearly every one abandoned it. Nixon was way to the left of either Obama or Romney, a hawkish Jill Stein. He proposed a real national healthcare system, a guaranteed government job for everyone, even a negative income tax. He started the EPA. In important ways, the Repubs moved to the left of the Dems in actuality, if not rhetoric. To attack Reagan, the Dems became deficit terrorists inventing risible, imaginary problems with deficit spending. So with a shift understood only by a few economists & Chomsky at the time in the intelligentsia, but understood by ordinary voters, reversing the pattern since FDR or a bit before, the Republicans became the big spenders, with low unemployment and a good economy. Even though Republican spending was on the worst, most corrupt, welfare-for-the-rich things, and they raised taxes on the 99%, it could be better than high-tax austerity under the Dems. I mean, I wish you were right AnonMoos, I really do.John Z (talk) 02:14, 16 October 2012 (UTC)[reply]
Why/how does the US system force the wide spectrum of political factions to coalesce into only two parties?
Looking at the US from the outside, it seems like the Republicans could "sensibly" be split into at least three or four distinct parties ranging from "hard right" to "centrist-tory" and the Democrats could similarly split ino a spectrum ranging from "lib-dem" to "classic socialists". In many respects there are wider differences inside the two parties than between them. Roger (talk) 18:19, 14 October 2012 (UTC)[reply]
Two-party_system#Causes might help a bit. Ankh.Morpork 18:22, 14 October 2012 (UTC)[reply]
Thanks! WHAAOE wins again! Roger (talk) 18:37, 14 October 2012 (UTC)[reply]
It is not that an ideological spectrum cannot exist — it does exist. It's that (as the article linked to discusses) we have a winner-take-all system in the United States. In individual races, there is no prize for coming in second place: the first place person gets everything. In the Presidential elections, each state (with one exception) basically gets to cast its votes for one person, in the end. The result of this is a system that has much less flexibility when it comes to minority parties than, say, European parliamentary systems, where a party that can must 33% of the vote is considered to be truly impressive, because they're get a third of the seats all to themselves. Third parties (much less fourth or fifth parties) serve only to split the vote from whatever of the other two parties they might have otherwise been inclined to support. (I'm not casting that as a moral argument — sometimes there's a good reason to want to split the vote of your own party.) --Mr.98 (talk) 18:36, 14 October 2012 (UTC)[reply]
The repeal of the Fairness Doctrine, in 1987, which required each media outlet to supply "equal time for opposing viewpoints" is a probably a prime cause of polarization. Under that policy, people were exposed to all viewpoints. Now, conservatives only watch conservative media outlets and liberals only watch liberal media outlets, so each has their own worldview reinforced and magnified, not moderated as before. StuRat (talk) 18:49, 14 October 2012 (UTC)[reply]
The polarization began well before the late 1980s. The idea that people were truly exposed to all viewpoints prior to the late 1980s is completely silly. (And while I respect, I guess, the attempt to make it look like this is something that affects "both sides," it should be noted that study after study has shown that it is the conservatives who are at the moment most limited in the media they consume, and that most non-conservatives are comparatively broad in what sorts of things they watch and read. There is no liberal equivalent to Fox News in terms of market share and complete polarization.)
A better historical approach, in my view, would look at the broader evolution of partisan politics over the 20th century. The period of most harmony between the political parties was the early 1960s. A lot of stuff happened since then, though, which led to more or less the politics we have today. A very lively account of this is Rick Perlstein's Nixonland: The Rise of a President and the Fracturing of America. --Mr.98 (talk) 21:10, 14 October 2012 (UTC)[reply]
California has changed its closed primary system and we still see examples of extremist candidates. The state legislative district one district east of where I live has two extremist right wing Republicans running against each other, because the Democratic and moderate Republican candidates were eliminated in the primary. 69.62.243.48 (talk) 23:53, 14 October 2012 (UTC)[reply]
They don't have the Republican winner of the primary run against the Democratic winner ? StuRat (talk) 05:10, 15 October 2012 (UTC)[reply]
If they are truly open primaries and voting in one doesn't stop you voting in the other, then there isn't actually a Republican primary and a Democratic one. What would be the difference between the two? --Tango (talk) 20:39, 15 October 2012 (UTC)[reply]
Presumably you still have to be a registered Republican to run in the Republican primary and vice-versa. StuRat (talk) 22:17, 15 October 2012 (UTC)[reply]
Perhaps, but there is nothing stopping a Democrat registering as a Republican. It might stop someone running in both primaries, but it wouldn't control which primary they run in. --Tango (talk) 11:44, 16 October 2012 (UTC)[reply]

Speaking as European, your politics don't look particularly polarised to me. You seem to have a left of right party and a centrish right party. It's the way British politics looks to be heading, too. --Dweller (talk) 12:44, 16 October 2012 (UTC)[reply]

World Trade Center target long before 9/11

I just stumbled upon this CNN article in which it's stated that Eric Harris, one of the Columbine shooters, envisioned hijacking a plane and crashing it into a building in New York. My question is, since when did it become a target? And were there ways to avoid an attack? Thank you. Iowafromiowa (talk) 14:42, 14 October 2012 (UTC)[reply]

1) The WTC was evidently a target since before the World Trade Center bombing of 1993.
2) Eric Harris' delusional adolescent ravings make no reference to the WTC. CNN's reporters ought to learn to distinguish 'detailed plans' from 'extravagant fantasies'. Harris' proven ability to commit a firearms massacre is no guide as to his ability to 'hijack a load of bombs', etc.
3) It's easy to be wise after the fact. Presumably some of the anti-terrorist measures now in place might have been effective in preventing the attacks. However, introducing those measures would doubtless have causes the terrorists to choose a different strategy. The risk of an Al-Qaeda-backed hijacking was clearly known in the 1990s, as I recall an 'Alex' cartoon strip in the Daily Telegraph referring to 'this Bin Liner chappie' in such a context from well before 2001. AlexTiefling (talk) 14:55, 14 October 2012 (UTC)[reply]
There were a spate of prognosticators after the '93 attack saying that the next time they would use planes. There was also a lot of talk in August of 2001 that there was chatter about a big attack coming. I have searched for this on occasion, but it's very hard given 9/11 results drown out the desired hits. μηδείς (talk) 17:06, 14 October 2012 (UTC)[reply]
As far as ways to avoid an attack, simply banning people from bringing weapons on board in the form of box-cutters went a long way. Whoever decided to allow those was a moron and should have been fired for incompetence. There are security checks which put quite a burden on the passengers, like pat downs, but how many people will be seriously inconvenienced by not being able to carry box-cutter knives with them ? StuRat (talk) 18:41, 14 October 2012 (UTC)[reply]
See this previous thread about the boxcutter issue. It's not even certain that the weapons (if any), were boxcutters (Stanley knives for UK readers). Alansplodge (talk) 20:11, 14 October 2012 (UTC)[reply]
Well... [21]. Etcetera. They're likely if they can spot a hand grenade lying in a suitcase. A box cutter is so simple, I can't imagine how they could stop it. I mean, how do they tell if a Zippo lighter has a square of razor blade hidden in it? How do they stop someone from mounting that on some kind of plastic handle they carried along? Wnt (talk) 20:05, 15 October 2012 (UTC)[reply]
Razor blades should show up well on X-ray machines. Also, a bare razor blade wouldn't be a very effective weapon, you really need the rest of the tool to go with it. And, lighters shouldn't be allowed on a plane, either. StuRat (talk) 03:58, 16 October 2012 (UTC)[reply]
I should note that that older thread included an incorrect assertion that the knives couldn't really kill anyone. Apparently Daniel M. Lewin, the first casualty of the September 11th attacks, was killed with one of the knives by having his throat stabbed or slashed - not sure how many others died this way. This was despite Lewin having some military training as described in American Airlines Flight 11. Wnt (talk) 20:15, 15 October 2012 (UTC)[reply]

Autism & Mercury - Government Conspiracy for Population Control?

The reference desk does not answer requests for opinions or predictions about future events. Do not start a debate; please seek an internet forum instead.
The following discussion has been closed. Please do not modify it.

The Autism Epidemic keeps progressing unchecked. It was 1 / 166, 1 / 150, and now 1 / 88 kids who are born have autism.

China gets to be overt with their population control policy. Their citizens do not have as much power to fight back against it; they are often repressed and brutalized.

However, way too many Americans would be up in arms about anything similar.

Therefore, what if a different way to control population growth happens behind our backs, out of sight of the media?

It could be that they leave pollutants in the food (and sometimes water?) supplies on purpose, so that pregnant mothers who take them cause their babies to be born with defects that make them "socially ineligible" to marry and have kids of their own someday. Do you see the EPA mandating mercury removal from the catches of fish?

And so, with enough children having such defects, the government can discreetly breathe a sigh of relief and not worry about population overgrowth.

And possibly, because I post this conspiracy theory here, I could get taken away by trenchcoated men.

So,

A. How do we know that there is no governmental conspiracy to control population growth by infecting kids with autism-inducing pollutants before they're born? (Please do not answer if you are a governmental agent who is capable of participating in said conspiracy.)

B. IF there has been no secret population-control conspiracy all this time, why has there no effort been shown to remove all mercury from all food? As well as all other toxins from food causing birth defects that lead to social ineligibility to reproduce later in life?

C. Society fought long and HARD against HIV, and we're (slowly) winning that battle; why can't we fight as hard in a battle against Autism by at least isolating catalysts thereof and fighting / removing them?

D. Children of Men depicted a sterile society "due to pollution" and unknown factors. If pollutants cause Autism, why didn't we see many Autistic ("socially sterile") citizens all over the film? --70.179.167.78 (talk) 18:29, 14 October 2012 (UTC)[reply]

The OP has asked a related question here. - Karenjc 19:18, 14 October 2012 (UTC)[reply]
A) You seem to think that the proper way to handle conspiracy theories is to believe them all, unless they can be disproven. This will lead you to believe an infinite number of conspiracy theories. The only reasonable approach is to require proof that a conspiracy exists, before you believe it. This particular conspiracy theory is one of the silliest I've ever heard. Overpopulation is a problem in the third world, not in the developed world, where people have chosen to have fewer children. If there's any conspiracy regarding population, it's the opposite, to ignore the issue of global overpopulation, due to resistance to birth control and abortion. StuRat (talk) 19:24, 14 October 2012 (UTC)[reply]
B) Those "toxins" just aren't the serious issue you think they are. The number of people who die from those is minuscule. Far more serious issues with food are overconsumption of calories, sugar, salt, animal fats, saturated fats, trans fats, and bad cholesterol. StuRat (talk) 19:29, 14 October 2012 (UTC)[reply]
C) If you want a conspiracy theory, the more likely reason for more cases of autism being reported is because the medical establishment can make money off those cases. So, while in former generations a kid might have been viewed as "just shy", the same kid is now given an "autism spectrum disorder" label and medicated. StuRat (talk) 19:33, 14 October 2012 (UTC)[reply]
A) Because there are far better ways of reducing the population than having a small number of people born with a slightly reduced chance of reproducing that also have a high chance of not being productive members of society and being a drain on resources (those ones that are productive members of society will tend to be the ones that do successfully reproduce). If the government wanted to reduce population growth, they would be better off doing things that reduce fertility (and fertility numbers have been dropping over the last few decades, so you can have a field day with that one!).
B) The evidence that these toxins are particulary harmful is pretty thin. In fact, there isn't even good evidence that autism has been on the increase (diagnoses of autism have, but that isn't the same thing).
C) The causes of AIDS are well known - it is a virus (HIV) that is transmitted sexually. That makes it fairly easy to fight. The causes of autism are not at all well known.
D) Because whoever wrote that film didn't think of it? Or perhaps because you are wrong that autists tend to be "socially sterile"? --Tango (talk) 19:45, 14 October 2012 (UTC)[reply]
OP, the Government are too preoccupied with implementing their MMR vaccine stratagem to bother with mercury. Ankh.Morpork 19:53, 14 October 2012 (UTC)[reply]
The major flaw in an autism population-control theory is that autism doesn't cause population control. Autistic people still reproduce. They are not "socially ineligible" except on the furthest end of the spectrum. Similarly, the socioeconomic groups hardest hit by the alleged autism epidemic are those far above the poverty line — those which reproduce relatively less frequently anyway and are not the target of any kind of population control efforts (the government does target populations for reduced family size, but this is usually the poor). In any case, it is not in the US government's interest to reduce the population rate overall — it gets nothing out of that.
None of the logic on this works out: the government is not desperate about reducing the domestic population; autism doesn't affect the classes of people who you'd expect it to if the government was trying to do that (in other words, it doesn't affect the classes of people who are traditionally targeted for population control measures); and autism doesn't affect the fecundity rates at all. --Mr.98 (talk) 21:03, 14 October 2012 (UTC)[reply]
There is no autism epidemic. Its existence is a pernicious lie. Detection rates are improving because awareness is better; the diagnostic criteria have changed over the years, too - and they're changing again next year with the release of DSM-V. But people on the autistic spectrum are not 'socially ineligible to marry', or anything of the sort. I should know; I'm engaged to one.
This is the second crazy 'autism epidemic' question we've had lately. The only thing I've seen an epidemic of is (in the US) prescribing ADHD drugs (which, funnily enough, are a hair's-breadth in chemical terms from being crystal meth) to kids with ASD, or without any neurological difference at all. AlexTiefling (talk) 21:24, 14 October 2012 (UTC)[reply]
Congratulations, great news, mazel tov! μηδείς (talk) 21:53, 14 October 2012 (UTC)[reply]
Thank you! AlexTiefling (talk) 07:04, 15 October 2012 (UTC)[reply]
I admit that, even if I don't agree completely with your logic, you have some valid points. Autism is on the way up and another amazing thing: a teacher who is a friend of mine, told me that governmental agents ask regularly about children who tend to be autistic, and offer early educational packages about several technologies. It's amazing but true, maybe they are not trying to provoke autism to reduce population, but to cause autism to obtain more technology minded children. OsmanRF34 (talk) 21:46, 14 October 2012 (UTC)[reply]
OsmanRF34, you piqued my curiosity, what do you mean by "early educational packages". Thanks for the clarity! Marketdiamond (talk) 08:17, 15 October 2012 (UTC)[reply]
"...some valid points". Which ones precisely? An unsupported assumption that the increase in autism diagnoses equals an rise in incidence of the condition. An unsupported assumption about rhe causes of autism. Some speculative "what if"s and "it could be"s about an unsupported conspiracy theory. And in response, a FOAFtale and conspiracy theory mark 2, and not a reference in sight. This is the Reference Desk. It "does not answer requests for opinions or predictions about future events. Do not start a debate; please seek an internet forum instead." - Karenjc 12:04, 15 October 2012 (UTC)[reply]
The general question - how do we know there isn't a government conspiracy to sicken us? - is a serious question, very much worth asking. Especially when it comes to other governments who might have a grudge against us... Biological warfare is serious business, and there have been credible-sounding accusations of for example the use of thallium, parathion, cholera, ricin, anthrax spores and God knows what else during the civil war in Rhodesia.[22] A lot of these allegations remain difficult to be sure about to this day, despite all the evidence that's come up. The prospect that some other country could be spreading obesity-causing adenoviruses or asthma-inducing toxins or whatever it is that causes autism is far from impossible. I certainly hope that there are real government agencies that make a point of watching for such things. But that shouldn't stop us from asking the question ourselves - how can we know? Wnt (talk) 15:01, 15 October 2012 (UTC)[reply]
The fact that we don't know what causes the vast majority of cases of autism would tend to suggest that no-one is spreading it on purpose. AlexTiefling (talk) 15:14, 15 October 2012 (UTC)[reply]
I don't think that is altogether persuasive on its own. Remember, an unethical regime wouldn't be hobbled by the same ethical considerations as researchers - instead of studying correlations or trying to develop an animal model, they can simply inject 50 pregnant women with "vitamin supplements" and see if a suspected cause is real or not. Or they might work through things the other way, exposing women to various agents to see what happens; a result might be autism or any other disease at random. So we can't be entirely sure that they aren't able to attack us with something we don't know is dangerous, which would make it a lot harder to surveil for.
That said, I do think it is very, very, unlikely. A simple attack would be expected to affect people more in one part of a country, in a potentially recognizable pattern, depending on where a contaminated product was sold. And cases should spike in a narrow window of time when the agent is deployed. Yes, I can picture other scenarios, like if someone has long term control over the plant where cocaine-free coca extract is prepared for Coca-cola, so that they could gradually introduce agents over time that would be distributed to a large part of the world; but such a scenario still leaves them hanging out in the wind, afraid one curious person is going to spot the contaminating compound and wonder why it's there and what effect it has on people. Nonetheless, I'm really speculating here, and I know there must be some serious work being published by experts somewhere on actually looking for such threats. I just want to encourage people to think of how we can find such sources. Wnt (talk) 16:27, 15 October 2012 (UTC)[reply]
How do we "know" with 100% epistemic certainty that the inhabitants of the planet Tralfamadore aren't sprinkling pixie-dust on our breakfast cereal every morning? How do you "know" that you're not the only one who exists? -- AnonMoos (talk) 16:18, 15 October 2012 (UTC)[reply]
Well, I doubt Homeland Security is looking for pixie dust ... but poisons, on the other hand ... Wnt (talk) 16:27, 15 October 2012 (UTC)[reply]

Arlen Specter's middle name

Hi. I've posted my inquiry at Talk:Arlen Specter#Middle name. Any help from the reference desk wizards would be appreciated. :-) --MZMcBride (talk) 20:40, 14 October 2012 (UTC)[reply]

Native Pennsylvanian here and just barely old enough to remember the day prior to Specter talking his Senate seat, for the life of me and I actually did use some nifty Pennsylvania data mines I know of online I have never known him as anything more then Arlen J. Specter, and alas all my data resources failed me, though I did not dig through Philadelphia archives or Google news prior to the 1980's. Stumper, and deceptively so.Marketdiamond (talk) 08:15, 15 October 2012 (UTC)[reply]

Brazil HDI

How does Brazil have a high HDI with stuff like this happening all the time? That's the kind of stuff you'd expect in a war zone, not a developed country. --128.42.221.109 (talk) 22:07, 14 October 2012 (UTC)[reply]

It's actually a developing country. Consider that also the US has problem zones and a violence problem. A country can improve on average, even if that doesn't include all its citizens. OsmanRF34 (talk) 22:21, 14 October 2012 (UTC)[reply]
How does a developing country have a high HDI? --168.7.232.11 (talk) 22:31, 14 October 2012 (UTC)[reply]
There are four stages of HDI: low, medium, high and very high. Brazil has high., which is not a big deal, since it is the 87th country in the world. Mexico is the 57th and Cuba the 51th. Calling it high doesn't mean much, and won't mean that there is not lots of poverty there, it's just that others are even worse off. OsmanRF34 (talk) 00:00, 15 October 2012 (UTC)[reply]
Could somebody explain what HDI is, or link to an article? 69.62.243.48 (talk) 00:03, 15 October 2012 (UTC)[reply]
Human Development Index --168.7.229.36 (talk) 00:17, 15 October 2012 (UTC)[reply]

What do they mean by "He was the last known biological grandchild of renowned physicist Albert Einstein."? Is it mean that he is the last grandson of Albert Einstein that died?184.97.253.165 (talk) 22:36, 14 October 2012 (UTC)[reply]

I would phrase it "Of the biological grandchildren we know of, he was the grandchild of Albert Einstein who died last." StuRat (talk) 23:34, 14 October 2012 (UTC)[reply]
Ugh. "He was the last surviving known biological grandchild of renowned physicist Albert Einstein (relatively speaking)." Back to bureaucratese school for you, Stu. Clarityfiend (talk) 23:51, 14 October 2012 (UTC)[reply]
Hah. Almost the exact same wording is used in "The legacy of Albert Einstein lives on with Dr. Thomas Martin Einstein", although it's not exactly what you'd call a reliable source (a blog). Clarityfiend (talk) 00:06, 15 October 2012 (UTC)[reply]
It looks like old Al only had two four biological grandchildren. Aside from Bernhard, Klaus and two others all died young. The "known" part comes about because we don't know what happened to his daughter Lieserl Einstein. Clarityfiend (talk) 00:12, 15 October 2012 (UTC)[reply]
Albert was quite the ladies' man, and slept with numerous women in addition to his first and second wives. His "string of mistresses" might well have resulted in one or more pregnancies, in the era before the pill. It was not at all a rare or occasional thing. He screwed one mistress twice a week for close to a year, and this was tolerated by his second wife. He might have had biological grandchildren from his mistresses, although I have seen no reliable accounts of same. It would be interesting to see if any likely relatives turned up if his known descendants or other relatives got their DNA analyzed by the genealogical DNA services which now offer DNA analysis of a spit sample for a few hundred dollars and then list all relatives from immediate family to 5th cousins among others who've had the test done. Edison (talk) 16:01, 15 October 2012 (UTC)[reply]

October 15

Characters who can't stand being loved

I recall a professor mentioning a Shakespeare character who couldn't stand being loved, but I can't recall which. Also, can anyone think of any other characters who had similar issues? Thanks. 129.3.150.196 (talk) 00:08, 15 October 2012 (UTC)[reply]

Well, there was King Lear, who got a bit narked off with his youngest daughter Cordelia because she wouldn't express her love for him in the same fulsome terms as his other two daughters. Not sure if that's the kind of thing you had in mind. --Viennese Waltz 09:01, 15 October 2012 (UTC)[reply]
This is from the opening of Richard III (Shakespeare):
And therefore, since I cannot prove a lover,
To entertain these fair well-spoken days,
I am determined to prove a villain
And hate the idle pleasures of these days.
The Moor in Titus Andronicus expresses something similar I believe. μηδείς (talk) 18:30, 15 October 2012 (UTC)[reply]
Don't Beatrice and Benedict have something similar going on before they hook up? AlexTiefling (talk) 21:19, 15 October 2012 (UTC)[reply]

Effects of listening to the same thing over and over

I've been listening to this video for about the past 5 minutes, and I'm starting to feel a bit dizzy...

Are there any documented cases or research regarding listening to annoying sounds over and over again or annoying statements over and over again, and how it would affect one's psyche?

Thanks. --Jethro B 01:55, 15 October 2012 (UTC)[reply]

Binaural beats (as well as Monaural beats, if the proper frequencies can be isolated and made audible through a single channel) can introduce altered states of consciousness via brainwave entrainment under the right conditions, though in most cases I believe it would take longer than five minutes to take effect. You would also probably need to be otherwise unengaged (not surfing the internet, not reading or concentrating on anything in particular with much intensity, etc.), so I'm not sure that explain this case. Evanh2008 (talk|contribs) 02:07, 15 October 2012 (UTC)[reply]
Interesting discussion reminded me of this but it seems that was more visual, then I remembered both the "Suicide causing Song" and the Long Range Acoustic Device, just a comment on how little the average person is aware of these effects.Marketdiamond (talk) 03:31, 15 October 2012 (UTC)[reply]
I remember at the Detroit Auto Show one of the displays had a jingle that repeated continuously. By the end of the show, the employees who worked there looked like the walking dead. StuRat (talk) 05:07, 15 October 2012 (UTC)[reply]
That may just be average Detroiters--or those exposed to Detroit for an extended period. Marketdiamond (talk) 08:11, 15 October 2012 (UTC)[reply]
I was gonna say that spending all day talking about cars would turn anyone into a zombie. Except petrol heads, of course. To each his own. -- Jack of Oz [Talk] 10:45, 15 October 2012 (UTC) [reply]
Well, boring old econobox cars, perhaps. But supercars are always fun. Then there's the pretty models. And, even if they aren't your cup of tea, there's lots of extras, like a dancing fountain display, the new version of WII to try out, etc. StuRat (talk) 16:03, 15 October 2012 (UTC) [reply]
A paper by Warren notes that "Verbal satiation" was described by psychologist Titchener in 1915, wherein prolonged repetition of any common word causes it to lose meaning. It has been a common area of research since the 1960's, See also a recent self-help book which says that when you hear a word like "milk" you associate it with liquid food, but if you say it aloud rapidly for 20 seconds, it becomes a meaningless auditory sensation. The book suggests that one might lessen the hurtful impact of some demeaning term used against the person. Also there is distortion and spontaneous changes when the same thing is heard repeatedly, such as "say" becoming "ace" or "ticktock, ticktock" becoming "toctic. " In Warren's study, repetition of a word for 3 minutes caused the perception to shift to words which were nothing like the actual word. Other books I've read (no handy link) have noted this same loss of linkage between auditory input or vocal output and lexical meaning, like the fatiguing of a neural pathway. I recall the era when carmakers put in a chip which endlessly repeated "The door is ajar" as long as the car sat with the door open, and how it started to fatigue the mind after a number of reps. There have been cases of a bad door switch which caused this phrase to be repeated continuously while the car was being driven. Such repetition seemed like a good candidate for a brainwashing method. (The only recourse was to reply, "No, the door is a door. A jar is where the jam is found.") Car manufacturers seem to have switched to chimes to nag drivers. (Added:) We have an article on this called Semantic satiation, but it is a bit lame. Edison (talk) 15:37, 15 October 2012 (UTC)[reply]
I don't remember that, but it probably explains why there's a large lighted sign saying "YOUR DOOR IS AJAR" in the Major Tom (Coming Home) music video... AnonMoos (talk) 16:04, 15 October 2012 (UTC)[reply]

The abiding memory of my one and only visit to one of the Disney empire's theme parks, was the endless repetition of a song called "It's a Small World", which nearly drove me bonkers. Why they do that, I have no idea. Alansplodge (talk) 17:44, 15 October 2012 (UTC)[reply]

James Kerasiotes

Could you please tell me why the name James F. Carlin does not appear in your report. Mr. Carlin facilitated the original appointment to the Mass Turnpike, via members of his previous administration as Secrectary of Transportation? Thank you. — Preceding unsigned comment added by 76.119.25.121 (talk) 04:52, 15 October 2012 (UTC)[reply]

You should put this on Talk:Massachusetts Turnpike or add it to the Massachusetts Turnpike article yourself, preferably with a source. BTW, who is James Kerasiotes ? Is that you ? StuRat (talk) 05:03, 15 October 2012 (UTC)[reply]
James Kerasiotes is undoubtedly James Kerasiotes. Looie496 (talk) 05:52, 15 October 2012 (UTC)[reply]
OK, then my comments apply to that article and talk page, as well. StuRat (talk) 06:09, 15 October 2012 (UTC)[reply]

Chinese economy

When will China's economy surpass America's? --168.7.237.77 (talk) 05:31, 15 October 2012 (UTC)[reply]

It may not happen. They will run into some rather serious walls, like a demographics bomb due to the One Child Policy, rioting farmers who had their land stolen, environmental problems, and, ironically, they could have a communist revolution from those who resent the wealth being so unevenly and unfairly distributed to those who know influential Party members. StuRat (talk) 05:38, 15 October 2012 (UTC)[reply]
Then what country will be the first to beat the US? India? Brazil? --168.7.234.107 (talk) 05:54, 15 October 2012 (UTC)[reply]
Could be China, India, Brazil, or perhaps the EU will be considered a single nation at one point. StuRat (talk) 06:08, 15 October 2012 (UTC)[reply]
Until it actually happens, it is certainly true that it may not happen (the entire country may be wiped off the map by an asteroid tomorrow). It would take something pretty big to prevent it, though. The demographic bomb you speak of won't act fast enough - we're talking 5-10 years, not a generation. Environmental problems aren't going to do much on that timescale either. Some kind of revolution that decimates the economy is certainly a possibility, but there aren't really any signs of that happening in the near future. In short, we can be pretty sure China's economy will overtake America's - the difference in population is so much that it has been essentially inevitable for a long time. The only question is when and, once you've agreed on a metric, there isn't even much of a question there. One or two percentage points difference in either the US or Chinese growth is only going to change the estimate by a couple of years or so. --Tango (talk) 20:48, 15 October 2012 (UTC)[reply]
This chart from The Economist estimates that the Chinese Gross Domestic Product will surpass that of the United States (as measured at market-exchange rates) in 2018, if that's what you mean. There are, of course, lots of other measures of "an economy" – several of which are detailed there. Gabbe (talk) 06:52, 15 October 2012 (UTC)[reply]
As for other countries than China surpassing the US, looking at "List of countries by past and future GDP (nominal)", it seems that there are estimates saying that the second country to surpass the US in this sense will be India. According to estimates, India might have a greater GDP than the US sometime in the 2030s. But projections so far ahead should be taken with a grain of salt. Gabbe (talk) 07:04, 15 October 2012 (UTC)[reply]
IMF's projections puts it at 2017.A8875 (talk) 07:12, 15 October 2012 (UTC)[reply]
Think some say that China has already surpassed the USA. Using purchasing power parity, and depending on how you measure rural production and living standards, iirc. There's close to zero doubt that it will surpass by increasingly many measures. The Economist lists some already achieved, and should be just about all, pretty soon.John Z (talk) 07:26, 15 October 2012 (UTC)[reply]

Characters at the top of this here picture

וחוח, or something?

The picture at right is an illustration of the 21st article of the Augsburg Confession, which relates to belief in the doctrine of sainthood. As you can see, there appear to be four designs at the top of the picture, amidst the clouds. As far as I can tell, the characters are an attempt to reconstruct the Tetragrammaton by someone unversed in the Hebrew alphabet. Since I can't tell quite how that relates to the doctrine in question, though, and since that first letter looks nothing like any yod I've ever seen, I feel compelled to doubt my first instinct. Does anyone have a better idea? Evanh2008 (talk|contribs) 07:23, 15 October 2012 (UTC)[reply]

Educated guess here emphasis on "guess", it appears like some kind of hebrew or derivation, very similar to the Chai (symbol) just repeated. Marketdiamond (talk) 08:04, 15 October 2012 (UTC)[reply]
So instead of one yod that doesn't look like a yod, you see two yods (yodim?) that don't look like yod? It says pipi in Greek (πιπι).Tamfang (talk) 08:06, 15 October 2012 (UTC)[reply]
In Hellenistic or Roman Empire times, when some Greek-speakers without knowledge of Hebrew sometimes tried to make use of Jewish names for mystical or occult purposes, YHWH written right-to-left in the Hebrew alphabet was occasionally misunderstood as PIPI written left-to-right in the Greek alphabet. However, I really don't think this would apply to Renaissance pious illustrations, and the artist would not have drawn the characters with wavy outlines if he had intended to show the Greek alphabet... AnonMoos (talk) 14:26, 15 October 2012 (UTC)[reply]
File:YHWH.png
William Gesenius's Hebrew punctuation (i.e., Yahweh)
I think your first instinct was right. Specifically, it looks like they substituted in Greek lowercase letters, pi and tau. I can't explain the split vertical bar, though. StuRat (talk) 08:09, 15 October 2012 (UTC)[reply]

I'd go with Evan's suggestion it's an attempt at the Tetragrammaton by someone unlearned. I've seen similar on Judaica items from the Far-East, where people simply draw what they see, not knowing the difference between significant and insignificant elements. I'd guess the split bar of the vav is due to uneven printing in the source material used. --Dweller (talk) 13:32, 15 October 2012 (UTC)[reply]

Go to commons:Category:Tetragrammaton in Christian art and its subcategories, and you can see plenty of similar renderings... AnonMoos (talk) 14:18, 15 October 2012 (UTC)[reply]
In my view, the artist clearly knew the difference. The yod and the vav are drawn differently.--Wehwalt (talk) 16:03, 15 October 2012 (UTC)[reply]
However the yod seems to be drawn more like a zayin than a yod... AnonMoos (talk) 16:08, 15 October 2012 (UTC)[reply]
And the Heys are Chets. The artist didn't have a clue. --Dweller (talk) 18:34, 15 October 2012 (UTC)[reply]

Some context might be in our article on Christian Kabbalah, the mystical interpretation of Hebrew characters in a Christian context, which came to prominence during the Renaissance. Not listed in the article is our man John Dee, who believed that encoded in the Hebrew alphabet was the sacred language of the angels; which once mastered, would reveal all the mysteries of the universe. Alansplodge (talk) 17:03, 15 October 2012 (UTC)[reply]

I'm not really seeing much hard data here - because Yodh links to all sorts of very different scripts, is there a chance this is just some other script not mentioned by Wikipedia? Wnt (talk) 19:57, 15 October 2012 (UTC)[reply]
The only ones known to Europeans ca. 1650 would have been Hebrew, Syriac, Arabic, and certain quasi-occult derivatives of Hebrew, such as Transitus Fluvii, Celestial Alphabet, Enochian alphabet, and possibly the Theban alphabet. I really don't know why a Christian artist producing a solemn exposition of the highest truths of religion would have used any of them except Hebrew... AnonMoos (talk) 22:23, 15 October 2012 (UTC)[reply]
There are several tetragrammata in the Hollar collection allegedly all from the same artist, Protestant, Catholic and neuter, German, Latin and English? --Pp.paul.4 (talk) 21:39, 15 October 2012 (UTC)[reply]

Pro-choice activism

Hi, I don't want to debate political or ethical questions. Does anyone know a good printed manual/guide to pro-choice-activism? Thank you for your help. --Desir usrn (talk) 13:38, 15 October 2012 (UTC)[reply]

Planned Parenthood might be a good place to start, at least in the US: http://www.plannedparenthoodaction.org. I didn't see a manual there, but they might be able to direct you to where you can get one. StuRat (talk) 16:08, 15 October 2012 (UTC)[reply]
Or NARAL Pro-Choice America. Duoduoduo (talk) 23:28, 15 October 2012 (UTC)[reply]

International Joint Commission

How does one get on the International Joint Commission? Are the Americans appointed just like people are appointed to independent U.S. government agencies? I don't even have a clue on how the Canadians get appointed. If you have a reference, could you expand the article? 2001:18E8:2:1020:749C:5B76:1D8E:3D22 (talk) 15:59, 15 October 2012 (UTC)[reply]

The single reference given in the article indicates that the Canadian commissioners are appointed by the Department of Foreign Affairs and International Trade and the American commissioners are appointed by the President. Regards, Orange Suede Sofa (talk) 20:01, 15 October 2012 (UTC)[reply]

Information about World War II massacre

I visited the Imperial War Museum the other day and saw description within their holocaust exhibition of a massacre that occurred against Jews in one of the Baltic states, I think Lithuania. It was accompanied by a photo of a man 'proudly' holding a club with a large number of bodies strewn around him, and a crowd watching. The caption said something like 'Massacre perpetrated by the Clubber of Vilnius and assistants, ...' (although I'm not sure if the city was Vilnius). The photo, caption, and use of a grisly moniker made it seem like one man was largely responsible for the killings. I was wondering if anyone could give me the name of this event / person, or knows the photo. Thanks. --82.13.141.22 (talk) 17:19, 15 October 2012 (UTC)[reply]

"In Kaunas, Lithuania’s second city, Jews who had just been released from prison were clubbed to death by a local ‘patriot’ known as the ‘Death Dealer’, who after he had killed them, climbed on the bodies and played the Lithuanian national anthem on an accordion." WW2History.com - 22nd June 1941 - Einsatzgruppen killing squads start. There may have been other, similar events though. Alansplodge (talk) 17:55, 15 October 2012 (UTC)[reply]
Is this the picture in question? If so, there is more detail of the event at HITLER'S FOREIGN EXECUTIONERS: EUROPE'S DIRTY SECRET which occurred at the Lietukis garage on 25 June 1941. Further detail at Einsatzgruppen A: The Massacres in Kovno - Reports and Eyewitness Accounts ("Kovno" was the English language name for Kaunas at that time). Perhaps brief details of this event might be added to our article , which rather blandly mentions that Jews were "attacked". Alansplodge (talk) 18:05, 15 October 2012 (UTC)[reply]
Yes this is what the display was referring to and I'm sure that's the right photo. I was beginning to think I'd got a major detail wrong as no amount of googling was finding it. Thanks very much. --82.13.141.22 (talk) 20:09, 15 October 2012 (UTC)[reply]
It was a bit tricky - searching for "Clubber of Vilnius" brought up lots of details about night clubs in the town - it seems to be a major venue for British stag parties. Alansplodge (talk) 20:38, 15 October 2012 (UTC)[reply]

Most library-dense cities

I have a personal theory that Washington, D.C. has more libraries per capita of any city in the world. Besides the Library of Congress (plus the NAL and NLM, if we're talking metro area), most federal departments and agencies have their own libraries, there's a large public library system, and many public and private universities and K-12 schools with libraries. Add in private companies, especially law firms, who maintain their own libraries and you can see where I got my theory. But similar situations could certainly exist in other national capitals. Any hard data out there? (I generally phrase this as "libraries per capita," but I'd also be interested in library counts per area). --BDD (talk) 20:30, 15 October 2012 (UTC)[reply]

If you count law firms, do you count private libraries? I have a about 1000 books at home - does that make my flat a library? --Stephan Schulz (talk) 22:45, 15 October 2012 (UTC)[reply]
No, it's more a question of workforce. I should clarify and say professionally staffed libraries. --BDD (talk) 02:14, 16 October 2012 (UTC)[reply]
The only thing that, a priori, distinguishes Washington D.C. from any other major city of the US in terms of the number of libraries lies in the presence of the federal government. Most other major cities also have many government departments, many law firms, many universities and many schools. Counted that way, the extra few national libraries is probably not so significant.
If you are looking for largest numbers of libraries per capita by city, my guess would be large, library-dense college / university towns. Oxford, for example, with a population of 150,000 or so, has 40 libraries in the Bodleian system, 38 colleges with at least one library each, 10 or so municipal libraries, plus two dozens or so other schools and higher education institutions, which would have at least one library each. That's more than 100 libraries without counting private or corporate libraries. --PalaceGuard008 (Talk) 12:32, 16 October 2012 (UTC)[reply]
I'm not so sure. Yes, DC has a few federal libraries — but only a few, assuming you are counting only those that the public has access to. It has some universities and public libraries, but it is not an especially large city in terms of population or geographic size (only ~68 square miles). I would wonder if someplace like Los Angeles, California, which contains many more people and a much larger geographic area (~467 square miles), wouldn't have a larger count just on the basis of public libraries alone (but not probably per capita), much less additional small colleges and universities. If one is talking about number of unique volumes, DC probably does handily along with a few other university towns (e.g. Cambridge, Mass.), but in terms of a raw number of libraries, I'm unsure. Just a raw count of public libraries within their city borders, DC has 26, LA has has 46. LA has maybe 11 universities within its city borders, DC has 24, though I don't know how open to the public most of those are (they are weird little graduate institutions for the most part). --Mr.98 (talk) 15:11, 16 October 2012 (UTC)[reply]
I'm not just interested in libraries that the public has access to, though. So when I say federal libraries, I don't just mean the LC, NLM, and NAL. The EPA has a library, the Department of Education does, the FTC does—I imagine most federal departments, agencies, and commissions do. Oxford and Cambridge (UK) must be up there, though, with all their colleges. How about London? --BDD (talk) 17:30, 16 October 2012 (UTC)[reply]
If we are excluding private libraries (law firms, large homes, etc), I think I can give a relatively precise estimate for Oxford. This brochure lists 55 university libraries (which includes two of the colelge libraries (All Souls and Nuffield), 11 other libraries (which includes municipal and most of the other non-OU higher educational institutions). To this we can add 36 college libraries (i.e. excluding the two already counted), which makes for 102. As to primary and secondary schools, we don't have a list of schools in Oxford itself, but there is a List of schools in Oxfordshire. Looking only at those schools which are labelled as being in Oxford (or as "Oxford [x] School"), there are 15 schools here. So the total is 117. This is a conservative estimate, as (1) the Bodleian brochure first cited counts libraries by location rather than institution, so all of the social science libraries are counted as one, and (2) we have only included those primary and secondary schools which are labelled as being in Oxford in some way. Based on 150,000 people, that makes for 1 library per 1282 people.
Having gotten that result, I think I can immediately beat myself. The City of London, the historical centre of London, has a population of only 7,000. Yet it has 5 municipal libraries, 6 primary and secondary schools, and 7 higher educational institutions. That makes for at least 18 "public" libraries. That's 1 library per 390 people or so. --PalaceGuard008 (Talk) 18:28, 16 October 2012 (UTC)[reply]
Obviously, there should be a small city somewhere with just 1 library and the highest number of libraries/inhabitants. OsmanRF34 (talk) 23:45, 16 October 2012 (UTC)[reply]

historic total US employment statistics

I can readily find out what the unemployment rate is. However, I don't care. I'd like to know haw many Americans were actually working in a given year, and how many working 5 years later. I can't find these statistics. Why not?? — Preceding unsigned comment added by 76.188.144.228 (talk) 20:50, 15 October 2012 (UTC)[reply]

Actually, the Bureau of Labor Statistics has the very thing you are looking for. Here you can search for employment numbers by year, or look at total average hours worked or any other number of statistics. Its quite comprehensive. Livewireo (talk) 21:26, 15 October 2012 (UTC)[reply]

Thank you! I am familiar with the BLS website, but it is a nightmare to navigate and find what I'm looking for. This helps. — Preceding unsigned comment added by 76.188.144.228 (talk) 13:04, 16 October 2012 (UTC)[reply]

Some interesting information can be extracted from the IRS statistics. E.g., in 2009 tax year, 116 million individual tax returns showed some wage/salary income (out of the total of 149 mln returns filed for that year). Unfortunately, this cannot be directly mapped to the number of individuals who received forms W-2, because some of these 116 mln tax returns are joint tax returns with both spouses receiving wage income. Still, these stats may be interesting for things such as year-on-year changes, or the distribution of wage income by income level. -- Vmenkov (talk) 18:50, 16 October 2012 (UTC)[reply]

meditating

I have been trying to get into meditating for the past few months but I don't think I am handling it well. I've looked online, but I am not sure which is a good site or not. Anybody have any tips or websites I could go to to help with it. Any help would be appreciated 152.27.56.87 (talk) 22:49, 15 October 2012 (UTC)[reply]

This, [23], is a good, short introduction to Zen style meditation; a great deal more in depth in the book Finding the Still Point. Another good book, I think, is The posture of meditation, which focuses on posture and not any particular tradition—that is, it's applicable to basically any tradition. Better than books and websites would be finding a group of people who meditate together from whom who can get personal instruction and feedback. Pfly (talk) 01:11, 16 October 2012 (UTC)[reply]
There are many different kinds of meditation, but they are all easier if you can go along to a class for personal tuition. But if you can't get to a class I recommend this website.--Shantavira|feed me 07:41, 16 October 2012 (UTC)[reply]


Thanks. Aorund where I live there isn't that much I can see where I could go so thanks.````152.27.56.60 (talk) 19:50, 16 October 2012 (UTC)[reply]

Eligibility for voting on the Scottish independence referendum

Among others eligible to vote on the Scottish independence referendum, 2014 are:

  • British citizens resident in Scotland

and

  • members of the House of Lords resident in Scotland

Are members of the House of Lords not British citizens? 69.62.243.48 (talk) 23:01, 15 October 2012 (UTC)[reply]

House of Lords#Qualifications says: "Furthermore, only citizens of the United Kingdom, Commonwealth citizens, and citizens of Ireland may sit in the House of Lords. The nationality restrictions were previously more stringent: under the Act of Settlement 1701, and prior to the British Nationality Act 1948, only natural-born subjects were qualified." PrimeHunter (talk) 23:07, 15 October 2012 (UTC)[reply]
It is a redundant item in that list, but presumably refers to the fact that members of the Lords do not have a vote in elections of members of the House of Commons. Therefore the referendum legislation will presumably make it clear they do qualify - just for certainty. Sussexonian (talk) 23:31, 15 October 2012 (UTC)[reply]
It is not a redundant item. Commonwealth citizens and citizens of Ireland are not per se British citizens. --PalaceGuard008 (Talk) 12:18, 16 October 2012 (UTC)[reply]
I believe Sussexonian was referred to the list cited by the OP, not the one quoted by PrimeHunter. The indentations and the meaning both point that way. "Members of the House of Lords" is redundant in a list that already has "British citizens" (with identical restrictive phrases), but is a useful clarification because of the ineligibility of Lords members to vote in Commons elections, as Sussexonian correctly observes. AlexTiefling (talk) 12:23, 16 October 2012 (UTC)[reply]
Not that I expect the number of Lords resident in Scotland will decide the vote, but if there was a "yes" vote would Scotland automatically be a Commonwealth country? In other words would voting "yes" be voting themselves out of a job? -- Q Chris (talk) 12:28, 16 October 2012 (UTC)[reply]
Oh, wait. My apologies, PalaceGuard008 - I now see the group whose inclusion is in question. I have no idea whether there are any Members of the House of Lords who are resident in Scotland but citizens of countries other than the UK. If there are any, I suspect the group to be very small indeed. I have no idea whether the proposed criteria are intended to include or exclude them. As the dissolution of the Union would not occur immediately upon a 'yes' vote, but take place over a period of time following, the questions of Commonwealth membership and of the eligibility of Scottish citizens to be Anglo-Welsh peers would be decided during the negotiations. There are still peerages extant from the Peerage of Scotland (pre-1707); however, none of them intrinsically grant the right to sit in the House of Lords these days. As it is the stated intention of the SNP that the present monarch should remain head of state, and Scotland remain a monarchy, would a Scottish Peerage be re-established? AlexTiefling (talk) 12:40, 16 October 2012 (UTC)[reply]

October 16

William J. Clinton

I need help finding out the names of, and how many boards of directors Bill Clinton serves on.

The wiki article on him does not have this information, and neither does GOOGLE!

This is not homework. RJSEAB (talk) 00:19, 16 October 2012 (UTC)[reply]

They may know at the William J. Clinton Center. Here is their "contact us" page. I would try there first. --Jayron32 02:22, 16 October 2012 (UTC)[reply]

Out-of-state employment discrimination laws in the US

The biggest thing keeping me from coming out openly is that I work for a company in a small conservative town, and the state and local anti-discrimination laws don't offer protection for discrimination based on sexual orientation. However, my company is owned by a company in a state with better anti-discrimination laws. My paycheck comes from the parent company, but come tax time the income counts as from my state. I'm trying to figure out if the state law of the parent company offers me protection. I know you can't provide legal advice, but I'm looking for resources or groups that can help me figure out the answer. I contacted the Human Rights Campaign earlier today, because they are very active in campaigning for LGBT anti-discrimination laws. Does anyone else know of any organizations I could contact that would be able to help answer my questions? 108.194.140.240 (talk) 01:48, 16 October 2012 (UTC)[reply]

What state is the parent company located in? Someguy1221 (talk) 01:54, 16 October 2012 (UTC)[reply]
You might start with the company itself: What is its official policy on this subject, if any? If they are openly/officially anti-discriminatory, maybe their legal departmen could offer some advice. If not, you'll probably want to keep it to yourself within the company, and seek counsel elsewhere. This is a shot in the dark, but it occurs to me that Planned Parenthood is a pretty open-minded organization. You might call them and ask they can direct you to any good organizations. ←Baseball Bugs What's up, Doc? carrots01:57, 16 October 2012 (UTC)[reply]
The parent company is in Wisconsin, and the company I work for is in Michigan. My company has nothing about sexual orientation in their anti-discrimination policy, and I don't know where to find the parent company's policies. 108.194.140.240 (talk) 02:11, 16 October 2012 (UTC)[reply]
Not being anything resembling a lawyer, I may be totally wrong on this, but I can't imagine a scenario in which the laws of Wisconsin would hold any sway in Michigan, or vice versa. That's "state sovereignty". Now, if Michigan's discrimination violates a federal law, that could be a different story. Back to your company: As you're an employee, maybe you could approach H.R. and ask to see or to have a copy of the policy manual... to see what it says about various kinds of discrimination that they don't allow. If they ask why you want it, don't give away the game, but make up something vague and noncontroversial, until you've had a chance to read the part you're interested in. ←Baseball Bugs What's up, Doc? carrots03:50, 16 October 2012 (UTC)[reply]
My experience (having been fired for being queer, and having been refused as a candidate for coming out to the hiring agent) is that your employer will find whatever excuse they want or need to fire (or not hire) you on whatever grounds. Non-discrimination laws largely apply for wrongful termination. Are you trying to get yourself fired? If not, why do you need to come out to your employer as such? You can tell your co-workers whatever you like. Your employer won't ask you diddly, unless they are looking to get sued. Not that I am offering you legal advice, just personal opinion based on unfortunate experience. μηδείς (talk) 04:49, 16 October 2012 (UTC)[reply]
I'm getting to be good friends with my immediate coworkers, and would like to be able to be open about it with them. There have already been a few times where I almost said something that would give it away and caught myself. The problem is that I have no idea what they think of homosexuality, and if one will mention the "problem" to anyone else. Word gets around fast. The company is also pretty large and the town is small enough that I have met friends outside of work that end up having a connection back to the company. One friend turned out to the daughter of one of the higher-ups. 108.194.140.240 (talk) 10:58, 16 October 2012 (UTC)[reply]

I need to mention this question asks way too specific a factual question, and probably should not be answered because that would be legal advice. Shadowjams (talk) 08:28, 16 October 2012 (UTC)[reply]

I'm not looking for an answer to the question here, but for help finding groups that can answer the question. I know they exist, and I would prefer to not have to go to a lawyer with the question if there is a group that has experts on the subject that exists to help people in situations like mine. 108.194.140.240 (talk) 10:58, 16 October 2012 (UTC)[reply]


There is a list of such organizations on this page. Here is a similar list. Equality Michigan is a referral service for anyone with questions such as yours. Taknaran (talk) 10:59, 16 October 2012 (UTC)[reply]

Now that I've thought about it for a bit, I'm thinking of contacting the parent company to see their anti-discrimination policy. Odds are it mentions sexual orientation because it is in the state law. If it does, I'll see if I can get them to work on making sure that all of their companies adopt a similar policy. 108.194.140.240 (talk) 00:13, 17 October 2012 (UTC)[reply]

Marbles and Jews

What in detail is the connection between Judaism and marble playing? This early WP essay asserts one [24] and there is a symbolic link between the the Jewish orphan of S01E03 of the new Upstairs, Downstairs (remake) and her mother in the previous episode. μηδείς (talk) 03:45, 16 October 2012 (UTC)[reply]

Err well I doubt one exists, Atlas Shrugged was a novel and at that time, marble playing was fairly common so it could be why it was included. Today, it's generally less common, so I don't know how relevant it would be. I don't know if there's any connection though, there certainly isn't a religious one... --Jethro B 04:10, 16 October 2012 (UTC)[reply]
The author of the essay, Adam Reed, is (or was) a well educated Hungarian Jew, the name is a pseudonym he adopted when he emigrated to the US to work for Bell Labs. He seems to believe there is at least a cultural a connection:

"Francisco Domingo Carlos Andres Sebastian D'Anconia sat on the floor, playing marbles." Playing a child's game on the floor is a celebration of the Maccabee revolt. The Maccabees were guerillas, and chose carefully the few battles they fought. They hid for years in caves and wadis, and fought boredom with games played on the ground. The reader who knows the origin of this custom now also knows that Francisco must be, in some sense, the soldier of a rebellion, fighting from cover behind enemy lines.

and the U/D episodes seem to back him up. Atlas Shrugged itself is irrelevant here, so far as I am aware, except that Rand herself was a Russian Jew. μηδείς (talk) 04:32, 16 October 2012 (UTC)[reply]
Maybe marbles is seen as being similar to the game played with a Dreidel. Bus stop (talk) 04:44, 16 October 2012 (UTC)[reply]
(Unrelated question: How do I get rid of the "SAM COHEN IS A JEW!" at the top of the article I Have a Little Dreidel?) Bus stop (talk) 04:52, 16 October 2012 (UTC)[reply]
(Done μηδείς (talk) 04:59, 16 October 2012 (UTC))[reply]
How did you do it? Bus stop (talk) 05:03, 16 October 2012 (UTC)[reply]
At the top of the vandalized article was a {{link}} to another article, which traced to another embedded article, to another embedded article, eventually to Template:JewishMusic which had the offending text. I deleted that text from the multi-embedded article. μηδείς (talk) 05:11, 16 October 2012 (UTC)[reply]
Great. Thanks. Bus stop (talk) 05:12, 16 October 2012 (UTC)[reply]
The dreidel/marbles connection is actually part of Reed's essay, which was an early contribution to WP. But I am hoping for some independent confirmation of it. (I am loath to reproduce his entire essay, although he did give me permission to do so. People should read the link I provided in the first line and second sentence of this thread.) μηδείς (talk) 04:56, 16 October 2012 (UTC)[reply]

The essay answers the question:

In Jewish custom, one commemorates one's ancestors by re-enacting their practice. Francisco's Converso ancestors could not keep Jewish objects, such as spinning tops with Hebrew letters, since such objects would have immediately marked them as secret Jews. They played, instead, with marbles unstrung from a rosary. If a curious neighbor or servant wandered in, all he would see would be an accidentally broken rosary, and children helping to pick beads off the floor. Francisco is hiding his true self, and things are not what they seem.

The author is suggesting that some conversos / crypto-Jews (sometimes referred to as marranos, which is a perjorative term and should be avoided) adopted playing marbles as a seemingly innocent way of referencing the custom of playing dreidel on Chanukah, as mentioned by other editors, above. I don't know how common such a thing might have been among conversos, but pretty much any converso behaviour is rare today among Jews - (OR time) anecdotal evidence I've heard down the years suggests that some crypto behaviours, such as women lighting candles in a cupboard on a Friday persists, but among people who do not consider themselves Jewish. More information can be found in the links I've given. I've not seen the TV episode you mention, but from the scant information in the question, I'd guess it's simply a case of a kid who happens to be Jewish playing marbles, a fairly popular game in Victorian Britain for Jewish and non-Jewish kids, alike. See Marble_(toy)#History. --Dweller (talk) 10:38, 16 October 2012 (UTC)[reply]

Marbles was still a very common children's game in London in the 1960s. It's normally a game for at least two players, but children are very good at working around problems like this; I recall as a child playing chess by myself a few times. Every child I knew had marbles in their toy box and boys would often carry them around in the pockets of their school shorts. Unusual ones would be the subject of playground swaps. You could buy a bag of a dozen glass ones (each with a coloured twist inside) for a shilling (5p) in Woolworths. I think that I'd have been rather surprised to be told that playing marbles was enacting a Jewish ritual. Alansplodge (talk) 13:19, 16 October 2012 (UTC)[reply]

Iran and homosexuality

I have a question regarding homosexuality and Iran, is a person liable to the death sentence just for being gay or for having gay sex? Thank you. Iowafromiowa (talk) 11:00, 16 October 2012 (UTC)[reply]

See LGBT_rights_in_Iran#Capital_punishment. 'Being gay' is not forbidden in any country. Performing homosexual acts may be punishable by death however, as seems to be the case in Iran. - Lindert (talk) 11:28, 16 October 2012 (UTC)[reply]

Homosexuality in the Old World before the spread of Judaism and Christianity

It's a well-known fact that homosexuality was widespread in the Roman Empire. In fact, it is believed that some Roman emperors were gay. However, ever since Christianity spread, homosexuality has been a sensitive topic at least in the Western world. However, before the spread of Judaism and Christianity, just how widespread was homosexuality in the Old World (Europe, Asia and Africa), and what were the views of the different societies and religions at the time on it? And before the Abrahamic religions were founded, were there any major societies in the Old World or religions that frowned upon homosexuality? And what were the contemporary non-Abrahamic religion societies that frowned upon homosexuality in the Old World? I'm sure there's a Wikipedia article about these topics but I'm not so sure. Narutolovehinata5 tccsdnew 11:28, 16 October 2012 (UTC)[reply]

I think it's important to distinguish between homosexual orientation and homosexual attraction or practice. As a bisexual person, I experience the attraction (and may engage in the practice) without possessing the orientation. Conversely, someone who is celibate but gay may possess the orientation and identification, but rarely experience the attraction and never engage in the practice.
Beyond that, it's crucial to note that the modern understanding of sexual orientation was devised by researchers and campaigners in the 19th century (Kertbeny, Ulrichs, Kraft-Ebbing &c). The classical world had no conception of orientation as we understand it. AlexTiefling (talk) 11:57, 16 October 2012 (UTC)[reply]
You should probably be cautious in assuming that people in ancient times who at least sometimes participated in male-male sex had a real "gay" self-identity in the modern sense. Most male-male relationships in Greco-Roman civilization fell into a specific middle-aged man and teenage boy pattern, which was not considered incompatible with heterosexual relationships. When St. Paul wanted to refer abstractly to those who did male homosexual sex (without getting involved with the specifics of the middle-aged man vs. teenage boy roles), he seemingly had to coin a new word on the spot ("arsenokoites"). Anyway, one reason why homosexuality was particularly disliked in the Old Testament seems to have been that a form of male prostitution was part of the ritual associated with a pagan polytheistic deity. Probably many ancient cultures were like many cultures described by anthropologists, which have certain rituals or roles (such as "third genders") which can involve homosexual sex, without really having a clear idea of gay identity as we would think of it today...AnonMoos (talk) 12:07, 16 October 2012 (UTC)[reply]
In case you haven't seen them, we have articles on history of homosexuality (with regional sections) history of human sexuality, and LGBT history. SemanticMantis (talk) 16:30, 16 October 2012 (UTC)[reply]

U.S military commander who - erroneously - believed that Israel couldn't withstand the Arabs alone prior to the upset that became the Six-Day War

Any ideas? It was around the time of the Baghdad Pact. Eisenikov (talk) 17:44, 16 October 2012 (UTC)[reply]

Most of them? Needs more detail to disambiguate. —Tamfang (talk) 19:06, 16 October 2012 (UTC)[reply]

Im not sure what you're referring to exactly. In 1967, the CIA believed that Israel would be able to defeat the surrounding Arab countries if attacked, saying that Israel could "defend successfully against simultaneous Arab attacks on all fronts . . . or hold on any three fronts while mounting successfully a major offensive on the fourth." Source = CIA

I haven't looked at the source enough, but I'm almost confident that the CIA also estimated that if Israel launched a preemptive strike against the buildup of Egyptian military force in the Sinai Peninsula after Egypt forced the peacekeeping force to leave, it would take them less days to win, while Israel would still be able to win had they not done this, but it would've taken more days (although a few days for a country with a small population can lead to significant civilian losses...)

Hope this helps! --Jethro B 23:09, 16 October 2012 (UTC)[reply]

Foreign language interpretation

Why is it on news reports when foreign language is contemporaneously interpreted with a voice-over, the interpretor always concludes slightly before the foreign speaker finishes speaking? Case in point. Ankh.Morpork 18:06, 16 October 2012 (UTC)[reply]

If this happens, it is one indication that the translation is not happening in real-time but was dubbed over the footage. Voice-over translation explains a bit of the process as used for news broadcasts and this academic article discusses how sometimes translations are tailored to suit the show’s thesis or agenda. Taknaran (talk) 18:32, 16 October 2012 (UTC)[reply]
Another reason could also be that a person, when answering a question spontaneously, uses a lot of fillers, such as 'umms' and 'ehhs', repeats what he has just said, or stops himself mid-sentence, to rephrase his point into a cogent sentence. The translator (assuming that the translation and dubbing was done between the interview and the boradcast) doesn't have to do this, but merely read the the translation. V85 (talk) 19:49, 16 October 2012 (UTC)[reply]
I once heard an anecdote told me by an EU translator in a bar in Brussels: he was waiting to take over from someone translating from German into English, and he heard him start a sentence, then pause, then a longer pause, then an anguished cry "The verb! Just say the frigging verb man!" (German is notorious for having the verb at the end of the sentence) --TammyMoet (talk) 20:54, 16 October 2012 (UTC)[reply]
Lovely story, Tammy, and one that's had many incarnations. I once heard Peter Ustinov telling it about an episode in the General Assembly of the UN, where the interpreter's long silence while waiting for the German Ambassador to say the verb caused various delegates to turn around to see if the interpreter needed medical help. He was indeed frustrated to the point of his brain exploding and his blood boiling, but was otherwise OK. -- Jack of Oz [Talk] 22:22, 16 October 2012 (UTC) [reply]

How to get the details on the Romney-Ryan tax plan?

close pointy non-question
The following discussion has been closed. Please do not modify it.

I tried this site:

http://www.romneytaxplan.com/

but it doesn't work :( . Count Iblis (talk) 18:27, 16 October 2012 (UTC)[reply]

Fake question. Please do not disrupt Wikipedia to make a point (even when it's a good one). — TransporterMan (TALK) 19:14, 16 October 2012 (UTC)[reply]
WP:Spoilsport (check later when I've written this up). Count Iblis (talk) 19:31, 16 October 2012 (UTC)[reply]
No need to wait, ironically I did it three years ago: {{The Spoilsport Award}}. Who would have thought... Regards, TransporterMan (TALK) 20:19, 16 October 2012 (UTC)[reply]
If you follow the link it amounts to a practical joke. This should probably just be deleted. μηδείς (talk) 21:08, 16 October 2012 (UTC)[reply]

List of most corrupt Congressmen

A political ad for a local US Congressional seat claims that the incumbent is on "the list of the most corrupt Congressmen". Is there such a list? Where can I find it? 69.62.243.48 (talk) 21:32, 16 October 2012 (UTC)[reply]

Googling this term mostly leads to an annual list by the "watchdog group" "Citizens for Responsibility and Ethics in Washington". It seems the latest edition was released about a month ago. 86.166.186.159 (talk) 22:00, 16 October 2012 (UTC)[reply]

Techno-religion

Is there a name for such cults with fringe ideas like how cryonics/cold fusion/AI/or space travel will save humanity? This kind of cults seem to be a separate category for me, with a technology as the meaning of life, in the same way that Abrahamic religions have God.OsmanRF34 (talk) 21:32, 16 October 2012 (UTC)[reply]

Transhumanism (and the associated technological singularity), while formally not a religion, has been described as the "rapture of the nerds". Technopaganism is a religion, but is less utopian and more spiritual. -- Finlay McWalterTalk 22:09, 16 October 2012 (UTC)[reply]

When did the English monarchy lose their power?

When did the King/Queen of England become a mere figurehead? ScienceApe (talk) 22:57, 16 October 2012 (UTC)[reply]

Power was lost throughout the 17th century. It wasn't, as has been suggested, on the restoration of Charles II. His brother, James II tried to emulate his father (and Louis XVI of France) and become an absolute monarch. he didn't succeed and lost his throne. William of Orange was offered the throne in his stead and Parliament began to take control. After the death of the last of the Stuarts, Queen Anne, the throne was handed to the Elector of Hanover who became George I and Parliament passed the Act of Settlement defining how the crown would descend from then. Throughout the 18th century, parliament as we now know it, with a Prime Minister and his Cabinet gradually became the norm and accepted. George III tried to influence his ministers, but by the time of his illness and the need for the Regency, followed by the reign of George IV parliament was completely in control and the monarch no more than a figurehead. OsmanRF34 (talk) 23:21, 16 October 2012 (UTC)[reply]
(ec) The transformation in England towards the modern constitutional monarchy in the United Kingdom was gradual. You could say that the development of "Parliament" paralleled the growth of constraints on the monarch's pwoer. The Magna Carta is often pointed to as a symbol of the king's acceptance of such constraints. Chopping off the king's head may also have been an important event. Our article constitutional monarchy cites the Glorious Revolution of 1688 as establishing the modern constitutional monarchy, but of course the monarch's powers continued to shrink after that. It is still hard to say whether the Queen is a mere figurehead even today - our Constitution of the United Kingdom article points out that the Queen last made an active choice (on advice from her Privy Councillors) as to the identity of the Prime Minister in 1974, though the last veto of a ministerial appointment was in 1892, the last dismissal and appointment of an appointment against convention was in 1834, and the last actual withholding of royal assent to legislation was 1708, though it remained a possibility even in 1914. --PalaceGuard008 (Talk) 23:24, 16 October 2012 (UTC)[reply]

Foreign relations of Gujarat

I found this article from something that was linked by a discussion over at WP:ANI. It leaves me very confused — why would Germany and the UK even consider engaging in foreign relations with Modi's government? Why wouldn't they just go through New Delhi for all of their foreign relations? Note that I've never heard of Modi before; perhaps there's a very big issue that I didn't notice in his article and that I couldn't find through a Google search. Nyttend (talk) 23:51, 16 October 2012 (UTC)[reply]

October 17